SlideShare a Scribd company logo
1 of 64
ĐẠI HOC THÁI NGUYÊN
TRƯ NG ĐẠI HOC KHOA HOC
Tải tài liệu tại sividoc.com
Viết đề tài giá sinh viên – ZALO:0973.287.149-TEAMLUANVAN.COM
HOÀNG TH± NGA
M T SO ỨNG DỤNG CỦA CÔNG THỨC
N I SUY LAGRANGE VÀ HERMITE
LU N VĂN THẠC SĨ TOÁN HOC
THÁI NGUYÊN - 2018
ĐẠI HOC THÁI NGUYÊN
TRƯ NG ĐẠI HOC KHOA HOC
Tải tài liệu tại sividoc.com
Viết đề tài giá sinh viên – ZALO:0973.287.149-TEAMLUANVAN.COM
HOÀNG TH± NGA
M T SO ỨNG DỤNG CỦA CÔNG THỨC
N I SUY LAGRANGE VÀ HERMITE
Chuyên ngành: PHƯƠNG PHÁP TOÁN SƠ CAP
Mã so: 8 46 01 13
LU N VĂN THẠC SĨ TOÁN HOC
Người hướng dan khoa hoc:
GS.TSKH. Nguyen Văn M u
THÁI NGUYÊN - 2018
i
Viết đề tài giá sinh viên – ZALO:0973.287.149-TEAMLUANVAN.COM
MỤC LỤC
M ĐAU ii
Chương 1. N i suy Lagrange và n i suy Hermite 1
1.1 Bài toán n®i suy Lagrange...................................................................... 1
1.2 Bài toán n®i suy Hermite........................................................................ 9
1.3 Bài toán n®i suy Lagrange - Newton.....................................................18
Chương 2. Ứng dnng n i suy tính nguyên hàm và tích phân các hàm
phân thfíc 21
2.1 Nguyên hàm của hàm phân thác với các cực điem đơn........................21
2.2 Nguyên hàm của hàm phân thác với các cực điem b c tùy ý...............26
Chương 3. M t so dạng toán liên quan 43
3.1 M®t so bài toán ve đa thác nh n giá trị nguyên...................................43
3.2 M®t so bài toán xác định đa thác .........................................................50
3.2.1 Tìm đa thác khi biet các nghi m của nó. ..................................50
3.2.2 Sả dụng công thác n®i suy Lagrange đe xác định h so của
đa thác. ......................................................................................53
3.2.3 M®t so bài toán xác định đa thác khác không liên quan đen
các công thác n®i suy ..................................................................... 56
KET LU N 59
TÀI LI U THAM KHẢO 59
ii
Viết đề tài giá sinh viên – ZALO:0973.287.149-TEAMLUANVAN.COM
M ĐAU
Trong các kì thi hoc sinh giỏi toán các cap, Olympic Toán sinh viên, các bài
toán liên quan tới đa thác thường xuyên được đe c p. Nhǎng dạng toán này
thường được xem là thu®c loại khó, hơn nǎa phan kien thác ve n®i suy đa thác
lại không nam trong chương trình chính thác của giáo trình Đại so và Giải tích
b c trung hoc phő thông.
Như ta đã biet, công thác n®i suy Lagrange đã được đe c p ở b c phő thông.
Tuy nhiên công thác n®i suy Hermite chỉ có trong các tài li u chuyên khảo. Vì
v y, tôi chon đe tài lu n văn ”M®t so áng dụng của công thác n®i suy Lagrange
và Hermite”.
Lu n văn nham cung cap m®t so áng dụng của công thác n®i suy Lagrange và
Hermite đe tìm nguyên hàm của hàm phân thác.
Lu n văn gom phan mở đau, ket lu n và 3 chương.
Chương 1. N®i suy Lagrange và n®i suy Hermite.
Chương 2. Úng dụng n®i suy tính nguyên hàm và tích phân các hàm phân thác
Chương 3. M®t so dạng toán liên quan.
Tiep theo, trong các chương đeu trình bày m®t h thong bài t p áp dụng giải
các đe thi HSG quoc gia và Olympic liên quan.
Lu n văn được thực hi n và hoàn thành tại Trường Đại hoc Khoa hoc, Đại hoc
Thái Nguyên dưới sự hướng dan của GS.TSKH. Nguyen Văn M u. Xin được gải
lời cảm ơn chân thành và sâu sac đen Thay, người đã t n tình hướng dan và chỉ
đạo tác giả t p dượt nghiên cáu khoa hoc trong suot quá trình tìm hieu tài li u,
viet và hoàn thi n Lu n văn. Đong thời em xin chân thành cảm ơn các quý thay
cô trong B® môn toán, Khoa Khoa hoc Tự nhiên, các Thay Cô trường Đại hoc
Khoa hoc Tự nhiên Hà N®i, các Thay Cô Vi n Toán hoc đã t n tình giảng dạy,
quan tâm và tạo moi đieu ki n thu n lợi ve thủ tục hành chính đe em hoàn thành
khoá hoc và bảo v lu n văn Thạc sĩ. Tôi cũng chân thành cảm ơn gia đình, bạn
bè và cơ quan, đoàn the nơi tôi công tác là Trường Trung hoc Phő thông Thu
Sơn, Sở Giáo dục và Đào tạo Hải Phòng, đã tạo moi đieu ki n ve v t chat lan
tinh than trong quá trình hoc t p, nghiên cáu và viet lu n văn.
Thái Nguyên, tháng 5 năm 2018
Tác giả
Hoàng Thị Nga
Viết đề tài giá sinh viên – ZALO:0973.287.149-TEAMLUANVAN.COM
1
Chương 1. N i suy Lagrange và n i suy Hermite
Chương này được dành đe trình bày ve các bài toán n®i suy Lagrange, bài toán
n®i suy Hermite và bài toán n®i suy Lagrange-Newton, tà định lí, h quả cho đen
m®t so ví dụ tính toán cụ the.
1.1 Bài toán n i suy Lagrange
Trong m®t so trường hợp, đe tính tőng hǎu hạn các phân thác, người ta thường
sả dụng m®t so tính chat của đa thác, đ c bi t là công thác n®i suy Lagrange.
Dưới đây là m®t so đong nhat thác cơ bản và áp dụng của chúng.
Định lj 1.1 (Đong nhat thác Lagrange). Neu x1, x2, . . . , xm là m giá tr tuỳ ý,
đôi m®t khác nhau và f(x) là đa thúc b¾c nhó thua m thì ta có đong nhat thúc
sau
f(x) = f(x1 )
(x − x2)(x − x3) . . . (x − xm)
+
(x1 − x2)(x1 − x3) . . . (x1 − xm)
+f(x2)
(x − x1)(x − x3) . . . (x − xm)
(x2 − x1)(x2 − x3) . . . (x2 − xm)
+ · · · + f(xm )
(x − x1)(x − x2) . . . (x − xm−1)
. (1.1)
(xm − x1)(xm − x2) . . . (xm − xm−1)
ChGng minh. Ta can cháng minh công thác
f(x) − f(x1 )
(x − x2)(x − x3) . . . (x − xm)
(x1 − x2)(x1 − x3) . . . (x1 − xm)
−f(x2)
(x − x1)(x − x3) . . . (x − xm)
(x2 − x1)(x2 − x3) . . . (x2 − xm)
— · · · − f(xm )
(x − x1)(x − x2) . . . (x − xm−1)
(xm − x1)(xm − x2) . . . (xm − xm−1)
≡ 0.
Nh n xét rang ve trái của công thác là m®t đa thác b c không quá m − 1 và có
ít nhat m nghi m phân bi t là x1, x2, . . . , xm. V y đa thác trên phải đong nhat
bang 0.
H quả 1.1. Tà Định lý 1.1, ta thu được các đong nhat thác sau đây.
−
2
Viết đề tài giá sinh viên – ZALO:0973.287.149-TEAMLUANVAN.COM
2
m 1 m 2 m m−1
(x −
√
3)(x −
√
5)(x −
√
7) (x −
√
2)(x −
√
5)(x −
√
7)
(
√
2 −
√
3)(
√
2 −
√
5)(
√
2 −
√
7)
+
(
√
3 −
√
2)(
√
3 −
√
5)(
√
3 −
√
7)
(x −
√
2)(x −
√
3)(x −
√
7) (x −
√
2)(x −
√
3)(x −
√
5)
+
(
√
5 −
√
2)(
√
5 −
√
3)(
√
5 −
√
7)
+
(
√
7 −
√
2)(
√
7 −
√
3)(
√
7 −
√
5)
≡ 1,
a2 (x − b)(x − c)
+ b2 (x − c)(x − a)
+ c2 (x − a)(x − b) ≡ x (a < b < c).
(a − b)(a − c) (b − c)(b − a) (c − a)(c − b)
Định lj 1.2. Giả sả f (x) là m®t đa thác b c nhỏ thua ho c bang m − 2 (m > 2)
và x1, x2, . . . , xm là m giá trị đôi m®t khác nhau cho trước tuỳ ý. Khi đó, ta
có đong nhat thác
f(x1) f(x2)
+
(x1 − x2)(x1 − x3) . . . (x1 − xm) (x2 − x1)(x2 − x3) . . . (x2 − xm)
f (xm)
+ · · · +
(x − x )(x − x ) . . . (x − x )
≡ 0.
ChGng minh. Nh n xét rang ve trái của đȁng thác đã cho chính là h so của
hạng tả áng với b c m − 1 trong cách viet chính tac của đa thác f (x). Đong nhat
các h so đong b c ta có ngay đieu phải cháng minh.
Dưới đây, ta xét m®t so áng dụng trực tiep của đong nhat thác Lagrange.
Ví dn 1.1. Tính tőng
cos 1o
cos 2o
S =
(cos 1o − cos 2o)(cos 1o − cos 3o)
+
(cos 2o − cos 1o)(cos 2o − cos 3o)
cos 3o
+
(cos 3o − cos 1o)(cos 3o − cos 2o)
L i giải. Áp dụng Định lý 1.2, với
f(x) = x, x1 = cos 1o
, x2 = cos 2o
, x3 = cos 3o
,
ta thu được S = 0.
Ví dn 1.2. Ta có các đong nhat thác
b + c + d
(b − a)(c − a)(d − a)(x − a)
d + a + b
c + d + a
+ +
(c − b)(d − b)(a − b)(x − b)
a + b + c
+
(d − c)(a − c)(b − c)(x − c) (a − d)(b − d)(c − d)(x − d)
x − a − b − c − d
.
(x − a)(x − b)(x − c)(x − d)
≡
3
Viết đề tài giá sinh viên – ZALO:0973.287.149-TEAMLUANVAN.COM
· · ·
L i giải. Th t v y, ta can cháng minh
(a + b + c + d) − a
+
(a + b + c + d) − b
+
(a − b)(a − c)(a − d)(a − x) (b − a)(b − c)(b − d)(b − x)
+
(a + b + c + d) − c
+
(a + b + c + d) − d
+
(c − a)(c − b)(c − d)(c − x) (d − a)(d − b)(d − c)(d − x)
+
(a + b + c + d) − x
(x − a)(x − b)(x − c)(x − d)
Ta có, với đa thác b c nhat
= 0.
f(y) = a + b + c + d − y, y1 = a, y2 = b, y3 = c, y4 = d, y5 = x,
theo Định lý 1.2 ta sě thu được ngay đieu phải cháng minh.
Định lj 1.3. Cho x1, x2, . . . , xm là m giá trị tuỳ ý đôi m®t khác nhau. Đ t
xn xn
Sn = 1
+ 2
(x1 − x2)(x1 − x3) . . . (x1 − xm) (x2 − x1)(x2 − x3) . . . (x2 − xm)
xn
Khi đó
+ + m
.
(xm − x1)(xm − x2) . . . (xm − xm−1)
a) Sn = 0 neu 0 ≤ n < m − 1,
b) Sm−1 = 1,
c) Sm+k bang tőng các tích, moi tích có k + 1 thàa so (giong nhau ho c khác
nhau) lay trong các so x1, x2, . . . , xm.
ChGng minh.
a) Theo Định lý 1.2, với
f(x) = 1, x, x2
, . . . , xm−2
,
ta được ngay S0 = S1 = . . . = Sm−2 = 0.
b) Đe cháng minh Sm−1 = 1, ta chỉ can thay f (x) trong Định lý 1.2 bởi xm−1
,
roi so sánh h so của hạng tả b c m − 1 ở hai ve của đong nhat thác vàa thu
được.
c) Đe tính Sn khi n > m − 1 ta làm như sau:
Giả sả x1, x2, . . . , xm thoả mãn phương trình b c m
αm
+ p1.αm−1
+ p2.αm−2
+ · · · + pm−1.α + pm = 0,
4
Viết đề tài giá sinh viên – ZALO:0973.287.149-TEAMLUANVAN.COM
trong đó
−p1 = x1 + x2 + · · · + xm
p2 = x1x2 + x1x3 + · · · + xm−1xm
. . . . . . . . .
(−1)k
.pk = x1x2x3 . . . xk + · · ·
Nhân cả hai ve của phương trình trên với αk
, ta được
αm+k
+ p1.αm+k−1
+ p2αm+k−2
+ · · · + pm−1.αk+1
+ pm.αk
= 0.
Thay α trong đȁng thác này lan lượt bởi x1, x2, . . . , xm; và lan lượt chia đȁng
thác thá nhat cho
đȁng thác thá hai cho
(x1 − x2)(x1 − x3) . . . (x1 − xm),
(x2 − x1)(x2 − x3) . . . (x2 − xm)
. . ., roi c®ng ve với ve các đȁng thác mới vàa nh n được, ta thu được
Sm+k + p1.Sm+k−1 + · · · + pm−1.Sk+1 + pm.Sk = 0. (1.2)
Đ t k = 0, ta thu được Sm + p1Sm−1 = 0.
Do đó Sm = −p1 = x1 + x2 + · · · + xm.
Nhờ đȁng thác (1.2) ta sě lan lượt tính tiep các bieu thác Sm+1, Sm+2, . . .
Ta đ t lan lượt
(x1
(x2
— x2
— x1
)(x1
)(x2
1
— x3
1
— x3
1
) . . . (x1
) . . . (x2
.
— xm
— xm
)
= α1;
)
= α2;
Khi đó ta có
(xm — x1)(xm — x2) . . . (xm — xm−1 )
= αm.
Sn = xn
α1 + xn
α2 + · · · + xn
αm.
Xét
1 2 m
α1 α2 αm
P = + + · · · + .
1 − x1z 1 − x2z 1 − xmz
Dùng công thác của cap so nhân với giả thiet rang z được chon sao cho
|x1z| < 1, |x2z| < 1, . . . , |xmz| < 1,
5
Viết đề tài giá sinh viên – ZALO:0973.287.149-TEAMLUANVAN.COM
m
ta khai trien tőng P thành chuoi vô hạn như sau:
P = α1(1 + x1z + x2
z2
+ · · · ) + α2(1 + x2z + x2
z2
+ · · · )+
hay
1 2
+ · · · + αm(1 + xm.z + x2
z2
+ · · · )
P = (α1 + α2 + · · · + αm) + (x1α1 + x2α2 + · · · + xmαm)z +
+(x2
α1 + x2
α2 + · · · + x2
αm)z2
+ · · ·
tác là
1 2 m
Đe cho gon, ta đ t
P = S0 + S1z + S2z2
+ S3z3
+ · · · .
(1 − x1z)(1 − x2z) . . . (1 − xmz) = Q.
Khai trien Q theo luy thàa của z, ta có the viet
Q = 1 − δ1z + δ2z2
+ · · · + (−1)m
δmzm
,
trong đó
δ1 = x1 + x2 + · · · + xm,
δ2 = x1x2 + x1x3 + · · · + xm−1xm..
. . . . . . . . .
Tiep theo, nhân cả hai ve của đȁng thác thá hai với
(1 − x1z)(1 − x2z) . . . (1 − xm.z),
ta có
PQ = α1(1 − x2z)(1 − x3z) . . . (1 − xmz)+
α2(1 − x1z)(1 − x3z) . . . (1 − xm.z)+
α3(1 − x1z)(1 − x2z)(1 − x4z) . . . (1 − xmz) + · · · +
αm(1 − x1z)(1 − x2z) . . . (1 − xm−1z).
Như v y PQ là m®t đa thác b c m − 1 đoi với z. Ta sě cháng minh rang nó
chính là zm−1
, tác là có đong nhat thác
PQ = zm−1
.
6
Viết đề tài giá sinh viên – ZALO:0973.287.149-TEAMLUANVAN.COM
1
x1 x1 x1 xm−1
xm−1
xm−1
Th t v y, bieu thác PQ − zm−1
tri t tiêu khi
vì, chȁng hạn, với
1
z = ,
x1
1 1
, . . . ,
x2 xm
1
z =
x1
thì
α 1 −
x2
1 −
x3
· · · 1 −
xm
−
1 1 1
= − = 0.
V y nên PQ − zm−1
= 0. Do đó
hay
zm−1
= P
Q
zm−1 1
1 − δ1z + δ2z2 − . . . + (−1)mδmzm
= S0 + S1 z + · · · + Sm−1zm−1
+ · · ·
Neu khai trien ve trái thành chuoi vô hạn theo luy thàa của z thì chuoi này bat
đau bang hạng tả cháa zm−1
. Vì v y, h so của các hạng tả b c 0, 1, 2, . . . , m − 2
trong ve phải bang không, tác là
S0 = S1 = . . . = Sm−2 = 0.
Ngoài ra, h so của hạng tả áng với b c m = 1 ở ve trái bang 1.
V y Sm−1 = 1.
Bây giờ đȁng thác can cháng minh có dạng sau:
1 − δ1 .z + δ2
zm−1
.z2 − · · · (−1)mδm .zm
= zm−1 + Sm.zm
+ Sm+1.zm+1
+ · · ·
Uớc lượng cả hai ve cho zm−1
, ta thu được
ho c
1
1 − δ1z + δ2z2 − · · · + (−1)mδmzm
= 1 + Sm z + Sm+1z2
+ · · ·
1 = (1 − δ1z + δ2z2
− · · · + (−1)m
δm.zm
)(1 + Smz + Sm+1z2
+ · · · )
1
1
1
7
Viết đề tài giá sinh viên – ZALO:0973.287.149-TEAMLUANVAN.COM
− − −
1 2 m 1 2 m
1 2 m
p=0 q=0 s=0
Khai trien ve phải theo lũy thàa của z và so sánh các h so ở hai ve, ta được
Sm − δ1 = 0,
δ2 − δ1.Sm + Sm+1 = 0,
. . . . . . . . .
Như v y, ta có the tính được Sm, Sm+1, Sm+2, . . .
Nham thiet l p được m nh đe mở r®ng cau trúc của Sm+k, ta xét
1 1
=
Q 1 x1.z
∞
1
.
1 x2.z
∞
.. .
1
1 xm.z
∞
=
Σ
xp
.zp
.
Σ
xq
.zq
· · ·
Σ
xs
zs
=
Σ
xp
xq
. . . xs
zp+q+···+s
.
M t khác
1
= 1 + Sm
Q
z + Sm+1z2
+ · · · + Sm+k zk+1
+ · · · ,
nên ta được
Sm+k =
Σ
xp
xq
. . . xs
Vì v y, ta thu được ket quả cuoi cùng Sm+k bang tőng các tích, moi tích có
k + 1 thàa so (giong nhau ho c khác nhau) lay trong các so x1, x2, . . . , xm. Nói
riêng
Sm+1 = x
Sm+2 = x
−1xm
−1xm
+x1x2x3 + · · · + xm−2xm−1xm, (đieu phải cháng minh).
H quả 1.2. Giả sả
ak bk ck
Khi đó
Sk = + + .
(a − b)(a − c) (b − a)(b − c) (c − a)(c − b)
S0 = S1 = 0 , S2 = 1 , S3 = a + b + c,
S4 = a2
+ b2
+ c2
+ ab + bc + ca,
S5 = a3
+ b3
+ c3
+ a2
b + b2
c + c2
a + ab2
+ bc2
+ ca2
+ abc.
p+q+···+s=k+1
2
+ x2
+ · · · + x2
+ x1x2 + x1x3 + · · · + xm
1 2 m
3
+ x3
+ · · · + x3
+ x2
x2 + x2
x3 + · · · + x2
1 2 m 1 1 m
8
Viết đề tài giá sinh viên – ZALO:0973.287.149-TEAMLUANVAN.COM
N
Σ
j=1 j=1
H quả 1.3. Giả sả
ak bk ck
Tk = + + +
(a − b)(a − c)(a − d) (b − a)(b − c)(b − d) (c − a)(c − b)(c − d)
dk
Khi đó
+ .
(d − a)(d − b)(d − c)
T0 = T1 = T2 = 0 , T3 = 1 , T4 = a + b + c + d.
Bây giờ ta chuyen sang khảo sát bài toán n®i suy Lagrange dưới ngôn ngǎ tőng
quát.
Bài toán 1.1 (Bài toán n®i suy Lagrange). Cho xi, ai ∈ R, với xi /= xj ∀i
j, (i, j = 1, 2, . . . , N). Hãy xác định đa thác L(x) có b c deg L(x) ≤ N − 1 thỏa
mãn đieu ki n
L(xi) = ai, ∀i = 1, 2, . . . , N. (1.3)
L i giải. Đe đơn giản, ký hi u
Li(x) =
Khi đó, de thay rang
j=
Y
1,
j=
/
x − xj
, (i = 1, 2, . . . , N).
i
xi − xj
hay Li(xj) = δij.
Li(xj ) =
1 khi i = j
0 khi i /= j
Tiep theo, ta cháng minh rang đa thác
N
L(x) = aiLi(x) (1.4)
i=1
là đa thác duy nhat thỏa mãn đieu ki n của bài toán n®i suy Lagrange (1.3), và
ta goi đa thác này là đa thúc n®i suy Lagrange.
Th t v y, de thay rang
Ngoài ra, ta có
deg L(x) ≤ N − 1.
N N
L(xi) =
Σ
ajLj(xi) =
Σ
ajδij
9
Viết đề tài giá sinh viên – ZALO:0973.287.149-TEAMLUANVAN.COM
Y
Y
hay
L(xi) = ai, ∀i = 1, 2, . . . , N.
Cuoi cùng, neu có đa thác L∗(x), có b c deg L∗(x) với deg L∗(x) ≤ N − 1
cũng thỏa mãn đieu ki n của bài toán (1.3) thì khi đó, đa thác
P(x) = L(x) − L∗(x)
cũng có b c deg P(x) ≤ N − 1 và thỏa mãn
P(xi) = 0, ∀i = 1, 2, . . . , N.
Tác là P (x) là đa thác có b c deg P (x) với deg P (x) ≤ N − 1 mà lại có ít
nhat N nghi m phân bi t x1, x2, . . . , xN , nên P(x) ≡ 0, và do đó L(x) = L∗(x).
Tà bài toán n®i suy Lagrange ta có nh n xét sau:
Nh n xét 1.1. Ve m t hình hoc, vi c xây dựng đa thác n®i suy Lagrange (1.3)
có nghĩa là xây dựng m®t đa thác m®t bien b c không quá N − 1 đi qua tat cả
các điem Mi(xi, yi), ∀i = 1, 2, . . . , N cho trước.
1.2 Bài toán n i suy Hermite
Bài toán n®i suy Newton là m®t mở r®ng tự nhiên của đong nhat thác
Taylor và tương áng, khai trien Taylor - Gontcharov là mở r®ng khai trien Taylor
cő đien. Bây giờ ta chuyen sang xét bài toán n®i suy Hermite là m®t mở r®ng tự
nhiên của bài toán n®i suy Lagrange và Taylor. Với đa thác
n
L(x) = (x − xj), xi
j=1
xj khi i j, i, j = 1, 2, . . . , n
thì đieu ki n Lagrange
L(xj) = 0, j = 1, 2, . . . , n
là đieu ki n tự nhiên đe xác định đa thác đơn P (x) ( đa thác có các nghi m đơn).
Khi P (x) có nghi m b®i thì đieu ki n Lagrange không đủ đe xác định P (x). Vì
the, ta can các đieu ki n tőng quát hơn đe đảm bảo ton tại duy nhat m®t đa thác
dạng
n
H(x) = (x − xj)αj
, xi
j=1
xj khi i j, i, j = 1, 2, . . . , n.
10
Viết đề tài giá sinh viên – ZALO:0973.287.149-TEAMLUANVAN.COM
Y
n
Σ Σ
Σ n p −1
j
l
i
(x − xi)pi
i
W(x)
i
(x − xi)pi−l
i i
(x − xj)pj−l
Rõ ràng đa thác H(x) có deg H(x) = α1 + α2 + · · · + αn và
H(k)
(xj) = 0, k = 0, 1, . . . , αj − 1; j = 1, 2, . . . , n.
Vì the, ta có the phát bieu bài toán n®i suy Hermite dưới dạng sau.
Bài toán 1.2 (N®i suy Hermite). Cho xi, aki ∈ R, với i = 1, 2, . . . , n; k =
0, 1, . . . , pi − 1 và xi /= xj ∀i j, trong đó p1 + p2 + · · · + pn = N.
Hãy xác định đa thác H(x) có b c deg H(x) ≤ N − 1 thỏa mãn đieu ki n
H(k)
(xi) = aki, ∀i = 1, 2, . . . , n; ∀k = 0, 1, . . . , pi − 1. (1.5)
L i giải. Ký hi u
n
W(x) = (x − xj)pj
j=1
W (x)
W (x) = =
Y
(x − x )pj
.
Tiep theo, giả sả H(x) là đa thác có b c deg H(x) với deg H(x) ≤ N − 1 và
thỏa mãn đieu ki n bài toán.
Ta can xác định các h so αli ∈ R sao cho đȁng thác sau được thoả mãn
n pi−1
H(x)
W (x)
=
αli
.
(x − xi)pi−l
Ta có
i=1 l=0
n pi−1
H(x)
= (x − x )pi
H(x)
= (x − x )pi
Σ Σ αli
pi−1
= α (x − x )l
+ (x − x )pi
Σ Σ αlj
.
Trong các phép bien đői tiep theo, đe ý rang
l p l p p
(fg)(l)
=
Σ
Ck
f (k)
g(l−k)
và
Σ Σ
Aki =
Σ Σ
Aki,
ta thu được
k=0 l=0 k=0 k=0 l=k
h H(x) i(l)
= l!αli.
Wi(x) (x=xi)
l=0
i
j=1, j
l=0
l=0
i=1
i
j=1, j
j
Wi(x)
li
11
Viết đề tài giá sinh viên – ZALO:0973.287.149-TEAMLUANVAN.COM
Σ Σ
n
ki
ki
Σ
Σ
Σ
i
Σ
Suy ra
αli =
Σ
aki
h 1 i(l−k)
.
Do đó
k=0
k!(l − k)! Wi(x) (x=xi)
H(x)
=
pi−1 l
aki
h 1 i(l−k) 1
W (x) i=1 l=0 k=0 k!(l − k)! Wi(x) (x=xi) (x − xi)pi−l
Σ p
Σ
i−1 p
Σ
i−1
a h 1 i(l−k) 1
Suy ra
i=1 k=0 l=k k!(l − k)! Wi(x) (x=xi) (x − xi)pi−l
H(x) = W (x)
Σ p
Σ
i−1 p
Σ
i−1
a h 1 i(l−k) 1
hay
i=1 k=0 l=k k!(l − k)! Wi(x) (x=xi) (x − xi)pi−l
H(x) =
Σ pi−1
aki (x − xi)k
Wi(x)
p
Σ
i−1 h 1 i(l−k)
(x − xi)l−k
i=1
k!
k=0 l=k
Wi(x) (x=xi) (l − k)!
Đői chỉ so ở tőng cuoi cùng của đȁng thác trên, ta thu được
H(x) =
Σ pi−1
aki (x − xi)k
Wi(x)
pi
Σ
−1−k h 1 i(l)
(x − xi)l
hay
i=1
k!
k=0 l=0
Wi(x) (x=xi) l!
H(x) =
Σ pi−1
aki (x − xi)k
Wi(x) T
, 1 ,(pi−1−k)
,
trong đó
i=1
k!
k=0
Wi(x) (x=xi)
T
, 1 ,(pi−1−k)
pi
Σ
−1−k h 1 i(l)
(x − x )l
Wi(x) (x=xi)
l=0
Wi(x) (x=xi) l!
1
là đoạn khai trien Taylor đen cap thá (pi − 1 − k) tại x = xi của hàm so
Ký hi u
.
Wi(x)
Hki(x) = (x − xi)k
Wi(x) T
, 1 ,(pi−1−k)
.
k!
Khi đó, de thay rang
Wi(x) (x=xi)
deg Hki(x) ≤ k + (N − pi) + (pi − 1 − k) = N − 1,
=
l
n
n
n
n
n
= .
.
12
Viết đề tài giá sinh viên – ZALO:0973.287.149-TEAMLUANVAN.COM
ki
ki
Σ
Σ
Σ
lj
Σ
H(l)
(xj
hay H(l)
(xj) = δklδij.
) =
1 neu k = l và i = j
0 neu k /= l ho c i /= j
Bây giờ, ta sě cháng minh rang
pi−1
akiHki(x)
=
Σ
pi−1
aki (x − xi)k
Wi(x) T
, 1 ,(pi−1−k)
(1.6)
hay
i=1 k=0 i=1
k!
k=0
wi(x) (x=xi)
H(x) =
Σ pi−1
aki (x − xi)k
Wi(x) T
, 1 ,(pi−1−k)
i=1
k!
k=0
wi(x) (x=xi)
là đa thác duy nhat thỏa mãn đieu ki n của bài toán n®i suy Hermite 1.5 và ta
goi đa thác (1.6) là đa thúc n®i suy Hermite.
Th t v y, de thay rang deg H(x) ≤ N − 1 và
n pj −1 n pj −1
H(k)
(xi) =
Σ Σ
aljH(k)
(xi) =
Σ Σ
aljδklδij.
Suy ra
j=1 l=0 j=1 l=0
H(k)
(xi) = aki, (∀i = 1, 2, . . . , n; ∀k = 0, 1, . . . , pi − 1).
Cuoi cùng, ta cháng minh tính duy nhat nghi m của bài toán n®i suy Hermite.
Giả sả ton tại đa thác H∗(x), có b c deg H∗(x) với deg H∗(x) ≤ N − 1,
cũng thỏa mãn đieu ki n của bài toán n®i suy Hermite. Khi đó, đa thác P (x) =
H(x) − H∗(x) cũng có b c deg P(x) ≤ N − 1 và thỏa mãn đieu ki n
P(k)
(xi) = 0, (∀i = 1, 2, . . . , n; ∀k = 0, 1, . . . , pi − 1).
Khi đó, theo như cách xây dựng đa thác H(x) ở phan trên, áng với trường hợp
aki = 0, ∀i = 1, 2, . . . , n; ∀k = 0, 1, . . . , pi − 1, ta suy ra P(x) ≡ 0 và do đó
H(x) = H∗(x).
Chú ý rang đa thác n®i suy Taylor và đa thác n®i suy Lagrange là nhǎng
trường hợp riêng của đa thác n®i suy Hermite.
Th t v y, đa thác n®i suy Taylor là trường hợp riêng của đa thác n®i suy
Hermite vì trong đa thác n®i suy Hermite (1.5) áng với n = 1 thì ta có
p1 = N, W(x) = (x − x1)p1
, W1(x) ≡ 1.
n n
n
13
Viết đề tài giá sinh viên – ZALO:0973.287.149-TEAMLUANVAN.COM
Σ
N
Σ
Σ
i
.
1
n
Wi(x) a0i T
(x=x )
+ a1i(x − xi) T
Suy ra
V y nên
Hk1(x) =
N−1
(x − x1)k
k!
k
H(x) = ak1
k=0
(x − x1)
k!
≡ T(x).
Tương tự, đa thác n®i suy Lagrange là trường hợp riêng của đa thác n®i suy
Hermite vì trong đa thác n®i suy Hermite (1.5), áng với k = 0, ta có
pi = 1 ∀i = 1, 2, . . . , n, và n = N.
Suy ra
V y nên
H0i
(x) =
Wi(x)
=
Wi(xi)
N
j=
Y
1,
j=
/
x − xj
i
xi − xj
= Li(x).
H(x) = a0iLi(x) ≡ L(x).
i=1
Tiep theo, ta xét m®t so trường hợp riêng đơn giản của đa thác n®i suy Hermite.
Xét đa thác n®i suy Hermite (1.5)
H(x) =
Σ pi−1
aki (x − xi)k
Wi(x) T
, 1 ,(pi−1−k)
,
trong đó
i=1
k!
k=0
Wi(x) (x=xi)
T
, 1 ,(pi−1−k)
pi
Σ
−1−k h 1 i(l)
(x − x )l
Wi(x) (x=xi)
l=0 Wi(x) (x=xi) l!
Ta xét m®t trường hợp khi h đieu ki n chỉ cháa đạo hàm b c nhat.
Trong đa thác n®i suy Hermite (1.5), neu pi = 2, ∀i = 1, 2, . . . , n thì khi đó
k = 0, 1 và ta có
H(x) =
Σ Σ
aki (x − xi)k
Wi(x) T
, 1 ,(1−k)
.
Suy ra
i=1
k!
k=0
Wi(x) (x=xi)
Σ h , 1 ,(1) , 1 ,(0) i
(x=xi)
Wi(x)
i
Wi(x)
i=1
=
.
n
n
H(x) =
14
Viết đề tài giá sinh viên – ZALO:0973.287.149-TEAMLUANVAN.COM
n
i
n
i i
0
0 0 0 0 0 0
Y
i i i
hay
H(x) =
Σ
Wi(x) h
a0i 1 −
Wi
′
(xi) (x − xi) + a1i(x − xi)
i
=
i=1
Wi(xi) Wi(xi)
=
Σ Wi(x) h
a0i −
Wi
a0i
′
(xi) — a1i (x − xi)
i
.
i=1
Wi(xi) Wi(xi)
Cuoi cùng, đe ý rang ta có các đong nhat thác
n 2
Wi(x)
Wi(xi)
=
j=1,
j=
/
(x − xj)
(xi − xj)2
= L2
(x),
Wi
′
(xi) = 2
h Y
x − xj
i′
= 2L
′
(xi).
W (x ) x − x x=x
i
j=1, j/=i
V y, ta đã thu được đa thác n®i suy Hermite trong trường hợp này như sau
H(x) =
Σ
i=1
L2
(x)
h
a0i − 2a0iL
′
(xi) − a1i (x − xi)
i
.
Tiep theo ta xét trường hợp khi h đieu ki n cháa các giá trị đạo hàm các cap
tại các nút n®i suy đeu bang 0 trà ra m®t điem. Trong đa thác n®i suy Hermite
(1.5), ta xét trường hợp đ c bi t, chȁng hạn ∃k0, i0 sao cho ak0i0 = a =
/ 0 v
à
aki = 0, ∀(k, i)
Khi đó, ta có
(k0, i0).
(x − xi )k0
, 1 ,(pi0 −1−k0)
Neu k0 = p0 − 1 và chon ak0i0 = (pi0 − 1)!Wi0(xi0) thì ta có
W (x)
V y, trong trường hợp đ c bi t
H(x) = .
x − xi0
H(k)
(xi ) = 0, ∀k = 0, 1, . . . , pi − 2; H(pi0
−1)
(xi ) = (pi − 1)!Wi (xi );
H(k)
(xi) = 0, ∀k = 0, 1, . . . , pi − 1, ∀i = 1, 2, . . . , n, i =
/ i0
thì đa thác n®i suy Hermite có dạng
W (x)
H(x) =
x − xi0
= (x−x1 )p1
· · · (x−x i0−1 )pi0 (x−xi )pi0−1
(x−x i0+1 )pi0 . . . (x−xn )pn
.
(x=xi0)
Wi0(x)
k0!
j
i
n
n
H(x) = ak0i0 Wi0(x) T .
i
0
15
Viết đề tài giá sinh viên – ZALO:0973.287.149-TEAMLUANVAN.COM
Y
0 0 0 0
Ngược lại, moi đa thác có dạng
H(x) = (x − x1)α1
(x − x2)α2
· · · (x − xn)αn
, xi
đeu là nghi m của các bài toán Hermite
xj ∀i =
/ j, i, j = 1, 2, . . . , n
H(k)
(xi) = 0, ∀k = 0, 1, . . . , αi − 1, i = 1, 2, . . . , n
H(αi0
)
(xi ) = αi !Wi (xi ).
Chȁng hạn, neu ta viet
α α α (x − x1)α1+1
(x − x2)α2
...(x − xn)αn
H(x) = (x − x1) 1
(x − x2) 2
...(x − xn) n
=
x − x1
thì H(x) là nghi m của bài toán Hermite sau đây
H(k)
(x1) = 0, ∀k = 0, 1, . . . , α1 − 1, H(α1)
(x1) = α1!W1(x1)
H(k)
(xi) = 0, ∀k = 0, 1, . . . , αi − 1, ∀i = 1, 2, . . . , n, i 1.
Tiep theo, trong phan này ta sě nêu m®t so ví dụ áp dụng các ky thu t cơ bản
đe xác định các đa thác khi biet m®t so đ c trưng của chúng dưới dạng nút n®i
suy.
Bài toán 1.3. Cho 0 < α < 1. Xác định tat cả các đa thác f (x) b c n (n ≥ 2)
sao cho ton tại dãy so r1, r2, . . . , rn (r1 < r2 < . . . < rn) thoả mãn các đieu ki n
sau
f(ri) = 0,
f′(αri + (1 − α)ri+1) = 0
(i = 1, 2, . . . , n).
L i giải. Nh n xét rang với a < b và x = αa +(1 − α)b ; α ∈ (0, 1) thì x ∈ (a, b).
Khi đó
1
p =
x − a
1
+
x − b
1
=
2α − 1
.
α(1 − α)(b − a)
1
Do v y p > 0 khi và chỉ khi α >
1
, p < 0 khi và chỉ khi α <
2 2
và p = 0 khi
và chỉ khi α =
nên
. Theo giả thiet thì
2
n
f(x) = c (x − ri)
i=1
f′(x)
f(x)
=
i=1
1
.
x − ri
n
Σ
16
Viết đề tài giá sinh viên – ZALO:0973.287.149-TEAMLUANVAN.COM
Σ
(−1) C P(k) = 0.
Σ
Σ
n−1
P (x) =
Σ
P (k)
(x − 0) · · · (x − (k − 1))(x − (k + 1)) · · · (x − (n − 1))
.
(−1)n
Cn
P(n) =
Σ
(−1)k
Ck
P(k).
(k − 0) · · · (k − (k − 1))(k − (k + 1)) · · · (k − (n − 1))
k=0
Với n ≥ 3 và 0 < α ≤
1
2 ta đ t x = αr1 + (1 − α)r2. Khi đó theo giả thiet thì
f′(x) = 0 và đong thời f(x) = 0. M t khác
mâu thuan.
f′(x)
f(x)
1
= +
x − r1
1
1
x − r2
+
i=3
1
x − ri
< 0,
Tương tự với n ≥ 3 và
1
< α < 1 ta cũng nh n được đieu vô lý.
2
Neu n = 2 và α /
= , thì tương tự như trên cũng dan đen đieu mâu thuan. Do
2
1
v y chỉ can xét trường hợp n = 2 và α = . Khi đó moi tam thác b c hai có 2
2
nghi m phân bi t đeu thoả mãn bài toán đã cho.
Bài toán 1.4. Xác định tat cả các đa thác P(x) b c nhỏ thua n và thoả mãn
đieu ki n
n
k k
n
k=0
L i giải. Áp dụng công thác n®i suy Lagrange với nút n®i suy xk = k, ta được
moi đa thác P(x) b c nhỏ thua n đeu có dạng
n−1
nên
P(x) = P(xk
k=0
n−1
)
(x − x0) · · · (x − xk−1)(x − xk+1) · · · (x − xn−1)
(xk − x0) · · · (xk − xk−1)(xk − xk+1) · · · (xk − xn−1)
Ta có
k=0 (k − 0) · · · (k − (k − 1))(k − (k + 1)) · · · (k − (n − 1))
n−1
P (n) = P(k)
(n − 0) · · · (n − k + 1)(n − k − 1) · · · 1
.
k!(−1)n−k(n − k − 1)!
Suy ra
k=0
n−1
n
V y đieu ki n bài toán được thỏa mãn.
n
k=0
Tóm lại, các đa thác can tìm có dạng
P (x) =
Σ
P (k)
(x − 0) · · · (x − (k − 1))(x − (k + 1)) · · · (x − (n − 1))
,
n
Σ
17
Viết đề tài giá sinh viên – ZALO:0973.287.149-TEAMLUANVAN.COM
−
P
.
≡
2q
q
p p
−
n
2 2 2
2q 2q
2q 2q 2q q
n
n
2 1000 10 10
L i giải. Xét đa thác
trong đó P(0), P(1), . . . , P(n − 1) là các giá trị tùy ý.
Tiep theo ta xét m®t so bài toán liên quan đen n®i suy theo xap xỉ Diophane.
Bài toán 1.5. Cháng minh rang ton tại đa thác Pn(x) b c n (n ≥ 1) với h so
nguyên sao cho
. (x) − . < ∀x ∈
h
,
i
.
1 1 1 9
P (x) =
1
[(2x 1)n
+ 1].
n
2
Ta thay ngay rang các h so của P (x) là các so nguyên. Khi đó với x ∈
h 1
,
9 i
ta có
n
. (x) −
1
. =
1
|2x − 1|n
≤
1
(0, 8)n
∀n ∈ N∗.
10 10
Chon n đe (0, 8)n
< 0, 002. Chȁng hạn n > 28 thì ta có ngay Pn(x) chính là đa
thác can tìm.
Bài toán 1.6. Cho hai so nguyên dương p, q. Cháng minh rang ton tại đa thác
Pn(x) b c n với h so nguyên sao cho
P p 1
(x) − . <
n
với moi x thu®c khoảng
1
,
3
.
q q2
L i giải. Với q = 1 thì ta chon P(x) p. Với q > 1 thì ta thay khoảng
I =
1
,
3
có đ® dài bang 1/q < 1. Chon m ∈ N sao cho
3 m
<
1
. Chon
n đủ lớn đe
và đ t
an
<
1
pq
với a = 1 −
1 m
P (x) =
p
[1 − (1 − qxm
)n
].
Khi đó rõ ràng Pn(x) là đa thác với h so nguyên và với x ∈ I thì
. (x) −
.
=
.
(1 − qxm
)n
. < p
an
<
1
n
q q
và
q q2
P (x) =
1
[(2x 1)n
+ 1].
n
2
Ta thay ngay rang các h so của Pn(x) là các so nguyên.
.
.
P
P
18
Viết đề tài giá sinh viên – ZALO:0973.287.149-TEAMLUANVAN.COM
Bài toán 1.7. Cháng minh rang không ton tại đa thác f(x) ∈ Z[x] mà f(2005) =
2005 và f(2007) = 2008.
L i giải. Giả sả ton tại đa thác
f(x) = anxn
+ an−1xn−1
+ · · · + a0, ai ∈ Z ∀i ∈ {0, 1, . . . , n}
thỏa mãn đieu ki n của bài toán.
Khi đó, ta có
f(2007) − f(2005) = an(2007n
− 2005n
)
+ an−1(2007n−1
− 2005n−1
) + · · · + a1(2007 − 2005)
chia het cho 2.
M t khác f(2007) − f(2005) = 2008 − 2005 = 3 và không chia het cho 2. V y
không ton tại đa thác f(x) thỏa mãn đieu ki n đe bài.
1.3 Bài toán n i suy Lagrange - Newton
Trước het ta xét bài toán n®i suy hon hợp Lagrange - Newton.
Bài toán 1.8 (N®i suy Lagrange - Newton). Cho xki, aki ∈ R, với xki /=
j; k = 0, 1, . . . , n − 1; i, j = 1, . . . , rk+1; trong đó
r0 = 0, r0 + r1 + · · · + rk = sk, r0 + r1 + · · · + rn = sn = N.
xkj ∀i
Hãy xác định đa thác f(x) có b c deg f(x) ≤ N − 1 và thỏa mãn đieu ki n
f(sk)
(xki) = aki, ∀k = 0, 1, . . . , n − 1, ∀i = 1, . . . , rk+1. (1.7)
L i giải. Ký hi u
rk+1
Lki(x) =
j=
Y
1, j/=i
x − xkj
xki − xkj
, (k = 0, 1, . . . , n − 1) và phép lay nguyên hàm .
Khi đó, de thay rang
deg Lki(x) ≤ rk+1 − 1,
Lki(xkj ) =
1 neu i = j
hay Lki(xkj) = δij.
0 neu i j
19
Viết đề tài giá sinh viên – ZALO:0973.287.149-TEAMLUANVAN.COM
k+1
Σ
Σ
Σ
Σ
n−1
n−1
n−1
Tiep theo, ta đ t yn(x) ≡ 0 và xây dựng dãy hàm
rk+1
yk(x) = Rsk
[aki − y(sk)
(xki)]Lki(x), (k = n − 1, . . . , 1, 0).
De thay rang
i=1
deg yk(x) ≤ N − 1 và deg yk(x) < sk+1.
Bây giờ, ta sě cháng minh rang đa thác
f(x) = y0(x) + y1(x) + · · · + yn−1(x)
chính là nghi m duy nhat của bài toán n®i suy Lagrange - Newton (1.7).
Th t v y, de thay rang deg f(x) ≤ N − 1. Ngoài ra, ta cũng có
f(sn−1)
(x(n−1)i) = (y0 + · · · + yn−2 + yn−1)(sn−1)
(x(n−1)i)
= y(sn−1)
(x
n−1
rn
(n−1)i)
= a(n−1)j.L(n−1)j(x(n−1)i)
j=1
rn
= a(n−1)j.δij = a(n−1)i
j=1
f(sn−2)
(x(n−2)i) = (y0 + · · · + yn−3 + yn−2 + yn−1)(sn−2)
(x(n−2)i)
= y
(sn−2)
(x(n−2)i) + y
(sn−2)
(x(n−2)i),
trong đó
y(sn−2)
(x
n−2 (n−2)i
rn−1
n−2
rn−1
) = [a
j=1
(n−2)j
n−1
— y(sn−2)
(x(n−2)j )].L(n−2)j (x(n−2)i)
=
Σ
[a(n−2)j − y
(sn−2)
(x(n−2)j)].δij
V y nên
j=1
= a(n−2)i − y
(sn−2)
(x(n−2)i).
f (sn−2)
(x(n−2)i) = a(n−2)i.
Bang cách tương tự, ta cháng minh được rang
f(sk)
(xki) = aki, ∀k = 0, 1, . . . , n − 1, ∀i = 1, . . . , rk+1.
20
Viết đề tài giá sinh viên – ZALO:0973.287.149-TEAMLUANVAN.COM
Cuoi cùng, neu có đa thác f∗(x) với b c deg f∗(x) ≤ N − 1 và thỏa mãn đieu
ki n của bài toán (1.7), thì khi đó đa thác
P(x) = f(x) − f∗(x)
cũng có b c deg P(x) ≤ N − 1 và thỏa mãn đieu ki n
P(sk)
(xki) = 0, ∀k = 0, 1, . . . , n − 1, ∀i = 1, . . . , rk+1.
Khi đó, theo như cách xây dựng đa thác f(x) ở phan trên, áng với trường hợp
aki = 0, ∀k = 0, 1, . . . , n − 1, ∀i = 1, . . . , rk+1,
ta có
yk = 0, ∀k = 0, 1, . . . , n − 1.
Suy ra P(x) ≡ 0, và do đó f(x) = f∗(x).
Tà bài toán trên ta có nh n xét sau:
Nh n xét 1.2. Neu k = 0, tương áng r2 = r3 = · · · = rn = 0, thì sk = 0. Khi
đó bài toán n®i suy Lagrange - Newton chính là bài toán n®i suy Lagrange quen
biet.
Neu i = 1, tương áng r1 = r2 = · · · = rn = 1, thì sk = k. Khi đó bài toán n®i
suy Lagrange - Newton chính là bài toán n®i suy Newton đã biet.
Viết đề tài giá sinh viên – ZALO:0973.287.149-TEAMLUANVAN.COM
Σ
0
p + 1
(
— m 1
)
21
Chương 2. Ứng dnng n i suy tính nguyên hàm và tích
phân các hàm phân thfíc
Chương này sě giới thi u cho chúng ta m®t so thu t toán tìm nguyên hàm của
m®t hàm so hǎu tỉ cho trước theo thá tự tà đơn giản đen phác tạp.
2.1 Nguyên hàm của hàm phân thfíc v i các cfic điem
đơn
Lớp hàm quen thu®c có nguyên hàm sơ cap là lớp các hàm so hǎu tỉ. Ket lu n
này được Laplace cháng minh lan đau tiên vào năm 1812 bang cách phân tích
m®t hàm so hǎu tỉ ra thành tőng của các phân thác đơn giản.
Định lj 2.1 (Laplace, 1812). Nguyên hàm của m®t hàm so hǎu tỉ là m®t hàm
sơ cap (đó là hàm so hǎu tỉ, ho c là tőng của m®t hàm so hǎu tỉ và m®t so hǎu
hạn của các logarit của nhǎng hàm so hǎu tỉ).
ChGng minh. Nh n xét rang moi đa thác Q(x) = b0xn
+ b1xn−1
+ · · · + bn đeu
có the được bieu dien được như sau
Q(x) = b0(x − a1)m1
(x − a2)m2
. . . (x − ar)mr
,
trong đó m1, . . . , mr là nhǎng so nguyên dương có tőng bang n và a1, . . . , ar là
nhǎng so thực ho c so phác.
Do đó moi hàm so hǎu tỉ R(x) có mau là đa thác Q(x) đeu có the bieu dien
dưới dạng
A0xp
+ A1xp−1
+ · · · + Ap +
Σ (
βs,1
+
βs,2 + · · · + βs,ms
)
.
Tà đó, suy ra
s=1 x − as (x − as) (x − as)ms
∫
R (x) dx = A
xp+1
+ A
xp
+ · · · + A x + C
r
+
s=1
βs,2
βs,1 ln (x − as) −
x − a
— · · ·
βs,ms
.
(ms − 1) (x − as) s−
2
r
s
1
p p
22
Viết đề tài giá sinh viên – ZALO:0973.287.149-TEAMLUANVAN.COM
m
∫ n−1
m 2
=
sin x 1 − sin x
cos xdx
∫ m n
2
2
Q
= −
4
+
2
ln .x − 1. + C
2
là hàm so sơ cap.
Xét các dạng toán minh hoa sau.
2x + 2
Bài toán 2.1. Ta có f (x) = là m®t hàm so hǎu tỉ. Nguyên hàm
x3 − 3x + 2
của nó có dạng là tőng của m®t hàm so hǎu tỉ và các logarit của nhǎng hàm so
hǎu tỉ.Th t v y, ta có
∫
2x + 2
dx =
∫
2x + 2
dx
x3 − 3x + 2 (x − 1)2
(x + 2)
=
∫
4
dx +
∫
2
dx −
∫
2
dx
3(x − 1)2 9 (x − 1) 9 (x + 2)
3 (x − 1)
trong đó C là hang so.
9 .x + 2.
Nh n xét 2.3. Neu m và n là các so nguyên, thì ta có các ket lu n sau.
i) Tích phân
∫
(1 − xn
)
1
dx là hàm so sơ cap neu và chỉ neu m = ±1 ho c
n = ±1 ho c m = −n.
ii) Tích phân sin xcos xdx là hàm so sơ cap với moi giá trị của m và n.
Th t v y, vì m, n là các so nguyên nên ta có
∫
sinm
x.cosn
xdx =
∫
sinm
x cos2
x
n−1
cos xdx
=
∫
um
1 − u2
n−1
du, (u = sin x) .
Tích phân trên là hàm so sơ cap neu và chỉ neu m lẻ ho c n lẻ ho c cả hai
n, m chȁn.
Định lý Laplace 2.1 cho ta ket lu n rang moi hàm hǎu tỉ đeu có nguyên hàm
sơ cap. Tuy nhiên, vi c tìm các nguyên hàm này không phải là m®t công vi c đơn
giản.
Sau đây là m®t so thu t toán giải bài toán trên.
Trong mục này, ta xét m®t thu t toán tìm nguyên hàm của hàm so hǎu tỉ có
P (x)
dạng f(x) = n
i=1
(x − xi)
trong đó P (x) là m®t đa thác, xi xj với moi i j.
Tà công thác trên, ta có m®t phương pháp tìm nguyên hàm của hàm so hǎu
tỉ mà mau thác b c n có n nghi m đơn phân bi t.
23
Viết đề tài giá sinh viên – ZALO:0973.287.149-TEAMLUANVAN.COM
Q
−
i=1
Đe tìm nguyên hàm của hàm so hǎu tỉ có dạng
P (x)
f(x) = n ,
(x xi)
i=1
trong đó P(x) là m®t đa thác và xi
tự như sau.
xj với moi i /= j. Ta tien hành theo trình
Bước 1. Áp dụng công thác n®i suy Lagrange ta phân tích P(x) thành dạng
Σ
n Q x − xi
tőng aj
j=1 i=1,i/=j x
j — xi
với aj = P(xj), với ∀j = 1, 2, . . . , n.
Q
n
Sau đây, xét m®t so ví dụ minh hoa.
Bài toán 2.2. Tìm nguyên hàm của hàm so f (x) =
2x2
.
(x + 1) (x − 1) (x − 2)
Lài giai. Đ t P (x) = 2x2
và xét ba nút n®i suy là x1 = −1; x2 = 1; x3 = 2. Ta
có a1 = 2, a2 = 2, a3 = 8. Áp dụng công thác n®i suy Lagrange, ta có
f (x) =
−1
−
1
+
8
.
3 (x + 1) x − 1 3 (x − 2)
Tà đây, ta suy ra nguyên hàm của hàm so can tìm là
∫
f (x) dx =
−1
ln |x + 1| − ln |x − 1| +
8
ln |x − 2| + C,
3
trong đó C là hang so.
Bài toán 2.3. Tìm nguyên hàm của hàm so f (x) =
3
x2
+ 3x + 8
.
(x + 1) (x − 1) (x − 2) (x + 3)
Lài giai. Đ t P (x) = x2
+ 3x + 8 và xét bon nút n®i suy là x1 = −1; x2 =
1; x3 = 2; x4 = −3. Ta có a1 = 6, a2 = 12, a3 = 18, a4 = 8. Áp dụng công thác
n®i suy Lagrange, ta có
1 3 6 1
f (x) =
2 (x + 1)
−
2 (x − 1)
+
5 (x − 2)
−
5 (x + 3)
.
Tà đây, ta suy ra nguyên hàm của hàm so can tìm là
∫
f (x) dx =
1
ln |x + 1| −
3
ln |x − 1| +
6
ln |x − 2| −
1
ln |x + 3| + C,
2 2 5 5
trong đó C là hang so.
Bước 2. Thực hi n phép chia P(x) cho
định dạng đơn giản.
(x − xi) ta sě được tích phân bat
24
Viết đề tài giá sinh viên – ZALO:0973.287.149-TEAMLUANVAN.COM
3
−
Bài toán 2.4. Tìm nguyên hàm của hàm so
x2
f(x) = .
(x − 1) (x − 2) (x − 3)
Lài giai. Đ t P(x) = x2
và xét ba nút n®i suy là x1 = 1; x2 = 2; x3 = 3. Ta có
a1 = 1, a2 = 4, a3 = 9. Áp dụng công thác n®i suy Lagrange ta được
9
P (x) = − (x − 2) (x − 3) − 4 (x − 1) (x − 2) +
Rút gon f(x) ta được hàm so sau.
f(x) =
−1
−
4
+
9
.
2
(x − 1) (x − 2) .
2(x − 1) (x − 2) 2(x − 3)
Tà đây, ta suy ra nguyên hàm của hàm so can tìm là
∫
f (x) dx = − ln |x − 1| − 4 ln |x − 2| +
9
ln |x − 3| + C,
trong đó C là m®t hang so.
Bài toán 2.5. Tìm nguyên hàm của hàm so
x2
5x + 10
f(x) = .
(x − 1) (x − 2) (x + 1) (x + 2)
Lài giai. Đ t P(x) = x2
−5x+10 và xét bon nút n®i suy là x1 = 1; x2 = 2; x3 =
−1; x4 = −2. Ta có a1 = 6, a2 = 4, a3 = 16, a4 = 24.
Áp dụng công thác n®i suy Lagrange ta được
P (x) = − (x − 2) (x + 1) (x + 2)
1
+
3
(x − 1) (x + 1) (x + 2)
8
−
3
(x − 1) (x − 2) (x + 2)
— 2 (x − 1) (x − 2) (x − 3) .
Rút gon f(x) ta được hàm so sau.
f (x) =
−1
+
1
+
8
−
2
.
(x − 1) 3 (x − 2) 3 (x + 1) (x + 2)
Tà đây, ta suy ra nguyên hàm của hàm so can tìm là
∫
f (x) dx = − ln |x − 1| +
1
ln |x − 2| +
8
ln |x + 1| − 2 ln |x − 4| + C,
3 3
trong đó C là m®t hang so.
25
Viết đề tài giá sinh viên – ZALO:0973.287.149-TEAMLUANVAN.COM
5
−
5 3 4 35 2 4 7 2 3
Bài toán 2.6. Tìm nguyên hàm của hàm so
x2
− 2x + 3
Lài giai. Đ t
f(x) =
24x3 − 10x2 − 3x + 1
.
Q(x) = 24x3
− 10x2
− 3x + 1 = 24(x −
1
)(x +
1
)(x −
1
).
2
Khi đó ta đ t P(x) = x2
và xét ba nút n®i suy là x1 =
3
1
2
; x2
4
=
−1
; x
3
1
= . Ta
4
có a1 =
9
4
, a2 =
34 41
9
, a3 =
16
.
Áp dụng công thác n®i suy Lagrange ta được
P (x) =
54
x +
1
x −
1
+
272
x −
1
x −
1
−
123
x −
1
x +
1
.
Rút gon f(x) ta được hàm so sau.
54 272 123
f(x) = 1
5(x −
2
)
+
1
+
x + 7
3
1
.
x −
4
Tà đây, ta suy ra nguyên hàm của hàm so can tìm là
∫
f (x) dx =
54
ln |x
trong đó C là m®t hang so.
1
—
2
|
272
+
35
ln |x
1
—
3
| −
123
7
ln |x
x3
1
—
4
| + C,
Bài toán 2.7. Tìm nguyên hàm của hàm so f(x) =
x4 − 5x2 + 4
.
Lài giai. Ta phân tích x4
− 5x2
+ 4 = (x − 1)(x − 2)(x + 1)(x + 2). Khi đó ta
đ t P (x) = x3
và xét bon nút n®i suy là x1 = 1; x2 = 2; x3 = −1; x4 = −2. Ta
có a1 = 1, a2 = 8, a3 = −1, a4 = −8.
Áp dụng công thác n®i suy Lagrange ta được
P(x) =
−1
(x 2)(x + 1)(x
6
1
+ 2) +
2
2
3
(x − 1)(x + 1)(x + 2)
—
6
(x − 1)(x − 2)(x + 2) +
3
(x − 1)(x − 2)(x + 1).
Rút gon f(x) ta được hàm so sau.
f(x) =
−1
+
2
−
1
+
2
.
6(x − 1) 3(x − 2) 6(x + 1) 3(x + 2)
3
26
Viết đề tài giá sinh viên – ZALO:0973.287.149-TEAMLUANVAN.COM
Q
Y
Y
1
1
· · ·
h Q(x) i(k)
1
Tà đây, ta suy ra nguyên hàm của hàm so can tìm là
∫
f (x) dx =
−1
ln |x − 1| +
2
ln |x − 2| −
1
ln |x + 1| +
2
ln |x + 2| + C,
6 3 6 3
trong đó C là m®t hang so.
Như v y, chúng ta thay rang đe tính nguyên hàm của hàm phân thác với các
cực điem đơn thì ta sě sả dụng công thác n®i suy Lagrange đe tính. V y với hàm
phân thác có các cực điem b c tùy ý ta sě tính nguyên hàm như the nào. Câu trả
lời sě được trình bày trong phan tiep theo.
2.2 Nguyên hàm của hàm phân thfíc v i các cfic điem
b c tùy j
Ta sả dụng công thác n®i suy Hermite đe tính nguyên hàm với hàm phân thác
với các cực điem b c tùy ý.
Trong mục này, ta sě giới thi u phương pháp tìm nguyên hàm của hàm so hǎu tỉ
có dạng
Ta có
Q(x)
W(x)
dx, với W(x) =
n
j=1
(x − xj)rj
với deg Q < r1 +r2 +· · ·+rn = N.
n
đ t
W(x) = (x − xj)rj
,
j=1
n
W1(x) = (x − xj)rj
j=2
với ∀i, j = 1, 2, . . . , n. Ta de thay,
Q(x) =
α0
+
α1
+ · · · +
αr1−1
+ Q (x),
W (x)
và
(x − x1)r
(x − x1)r1−1 (x − x1)
Q(x)
W1(x)
= α0
α1
+
x − x1
α2
+
(x − x )2
+ +
α{r1 − 1}
(x − x1)r1−1
+ Q1(x)(x − x1 )r1
với k!αk =
W1(x) x=x1
; k = 0, 1, . . . , r1 − 1, hay
1 h Q(x) i(k)
x=x1
(x)
1
W
k!
∫
αk = ; k = 0, 1, . . . , r1 − 1. (2.1)
27
Viết đề tài giá sinh viên – ZALO:0973.287.149-TEAMLUANVAN.COM
Q
n
∫
Σ
−
(x − x1)r
(x − x1)r1−1 (x − x1)
(x − xn)r (x − xn)rn−1 (x − xn)
N®i dung phương pháp:
- Bước 1: Xác định các nút n®i suy là nghi m đơn ho c nghi m b®i của mau
thác.
- Bước 2: Bieu dien đa thác đã cho theo các nút n®i suy theo công thác Hermite.
- Bước 3: Tính các h so bang công cụ đạo hàm và bieu dien.
∫
Q(x)
dx =
n
j=1
(x − xj)rj
=
∫
α0
+
α1
+ · · · +
αr1−1
dx + · · · +
+
∫
β0
+
β1
+ · · · +
βrn−1
dx.
Sau đó, nguyên hàm này có the tính được nhờ bảng nguyên hàm của các hàm so
thường g p.
Tiep theo, ta xét m®t vài ví dụ minh hoa.
Bài toán 2.8. Tìm nguyên hàm của hàm so f (x) =
Lài giai. Áp dụng công thác n®i suy Hermite, ta có
x2
+ x + 1
(x − 1)3
(x + 2)2
.
f (x) =
−1
+
1 1 2
+ + .
9(x + 2)2
3(x − 1)3
(x − 1)2 3 (x − 1)
V y nguyên hàm của hàm so can tìm là
∫
f (x) dx =
1
−
1
−
1
+
2
ln |x − 1| + C,
9 (x + 2)
trong đó C là hang so.
Bài toán 2.9. Tính tích phân
6(x − 1)2 x − 1 3
I =
Pn(x)dx
,
(x − a)n+1
trong đó Pn(x) là đa thác b c n của x.
Lài giai. Áp dụng khai trien Taylor cho đa thác Pn(x) tại x = a :
n (k)
P (x) =
Pn (a)
(x a)k
.
n
k!
k=0
1
28
Viết đề tài giá sinh viên – ZALO:0973.287.149-TEAMLUANVAN.COM
Σ
−
k!(n − k)(x − a)n−k n!
k!(2017 − k)(x − 1)2017−k 2017!
k=0
k=0
Khi đó
I =
∫
Pn(x) dx =
Σ
Pn
(k)
(a)
∫
dx
n−1
(x − a)n+1
(k)
k=0
k!
(n)
(x − a)n−k+1
=
Σ Pn (a)
+
Pn (a)
ln |x − a| + C.
Bài toán 2.10. Tính tích phân
I =
x2017dx
(x − 1)2018
.
Lài giai. Áp dụng khai trien Taylor cho đa thác P(x) = x2017
tại x = 1 :
2017 (k)
P (x) =
Pn (1)
(x 1)k
.
k!
Khi đó
k=0
∫
P(x) Σ
2017
P(k)
(1)
∫
dx
I =
2016
(x − 1)2018
dx =
(k)
k=0
k!
(2017)
(x − 1)2018−k
=
Σ P (1)
+
P (1)
ln |x − 1| + C,
với C là hang so tuỳ ý.
Bài toán 2.11. Tìm nguyên hàm của hàm so
1
Lài giai.
f(x) =
(x2 − 5x + 6)4
.
Bien đői (x2
− 5x + 6)4
= (x − 3)4
(x − 2)4
.
Theo thu t toán Hermite áp dụng với mau so là các nghi m b®i ta có:
1
(x − 3)4(x − 2)4
=
α0
(x − 3)4
+
α1
(x − 3)3
+
α2
(x − 3)2
+
α3
(x − 3)
β0 β1 β2 β3
+ + + + .
Ta có
(x − 2)4 (x − 2)3 (x − 2)2 (x − 2)
1
(x − 2)4
= α0 + α1 (x − 3) + α2 (x − 3)2
+ α3 (x − 3)3
n
∫
29
Viết đề tài giá sinh viên – ZALO:0973.287.149-TEAMLUANVAN.COM
− − − −
+ +
β0
+
β1
+
β2
+
β3
(x − 3)4
.
+
α0
+
α1
+
α2
+
α3
(x − 2)4
,
1
2
3
0 1
1
2
3
Trong đó
(x − 2)4 (x − 2)3 (x − 2)2
1
(x − 2)
α0 =
(3 − 2)4
= 1;
α =
1 h
(x − 2)−4
i(1)
= −4;
1! x=3
α =
1 h
(x − 2)−4
i(2)
= 10;
2! x=3
α =
1 h
(x − 2)−4
i(3)
= −20.
Ta lại có
1
= β + β
3!
(x − 2) + β
x=3
(x − 2)2
+ β (x − 2)3
+
(x − 1)3
với
(x − 3)4 (x − 3)3 (x − 3)2
1
(x − 3)
β0 =
(2 − 1)3
= 1;
β =
1 h
(x − 1)−4
i(1)
= 4;
1! x=2
β =
1 h
(x − 1)−4
i(2)
= 10;
2! x=2
β =
1 h
(x − 1)−4
i(3)
= 20.
3! x=2
V y ta có the viet
1
(x − 3)4(x − 2)4
1
=
(x 3)4
1
+
−4
(x 3)3
4
10
+
(x 3)2
10
+
−20
(x 3)
20
Tà đó, suy ra
+
(x − 2)4
+
(x − 2)3
+
(x − 2)2
+
(x − 2)
.
∫
1
dx =
∫
1
dx
(x2 − 5x + 6)4 (x − 3)4(x − 2)4
=
∫
(
1
+
−4
+
10
+
−20
)dx
(x − 3)4 (x − 3)3 (x − 3)2 (x − 3)
2 3
30
Viết đề tài giá sinh viên – ZALO:0973.287.149-TEAMLUANVAN.COM
1
2
0 1
+
∫
(
1
+
4
+
10
+
20
)dx
(x − 2)4 (x − 2)3 (x − 2)2 (x − 2)
=
−1
+
2
+
−10
− 20 ln |x − 3|
3(x − 3)3 (x − 3)2 (x − 3)
+
−1
+
−2
+
−10
+ 20 ln |x − 2| + C,
3(x − 2)3 (x − 2)2 (x − 2)
=
−1
+
2
+
−10
+ 20 ln
|x − 2|
3(x − 3)3 (x − 3)2 (x − 3) |x − 3|
+
−1
+
−2
+
−10
+ C,
3(x − 2)3 (x − 2)2 (x − 2)
trong đó C là hang so.
Bài toán 2.12. Tìm nguyên hàm của hàm so
1
f(x) =
(x − 1)3(x − 2)4
.
Lài giai. Theo thu t toán Hermite cho mau so có các nghi m b®i ta có:
1
(x − 1)3(x − 2)4
=
α0
(x − 1)3
+
α1
(x − 1)2
+
α2
+
(x − 1)
β0 β1 β2 β3
+ + + + .
(x − 2)4 (x − 2)3 (x − 2)2 (x − 2)
và
1
(x − 2)4 = α0 + α1 (x − 1) + α2 (x − 1)2
+
+
β0
+
β1
+
β2
+
β3
(x − 1)3
.
Trong đó
(x − 2)4 (x − 2)3 (x − 2)2 (x − 2)
1
α0 =
(1 − 2)4
= 1;
α =
1 h
(x − 2)−4
i(1)
= 4;
1! x=1
α =
1 h
(x − 2)−4
i(2)
= 10.
Ta lại có
1
= β + β
2!
(x − 2) + β
x=1
(x − 2)2
+ β (x − 2)3
(x − 3)4
2 3
31
Viết đề tài giá sinh viên – ZALO:0973.287.149-TEAMLUANVAN.COM
−
(x − 1)3(x − 2)4 (x − 1)3 (x − 1)2 (x − 1)
(x − 2)4 (x − 2)3 (x − 2)2 (x − 2)
1
2
3
+
α0
+
α1
+
α2
(x − 2)4
,
với
(x − 1)3 (x − 1)2 (x − 1)
1
β0 =
(2 − 1)3
= 1;
β =
1 h
(x − 1)−3
i(1)
= −3;
1! x=2
β =
1 h
(x − 1)−3
i(2)
= 6;
2! x=2
β =
1 h
(x − 1)−3
i(3)
= −10.
V y ta có the viet
1
3! x=2
1 4 10
(x − 1)3(x − 2)4
=
(x − 1)3
+
(x − 1)2
+
(x − 1)
Tà đó suy ra
1
+
(x − 2)4
+
−3
(x − 2)3
6
+
(x − 2)2
+
−10
.
(x − 2)
∫
1
dx =
∫
1
+
4
+
10
+
1
+
−3
+
6
+
−10
dx,
=
−1
+
−4
+ 10 ln |x − 1| +
−1
2(x − 1)2 (x − 1) 3(x − 2)3
+
3
+
−6
− 10 ln|x − 2| + C,
2(x − 2)2 (x − 2)
=
−1
+
−4
+ 10 ln
|x − 1|
2(x − 1)2 (x − 1) |x − 2|
+
−1
+
3
+
−6 + C,
3(x − 2)3 2(x − 2)2
trong đó C là hang so.
(x − 2)
Bài toán 2.13. Tìm nguyên hàm của hàm so
r
f(x) = =
q
2x2
2x + 5
(x − 1)3(x − 2)4
.
Lài giai. Theo thu t toán Hermite, ta có
2x2
− 2x + 5 α0 α1 α2
(x − 1)3(x − 2)4
=
(x − 1)3
+
(x − 1)2
+
(x − 1)
+
32
Viết đề tài giá sinh viên – ZALO:0973.287.149-TEAMLUANVAN.COM
−
x=1
x=1
−
x=2
x=2
x=2
+
β0
+
β1
+
β2
+
β3
(x − 1)3
,
+
α0
+
α1
+
α2
(x − 2)4
,
0
1
2
β0 β1 β2 β3
+ + + + ,
(x − 2)4 (x − 2)3 (x − 2)2 (x − 2)
và
2x2
2x + 5 2
(x − 2)4
= α0 + α1(x − 1) + α2(x − 1) +
trong đó
(x − 2)4 (x − 2)3 (x − 2)2 (x − 2)
α =
2.1 − 2.1 + 5
(1 − 2)4 = 5;
M t khác
α =
h
(2x2
− 2x + 5)(x − 2)−4
i(1)
α =
h
(2x2
− 2x + 5)(x − 2)−4
i(2)
= 22;
= 60.
2x2
2x + 5 2
(x − 1)3
= β0 + β1(x − 2) + β2(x − 2) + β3(x − 2)
trong đó
(x − 1)3 (x − 1)2
2.22
− 2.2 + 5
(x − 1)
β0 =
(2 − 1)3
= 9;
β1
β2
β3
V y ta có the viet
=
h
(2x2
− 2x + 5)(x − 1)−3
i(1)
=
h
(2x2
− 2x + 5)(x − 1)−3
i(2)
=
h
(2x2
− 2x + 5)(x − 1)−3
i(3)
= 9;
= 68;
= −360.
2x2
− 2x + 5 5 22 60
(x − 1)3(x − 2)4
=
(x − 1)3
+
(x − 1)2
+
(x − 1)
9
+
(x − 2)4
+
−21
(x − 2)3
68
+
(x − 2)2
+
−360
.
(x − 2)
3
33
Viết đề tài giá sinh viên – ZALO:0973.287.149-TEAMLUANVAN.COM
— −
−
(x − 1)3(x − 2)4
dx =
(x − 1)3
+
(x − 1)2
+
(x − 1)
(x − 2)4 (x − 2)3 (x − 2)2 (x − 2)
Tà đó, suy ra
∫
2x2
− 2x + 5
∫
5 22 60
+
9
+
−21
+
68
+
−360
dx
=
−5
+
−22
+ 60 ln |x − 1|
2(x − 1)2 (x − 1)
+
−3
+
21
+
−68
−360 ln |x − 2| + C,
(x − 2)3 2(x − 2)2 (x − 2)
trong đó C là hang so.
Bài toán 2.14 (Olympic sinh viên toàn quoc - 2001). Cháng minh rang ton tại
so thực x ∈ (0, 1) sao cho
1
t2000dt
(1 + t)(1 + t2) · · · (1 + t2001)
=
x2000
(1 + x)(1 + x2) · · · (1 + x2001)
.
Lài giai. Đ t
f(t) =
t2000
(1 + t)(1 + t2) · · · (1 + t2001)
, t ∈ [0, 1].
Bài toán 2.15. Tìm nguyên hàm của hàm so
r
f(x) = =
q
3x3
2x2
4x + 23
(x + 3)(x − 1)3
.
Lài giai. Ta có q = (x 1)3
(x + 3) .
r
Theo thu¾t toán Hermite, ta viet
q
dưới dạng
r a b c b
q
=
x + 3
+
x − 1
+
(x − 1)2
+
(x − 1)3
,
trong đó a, b, c, d là các hang so.
Đong nhat các h so ta có h phương trình sau
a + b = 3
−3a + b + c = −2
3a − 5b − 2c + d = −4
−a + 3b − 3c + 3d = 23.
∫
x
34
Viết đề tài giá sinh viên – ZALO:0973.287.149-TEAMLUANVAN.COM
— −
−
∫ ∫
Giải h ta có a = 1, b = 2, c = −1 và d = 5. Do đó
Tà đó suy ra
r 1
=
q x + 3
2
+
x − 1
+
−1
(x − 1)2
5
+
(x − 1)3
.
∫
r
dx =
∫
1
dx +
∫
2
dx +
∫
−1
dx +
∫
5
dx.
q x + 3 x − 1 (x − 1)2 (x − 1)3
V y, nguyên hàm của hàm so can tìm là
∫
r
dx = ln |x + 3| + 2ln|x − 1| +
1
+
−5
+ C,
q
trong đó C là m®t hang so.
Bài toán 2.16. Tìm nguyên hàm của hàm so
x − 1 2(x + 1)2
r
f(x) = =
q
7x3
34x2
+ 96x 54
(x − 2)3(3x + 1)2
.
Lài giai. Ta có q = (x 2)3
(3x + 1)2
.
r
Theo thu¾t toán Hermite, ta viet
q
dưới dạng
r a b c d e
q
=
3x + 1
+
(2x + 1)2
+
x − 2
+
(x − 2)2
+
(x − 2)3
,
trong đó a, b, c, d, e là các hang so. Đong nhat thác, ta thu được h phương trình
9a + 3d = 0
−30a + 9b − 17d = 7
13a − 12b + 9c + 30d + e = −34
20a + 11b + 6c − 12d − 14e = 96
4a − 2b + c − 8d + 4e = −54.
Giải h phương trình này ta được a = 0, b = 0, c = 2; d = 0 và e = 7. Do đó
r 2 7
q
=
(x − 2)3
+
(3x + 1)2
.
Ta có
2
dx =
−1
+ C và
7
dx =
−7
+ C , với
(x − 2)3
C1, C2 là các hang so.
Tà đó suy ra
(x − 2)2 (3x + 1)2 3(3x + 1) 2
∫
−1 7 −7x2
+ 5x − 4
f(x)dx =
với C là m®t hang so.
(x − 2)2
+
2(3x + 1)
=
3(x − 1)2(3x + 1)
+ C,
1
35
Viết đề tài giá sinh viên – ZALO:0973.287.149-TEAMLUANVAN.COM
−
∫
Bài toán 2.17. Tìm nguyên hàm của hàm so
x5
− x4
+ 3x3
− 3x2
+ x + 3
f(x) =
x4 − 2x3 + 3x2 − 2x + 1
.
Lài giai. Ta có
f(x) =
x5
− x4
+ 3x3
− 3x2
+ x + 3
x4 − 2x3 + 3x2 − 2x + 1
2x3
− x2
+ 4x + 2
Đ t
= x + 1 +
r 2x3
x2
+ 4x + 2
q
=
(x2 − x + 1)2
.
(x2 − x + 1)2
.
r
Theo thu¾t toán Hermite, ta viet
q
dưới dạng
r ax + b cx + d
q
=
x2 − x + 1
+
(x2 − x + 1)2
,
trong đó a, b, c, d là các hang so. Đong nhat hai ve, ta được h phương trình
a = 2
−a + b = −1
a + b + c = 4
b + d = 2.
Giải h phương trình này ta được a = 2, b = 1, c = 3 và d = 1. Tà đó suy ra
r 3x + 1 2x + 1
q
=
(x2 − x + 1)2
+
x2 − x + 1
.
2x + 1
Bây giờ, ta sě tìm
x2 − x + 1
dx. Ta có
∫
2x + 1
dx =
∫
2x − 1
dx + 2
∫
1
dx.
x2 − x + 1 x2 − x + 2 (x −
1
)2 +
3
Tà đó suy ra
∫
2x + 1 2
2 4
4 2x − 1
x2 − x + 1
dx = ln(x
với C1 là hang so.
— x + 1) + √
3
arctan √
3
+ C1,
36
Viết đề tài giá sinh viên – ZALO:0973.287.149-TEAMLUANVAN.COM
∫
∫
∫
3 3
−
−
−
∫
2
∫
2
3x + 1
Tiep sau đây chúng ta sě tìm
(x2 x + 1)2
dx. Theo thu¾t toán Hermite ta
có
3x + 1 = t(x2
− x + 1) + (ux + v)(2x − 1),
với t, u, v là các hang so. Đong nhat hai ve, ta được
Tà đó suy ra
2
t = ; u =
3
5
; v =
−1
.
3 3
3x + 1
dx =
(x2 − x + 1)2
2 5 1
= 3 dx +
x − x + 1
( x − )(2x − 1)
(x2 − x + 1)2
dx.
Ta de dàng tìm được nguyên hàm
2
3 dx =
x − x + 2
4
3
√
3
arctan
2x 1
√
3
+ C2,
với C2 hang so. Áp dụng phương pháp tích phân tàng phan
5 1 2x − 1
(đ t u = x − và dv = dx) ta được
3 3 (x2 − x + 1)2
5 1 5 1 5
∫ (
3
x −
3
)(2x − 1) (
3
x −
3
)
∫
3
(x2 − x + 1)2
dx = −
x2 − x + 1
+ x2 − x + 1
dx
5 1
(
3
x −
3
) 10 2x − 1
= −
x2 − x + 1
+
3
√
3
arctan
với C3 là m®t hang so.
Như v y, ta có
√
3
+ C3,
∫
f(x)dx = ln(x2
4
— x + 1) + √
3
arctan
2x 1
√
3
+
5 1
4 2x − 1 (
3
x −
3
) 10 2x − 1
+
3
√
3
arctan √
3
−
x2 − x + 1
+
3
√
3
arctan √
3
+ C
2 5x − 1 26 2x − 1
= ln(x − x + 1) −
3(x2 − x + 1)
+
3
√
3
arctan √
3
+ C,
với C là m®t hang so.
Tiep theo, ta xét m®t so ví dụ ve tính toán các nguyên hàm của m®t so lớp
hàm tőng quát.
37
Viết đề tài giá sinh viên – ZALO:0973.287.149-TEAMLUANVAN.COM
∫
−
∫
=
−
∫
∫
h
∫
Bài toán 2.18. Tính tích phân
I =
dx
.
n
(ax2 + bx + c)n
Lài giai. Đưa tam thác b c hai ve dạng chính tac
ax2 + bx + c = a x +
b 2
+
2a
4ac − b2
4a2
= a[t2
+ ∆]
b
trong đó t = x + ; ∆ =
2a
Khi đó
4ac b2
4a2
.
dx
In
(ax2 + bx + c)n
1
=
an
dt
.
(t2 + ∆)n
• Neu ∆ =
4ac b2
4a2
= 0 thì
In
4ac − b2
1
=
an
dt
t2n
1
=
an(1 − 2n)t2n−1)
+ C.
• Neu ∆ =
4a2
/= 0 thì ta có
M t khác
1
In =
an
dt
(t2 + ∆)n
1
=
an
Jn.
J =
dt
n
(t2 + ∆)n
1 t
= +
2n − 3
J
2(n − 1)∆ (t2 + ∆)n − 1
mà
I =
1 dt
2(n − 1)∆
1
= J
n−1
.
n−1 an−1
(t2 + ∆)n − 1 an−1 n−1
Suy ra Jn−1 = an−1
In−1 nên
J =
dt
n
(t2 + ∆)n
1
=
2(n − 1)∆
và
t
.
(t2 + ∆)n−1
+
2n − 3
2(n − 1)∆
an−1
In−1
In =
1
an
Jn
i
∫
∫
∫
38
Viết đề tài giá sinh viên – ZALO:0973.287.149-TEAMLUANVAN.COM
∫
∫
∫
∫
∫
| |
a sin x + b cos x + c a sin x + b cos x + c
1
=
2an(n − 1)∆
t
.
(t2 + ∆)n−1
+
2n − 3
2a(n − 1)∆
an−1
In−1.
Đây là công thác truy hoi cho phép tính các tích phân I2, I3, . . . sau khi biet
dx
I1 =
ax2 + bx + c
.
Bài toán 2.19. Tính tích phân
I =
a1 sin x + b1 cos x
dx.
a sin x + b cos x
Lài giai.
Ta có
a1 sin x + b1 cos x = A(a sin x + b cos x) + B(a sin x + b cos x)′.
Suy ra
I =
A(a sin x + b cos x) + B(a sin x + b cos x)′
dx
a sin x + b cos x
=Ax + B ln |a sin x + b cos x| + C.
Bài toán 2.20. Tính tích phân
I =
a1 sin x + b1 cos x + c1
dx.
a sin x + b cos x + c
Lài giai.
Ta có
a1 sin x + b1 cos x + c1 = A(a sin x + b cos x + c)+
+ B(a sin x + b cos x + c)′ + C.
Khi đó
I =A
∫
dx + B
∫
d(a sin x + b cos x + c)
+ C
∫
dx
=Ax + B ln a sin x + b cos x + c + C
dx
.
a sin x + b cos x + c
Với tích phân
dx
,
a sin x + b cos x + c
∫
39
Viết đề tài giá sinh viên – ZALO:0973.287.149-TEAMLUANVAN.COM
−
∫
∫
∫
∫ ∫
∫
n
x
đ t tan
2
Do đó
= t ta được
dx =
2dt
1 + t2
; sin x =
2t
1 + t2
; cos x =
1 t2
1 + t2
.
dx
a sin x + b cos x + c
Bài toán 2.21. Tính tích phân
=
2dt
.
(c − b)t2 + 2at + b + c
Lài giai. Đ t In
I =
dx
dx.
(a + b cos x)n
dx
=
(a + b cos x)n
. Ta có
In−2 =
dx
(a + bcosx)n−2
=
(a + b sin x)dx
(a + b cos x)n−1
= aIn−1 + b
d(sin x)
.
(a + b cos x)n−1
Ta có
sin x
In−2 = aIn−1 + b
(a + b cos x)n−1
− (n − 1)
b2
sin2
xdx
(a + b cos x)n
.
Khi đó
b2
sin2
x = −(a2
− b2
) + 2a(a + b cos x) − (a + b cos x)2
.
I =
−b sin x
+
(n − 1)(a2 − b2)(a + b cos x)n−1
+
(2n − 3)a
I
(n − 1)(a2 − b2)
Asin x
n−1
(n − 2)
I
(n − 1)(a2 − b2) n−2,
với
In = + BIn−1 + CIn−2,
(a + bcos x)n−1
A =
−b
; B =
(2n − 3)a
;
(n − 1)(a2 − b2) (n − 1)(a2 − b2)
C =
n − 2
.
(n − 1)(a2 − b2)
∫
∫
−
40
Viết đề tài giá sinh viên – ZALO:0973.287.149-TEAMLUANVAN.COM
∫
∫
x
x
x2
e
Bài toán 2.22. Tính tích phân
I =
∫
Lài giai. Ta có
Pn(x)dx
√
ax2 + bx + c
.
2Pn(x) = 2Rn−2(x)(ax2
+ bx + c) + Qn−1(x)(2ax + b) + 2α.
Đong nhat h so hai ve ta được Rn−2(x), Qn−1(x), α
hay
Pn(x)
√ = Q
′
(x)
√
ax2 + bx + c+
Qn
√
−1(x)(2ax + b)
+√
α
.
ax2 + bx + c
Suy ra
n−1
2 2a2 + bx + c 2a2 + bx + c
I =
∫
= Q
Pn(x)dx
√
ax2 + bx + c
(x)
√
ax2 + bx + c + α √
dx
.
n−1
Bài toán 2.23. Tính tích phân
ax2 + bx + c
P (x)dx
I =
Q(x)
√
ax2 + bx + c
.
Nh n xét 2.4. Đe tính tích phân này ta thực hi n như sau:
P (x)
- Phân tích phân thác
Q(x)
thành các phân thác toi giản.
- Tách thành các tích phân tương áng và chon các phương pháp thích hợp đe tính
các tích phân tàng phan.
Bài toán 2.24. Tìm đieu ki n đoi với đa thác P(x) đe tích phân
I =
∫
P
1
ex
dx
là m®t hàm sơ cap và xét các bài toán tương tự . . .
Lài giai.
Ta có
∫
P
1
dx =
∫
a0 + a1 + · · · + an
1 n x
= a0ex
+ a1
1
ex
dx + a
x
2
∫
1
ex
dx + · · · + a
1
ex
dx.
xn
x
∫ ∫
dx
n
41
Viết đề tài giá sinh viên – ZALO:0973.287.149-TEAMLUANVAN.COM
∫ x ∫ x
xn
−
3 3
2x2 2x 2
( )
x
0 2
2! x
3
3! x2
1
1! 2! 3! (n − 1)!
1
Kí hi u In
Đ t
1
= e dx, ta tính I
xn n−1
1
= e dx.
xn−1
Suy ra
u =
1
xn−1
dv = ex
dx
du = −
n − 1
dx
Khi đó
v = ex
.
I =
1
ex
+ (n − 1)
∫
1
ex
dx =
1
ex
+ (n − 1)I
hay
n−1
xn−1 xn xn−1 n
I = −
1
ex
+
1
I .
Tà đó suy ra
(n − 1)xn−1 n − 1
n−1
a2I2
=
a2
ex
+ a I
x 2 1
a I = −
a3
ex
−
a3
ex
+
a3
I
a I = −
a4
ex
−
a4
ex
−
a4
ex
+
a4
I
4 4
3x3
. . . . . . . . .
6x2 6x 6 1
Suy ra
∫
P
1
ex
dx = ex
h
a − a +
a3
+ . . .
1
−
— a +
a4
+ . . .
i 1
− . . .
i
+
+ a +
a2
+
a3
+
a4
+ · · · +
an
I .
Đe
∫
P (
1
)ex
dx là m®t hàm sơ cap thì can
x
a2 a3 a4 an
a + + + + · · · + = 0.
1
1! 2! 3! (n − 1)!
n
1
42
Viết đề tài giá sinh viên – ZALO:0973.287.149-TEAMLUANVAN.COM
(A − B)b = b
1
.
∫
∫
−
a e +
be + c
xm
=
(ln x) + C
; m, n ∈ N, n ≥ 2.
∫
a1ex
+ b1e−x
+ c1
dx + B
Bài toán 2.25. Tính tích phân
In =
lnn
xdx
xm
.
Lài giai. Bang cách l p lu n tương tự bài toán trên ta có the tính được
∫
lnn
xdx 1
Bài toán 2.26. Tính tích phân
Lài giai. Phân tích
I =
aex + be−x + c
dx.
aex
+ be−x
+ c = A(a1ex
+ b1e−x
+ c1) + B(a1ex
− b1e−x
) + C,
A, B, C được xác định tà h
Khi đó
∫
a1ex
+ b1e−x
+ c1
(A + B)a1 = a
Ac1 + C = c.
I =
aex + be−x + c
dx
∫ ∫
d(a1ex
+ b1e−x
+ c1)
∫
dx
= Ax + B ln .a1 ex
+ b1e−x
+ c1 + C
dx
.
a1ex + b1e−x + c1
dx
Đoi với tích phân x x
1 1 1
là tam thác b c hai .
ta đ t t = ex
chuyen ve tích phân có mau
1
e−x + c
1
ex + b
1
a
1
1
1
n
xm−1P
∫
In =
= A
a ex + b e−x + c
+ hspace ∗ 6cm + C
Viết đề tài giá sinh viên – ZALO:0973.287.149-TEAMLUANVAN.COM
n
x
43
Chương 3. M t so dạng toán liên quan
Công thác n®i suy Lagrange và Hermite có nhieu áng dụng thú vị. N®i dung
của chương này sě trình bày m®t so áng dụng của công thác n®i suy Lagrange và
Hermite.
3.1 M t so bài toán ve đa thfíc nh n giá trị nguyên
Ta xét m®t so bài toán ve đa thác nh n giá trị nguyên ho c với h so nguyên.
Bài toán 3.1. Đa thác
x(x − 1) · · · (x − n + 1)
P(x) =
n!
được ký hi u bởi P (x) = x
là đa thác nh n giá trị nguyên.
Lài giai. Nh n xét rang tích của n so tự nhiên liên tiep thì chia het cho n!. Giả
sả x(x − 1) · · · (x − n + 1) là tích của n so tự nhiên liên tiep (x ≥ n). Khi đó ta
có
là m®t so nguyên.
x(x − 1) · · · (x − n + 1)
n!
= Cn
Tà đó, ta suy ra tích của n so nguyên liên tiep thì chia het cho n! và do đó
Bài toán 3.1 được giải quyet.
Nh n xét 3.5. Với n ≥ 2 cho ta ket quả sau.
Ton tại đa thác nh n giá trị nguyên với các h so có the không bat bu®c là
nhǎng so nguyên. Với n ≥ 2, ton tại m®t đa thác với các h so đeu hǎu t không
nguyên nhưng nh n giá trị nguyên tại các điem nguyên.
Bài toán 3.2. Đa thác P(x) b c n nh n giá trị nguyên tại moi điem nguyên khi
và chỉ khi P(x) nh n giá trị nguyên tại (n + 1) điem nguyên liên tiep.
Lài giai.
Đieu ki n can là hien nhiên.
Đieu ki n đủ. Sả dụng công thác khai trien Abel với xi = a+i (i = 1, 2, . . . , n),
ta được
P(x) = b0 + b1(x − a − 1) + b2(x − a − 1)(x − a − 2)+
44
Viết đề tài giá sinh viên – ZALO:0973.287.149-TEAMLUANVAN.COM
1 n
1
1 k
+ bn(x − a − 1) · · · (x − a − n).
Ta có P(a + 1) ∈ Z nên b0 ∈ Z, P(a + 2) ∈ Z nên b0 + b1 ∈ Z và b1 ∈ Z,
P(a + 3) ∈ Z nên b0 + 2b1 + 2!b2 ∈ Z và 2!b2 ∈ Z.
Tương tự, ta thu được
P(a + n) ∈ Z nên (n − 1)!bn ∈ Z, P(a) ∈ Z nên n!bn ∈ Z.
Tà đó ta có k!bk ∈ Z (k = 0, 1, . . . , n), đieu phải cháng minh (vì tích của n so
nguyên liên tiep thì chia het cho k!).
Nh n xét 3.6. Thực ra, ta chỉ can đieu ki n P (x) là đa thác nh n giá trị nguyên
khi và chỉ khi P (x) nh n các giá trị nguyên khi x = 0, 1, . . . , n ho c P (x) nh n
các giá trị nguyên tại n + 1 giá trị nguyên liên tiep là đủ.
Bài toán 3.3. a) Cháng minh rang moi đa thác b c n đeu có the bieu dien được
dưới dạng
Pn(x) = b0 + b1 x
!
+ · · · + bn−1
x
n − 1
!
+ bn
x
!
. (3.1)
b) Đa thác Pn(x) là đa thác nh n giá trị nguyên khi và chỉ khi tat cả các h
so b0, b1, . . ., bn là các so nguyên.
Lài giai. a) Cháng minh bang quy nạp. Giả sả đa thác P(x) b c n có dạng
P(x) = anxn
+ an−1xn−1
+ · · · + a1x + a0.
Khi n = 1 thì
P1(x) = a1 x + a0 = a0 + a1
x
!
.
Chon b0 = a0, b1 = a1 ta nh n được (3.1) đúng.
Giả sả (3.1) đúng với moi n ≤ k. Khi đó ta có
Pk+1(x) = ak+1xk+1
+ akxk
+ · · · + a1x + a0
= ak+1
(k + 1)!
x
k + 1
!
+ Rk (x), deg Rk (x) ≤ k.
Theo giả thiet quy nạp thì
Rk(x) = b0 + b1
x
!
+ · · · + bk−1
x
k − 1
!
+ bk
x
!
.
45
Viết đề tài giá sinh viên – ZALO:0973.287.149-TEAMLUANVAN.COM
1 k
1
1 2
1 n
1
1
!
Do đó
Pk+1 (x) = b0 + b1
x
!
+ · · · + b
x
!
+ bk+1
x
k + 1
,
với bk+1 = ak+1(k + 1)!.
Do đó (3.1) đúng với k + 1.
b) Đieu ki n can.
Giả sả Pn(x) là đa thác nh n giá trị nguyên với x ∈ Z. Khi đó
P(0) = b0,
P(1) = b0
P(2) = b0
+ b1
+ b1
1
!
,
2
!
+ b
2
!
,
. . . . . . . . .
P(n) = b0 + b1
n
!
+ · · · + b
n
!
.
Vì P(0), P(1), . . ., P(n) ∈ Z và các h so nhị thác Newton đeu nguyên nên
b0, b1, . . . , bn ∈ Z.
Đieu ki n đủ.
Giả sả b0, b1, . . . , bn là các so nguyên. Khi đó P (0), P (1), . . . , P (n) cũng là các
so nguyên nên theo Bài toán 3.2 thì Pn(x) là đa thác nh n giá trị nguyên.
Bài toán 3.4. Neu đa thác P(x) b c n là đa thác nh n giá trị nguyên thì đa
thác Q(x) = n!P(x) ∈ Z[x].
Lài giai. Theo Bài toán 3.3 thì P(x) là đa thác b c n nh n giá trị nguyên nên
ta có bieu dien
Pn(x) = b0 + b1
x
!
+ · · · + bn−1
x
n − 1
!
+ bn x
!
,
Do đó
bj ∈ Z (j = 0, . . . , n).
n!P(x) = n!b0 + n!b1
x
!
+ · · · + n!bn−1
x
n − 1
!
+ n!bn x
!
.
n
n
k
2
n
46
Viết đề tài giá sinh viên – ZALO:0973.287.149-TEAMLUANVAN.COM
k
3 2 1
k
Ta có
b k!
x
!
= b x(x − 1)...(x − k + 1) ∈ Z.
Vì v y n!P(x) ∈ Z[x].
Bài toán 3.5. Cho đa thác
P(x) = anxn
+ an−1xn−1
+ · · · + a0.
Giả sả ton tại h so ak ∈ {a0, a1, . . . , an} sao cho n!ak /∈ Z thì P (x) không phải
là m®t đa thác nh n giá trị nguyên.
Lài giai. Giả sả P (x) là đa thác nh n giá trị nguyên. Theo Bài toán 3.4 thì
n!P (x) ∈ Z[x]. Đieu này dan đen mâu thuan với giả thiet. V y P (x) không là đa
thác nh n giá trị nguyên.
Bài toán 3.6. Cháng minh rang đa thác
f(x) =
1
x3
−
5
x2
+
19
x + 3
3 2 6
là m®t đa thác nh n giá trị nguyên.
Lài giai. Ta viet f(x) dưới dạng
f(x) = 2
x
!
− 3
x
!
+
x
!
+ 3.
Do đó theo Bài toán 3.3 thì f(x) là đa thác nh n giá trị nguyên.
Bài toán 3.7. Cho đa thác
x2002 2001
P(x) =
2003
+ a2001x + · · · + a1x + a0,
ai ∈ Z (i = 0, 1, . . . , 2001).
Cháng tỏ rang ton tại x0 ∈ Z sao cho P(x0) /∈ Z với (tác P(x) không phải là
m®t đa thác nh n giá trị nguyên).
Lài giai. De dàng nh n thay ngay rang
1
(2002!)
2003
/∈ Z.
V y theo Bài toán 3.5, thì P(x) không phải là m®t đa thác nh n giá trị nguyên.
k
47
Viết đề tài giá sinh viên – ZALO:0973.287.149-TEAMLUANVAN.COM
Bài toán 3.8. Xác định các so dương A, B, C sao cho đa thác
f(x) = Ax5
+ Bx3
+ Cx
là đa thác nh n giá trị nguyên với f(3) nh n giá trị nhỏ nhat.
Lài giai. Do A, B, C dương và f (x) ∈ Z khi x ∈ Z nên f (1), f (2), f (3) là
nhǎng so nguyên dương.
Ta có
f (1) = A + B + C,
f(2) = 32A + 8B + 2C,
f(3) = 243A + 27B + 3C.
Suy ra
f(2) − 2f(1) = 30A + 6B. (3.2)
Sả dụng công thác n®i suy Lagrange, ta có bieu dien
A = [f(3) − 4f(2) + 5f(1)], (3.3)
B = [−f(3) + 8f(2) − 13f(1)], (3.4)
C = [f(3) − 9f(2) + 45f(1)]. (3.5)
Vì f(1) nguyên dương nên f(1) ≥ 1.
Tà (3.2) do A, B dương và f(2), f(1) là nguyên dương nên f(2) − 2f(1) ≥ 1.
Suy ra
Tà (3.6) suy ra
Suy ra
f(2) ≥ 2f(1) + 1 ≥ 3. (3.6)
f(3) = 120A + 4f(2) − 5f(1)
= 120A + 4[f(2) − 2f(1)] + 3f(1).
f(3) > 4[f(2) − 2f(1)] + 3f(1) = 4 + 3 = 7
và vì v y f(3) ≥ 8.
Với f(1) = 1; f(2) = 3; f(3) = 8 thì tà (3.2), (3.3), (3.4) ta thu được
1
A =
120
, B =
1 13
, =
8 15
. (3.7)
Ta cháng tỏ với các giá trị A, B, C ở (3.7) thì f(x) nguyên khi x nguyên.
48
Viết đề tài giá sinh viên – ZALO:0973.287.149-TEAMLUANVAN.COM
120 8 15 5 3
Th t v y, ta có
f(x) =
1
x5
+
1
x3
+
13
x =
x + 2
!
+
x + 1
!
là m®t đa thác nh n giá trị nguyên.
Bài toán 3.9. Cho đa thác
P(x) = anxn
+ an−1xn−1
+ · · · + a0
thỏa mãn đieu ki n P(x) ∈ Q với moi x ∈ Z. Cháng minh rang P(x) ∈ Q[x] (hay
ak ∈ Q với moi k ∈ {0, . . . , n}).
Lài giai. Dùng bieu dien Abel của P(x), ta có
P(x) = b0 + b1x + b2x(x − 1) + · · · + bnx(x − 1) · · · (x − n + 1).
Lan lượt cho x = 0, 1, . . . , n − 1 ta thu được các bi ∈ Q hay P(x) ∈ Q[x].
Bài toán 3.10. Cho
f(x) =
1
ax + b
∈ Q với moi x ∈ Z.
Cháng minh rang ax + b ∈ Q[x] (hay a, b ∈ Q).
1
Lài giai. Do f(x) = ∈ Q với moi x ∈ Z. nên
ax + b
1
ax + b = với moi x ∈ Z.
f(x)
V y theo Bài toán 3.9 thì ax + b ∈ Q[x].
Bài toán 3.11. Cho hàm phân thác
ax + b
f(x) = ∈ Q với moi x ∈ Z.
cx + d
Cháng minh rang f(x) có the bieu dien được dưới dạng
Ax + B
f(x) = với A, B, C, D ∈ Z. (3.8)
Cx + D
Lài giai. Neu ad − bc = 0 thì f(x) = const (hang so) và bieu dien (3.8) là hien
nhiên.
49
Viết đề tài giá sinh viên – ZALO:0973.287.149-TEAMLUANVAN.COM
0
0
0
Xét trường hợp ad − bc /= 0.
Neu c = 0 thì bieu dien (3.8 là hien nhiên.
Neu c /= 0 thì sả dụng phân tích
1
f(x) − f(0) =
ta sě thu được ngay bieu dien (3.8).
αx + β
Bài toán 3.12. Cho f(x) là đa thác với h so thực nh n giá trị hǎu tỉ với moi
so x hǎu t và giá trị vô t với moi so vô t . Cháng minh rang f (x) là đa thác
tuyen tính với h so hǎu t .
Lài giai.
1) Ta cháng minh rang các h so của f(x) là hǎu tỉ. Ta cháng minh bang quy
nạp theo b c n của f(x).
Với n = 0, f(x) là hang so và nó là m®t so hǎu t (chȁng hạn bang f(0))
Giả sả khȁng định đúng với tat cả các đa thác b c nhỏ thua so tự nhiên n
(thỏa đieu ki n đe bài).
Đ t f(x) = a0xn
+ a1xn−1
+ · · · + an−1x + an.
De thay an = f(0) là hǎu t .
Đ t
g(x) = a0xn−1
+ a1 xn−2
+ . . . + an−1 =
f(x) − an
x
thì g(x) sě nh n giá trị hǎu t với bien hǎu tỉ x. Theo giả thiet qui nạp nhǎng so
a0, a1, . . . , an−1 là hǎu tỉ. Như v y các h so của f(x) là các so hǎu tỉ. Với đieu đó
f(x) không là hang so, vì trong trường hợp ngược lại f(x) sě là hǎu t với moi x.
Cho f(x) = a0xn
+ a1xn−1
+ · · · + an, n > 0. không mat tính tőng quát có
the cho rang ai là nguyên, ngoài ra đa thác
g(x) = an−1
(f(x) − an)
= (a0x)n
+ a1(a0x)n−1
+ . . . + an−1an−2
(a0x)
nghĩa là đa thác
h(y) = yn
+ a1yn−1
+ · · · + an−1an−2
y, y = a0x,
thỏa mãn đieu ki n đe bài.
Ta sě cháng minh rang với moi so nguyên đủ lớn m, phương trình h(y) = m
có nghi m. Th t v y, lay m > h(0) và ϕ(y) = h(y) − m. Khi đó ϕ(0) < 0 và
50
Viết đề tài giá sinh viên – ZALO:0973.287.149-TEAMLUANVAN.COM
⇔ − − ⇒
− −
√
3 2
(x 1) 3 + 6 (x
1) = 2 + 3(x 1) 2
— − −
≤ · · ·
−
√
3
lim
y→∞
ϕ(y) = +∞. Vì the phương trình ϕ(y) = 0 hay h(y) = m có nghi m dương
ym. Lay m = p là so nguyên to đủ lớn, ta có h(yp) = p. Tà giả thiet yp là so hǎu
t và vì h so cao nhat của h(y) là 1, thì yp nguyên và ngoài ra yp được chia het
bởi so hạng tự do của ϕ(y) ho c là yp là ước so của p. Nghĩa là yp = 1 ho c là
yp = p. Nhưng đȁng thác yp = 1 chỉ có khả năng nhieu nhat với m®t p, nghĩa là
yp = p cho tat cả so nguyên to đủ lớn p. Nói cách khác, ta đã nh n được h(p) = p
với tat cả so nguyên to đủ lớn. Đieu này chỉ xảy ra với h(y) = y và nghĩa là
f(x) = a0x + a1.
3.2 M t so bài toán xác định đa thfíc
3.2.1 Tìm đa thfíc khi biet các nghi m của nó.
Bài toán 3.13. Tìm tat cả các đa thác P (x) ∈ Z[x] nh n x = 1 +
√
2 +
√
3
3 làm
nghi m. Cháng minh rang deg P (x) ≥ 6.
Lài giai. Ta có
x = 1 +
√
2 +
√
3
3 x 1
√
3
3 =
√
2 x 1
√
2
3
=
√
3
3
⇔ (x − 1) − 2
√
2 − 3 (x − 1)
√
2 (x − 1) −
√
2 = 3
⇔ − − − −
3
2
2
2
⇒ (x − 1) − 3 + 6 (x − 1) = 2 + 3(x − 1) .2
Rút gon ta nh n được đa thác P (x) = x6
6x5
+ 9x4
2x3
+ 9x2
60x + 50.
Giả sả ton tại Q (x) = a5x5
+a4x4
+a3x3
+a2x2
+a1x+a0 nh n x = 1+
√
2+
√
3
3
làm nghi m và deg Q (x) 5. Khí đó ta có Q 1 +
√
2 +
√
3
3 = = k0+k1
√
2+
k2
√
3
3 + k3
√
3
9 + k4
√
2
√
3
3 + k5
√
2
√
3
9 = 0 không the xảy ra.
V y ta có đieu phải cháng minh.
Bài toán 3.14. Xét t p hợp các đa thác P (x) khác hang, thỏa mãn đieu ki n
P x2
− 1 = P (x) .P (−x) , ∀x ∈ R. Hãy tìm trong t p hợp đó 1 đa thác có b c
bé nhat nhưng có nghi m lớn nhat.
Lài giai. Do P (x) khác hang nên ta xét các trường hợp sau:
+) Trường hợp 1: Khi deg P (x) = 1 thì P (x) = ax + b, a /= 0. Đong nhat h
so trong giả thiet ta nh n được a = 1, b =
−1 ± 5
.
2
51
Viết đề tài giá sinh viên – ZALO:0973.287.149-TEAMLUANVAN.COM
0
2
2 2
44
∈
Σ
Khi đó, xét các đa thác P (x) = −x +
−1 +
√
5
∨ P (x) = −x +
−1 −
√
5
.
2
Ta thay các đa thác này có nghi m lan lượt là b = −1 ±
√
5
.
2
+) Trường hợp 2: Neu deg P (x) ≥ 2 và giả sả P (x0) = 0. Khi đó, P x2
− 1 =
0 hay x2
− 1 là nghi m của P (x).
0 √
Neu x >
−1 + 5
thì x2
−1 > x >
−1 +
√
5
. Khi đó P (x) có vô so nghi m,
0
2
vô lý.
0 0
2
√
V y neu deg P (x) ≥ 2 có nghi m thì các nghi m của nó đeu nhỏ hơn
−1 + 5
.
Tà đó suy ra đa thác can tìm là P (x) = −x + −1 +
√
5
.
2
Bài toán 3.15. Tìm tat cả các đa thác b c 4 dạng P (x) = x4
+bx2
+c, (b, c > 0)
sao cho P (x)−x2
= 0 không có nghi m thực nhưng P (P (x))− x4
= 0 có nghi m
thực.
Lài giai. Giả sả ton tại đa thác b c 4 dạng P (x) = x4
+ bx2
+ c, (b, c > 0).
Ta có P (x) − x2
= 0 không có nghi m thực hay x4
+ (b − 1) x2
+ c = 0 không
có nghi m thực nên suy ra Q (x) = x4
+(b − 1) x2
+ c > 0, ∀x. Suy ra P (x) > x2
và P (P (x)) > (P (x)) > x = x hay phương trình P (P (x)) − x = 0
không có nghi m thực, trái với giả thiet của bài ra.
V y không ton tại đa thác nào thỏa mãn yêu cau bài toán.
Bài toán 3.16. Tìm tat cả các đa thác P (x) R [x] có b c n, có n nghi m thực
và thỏa mãn P (x) .P 2x2
= P 2x3
+ x , ∀ ∈ R.
Lài giai. Giả sả P (x) =
n
i=0
aixi
, an /= 0. Ta có P (0) = a0 theo giả thiet
a0.a0 = a0 thì a0 = 1 ∨ a0 = 0.
+) Trường hợp 1: Xét a0 = 0. Khi đó P (x) = xk
Q (x) , k ∈ N, k ≥ 1, Q (x) ∈
R [x] , Q (0) /= 0. Thay vào giả thiet, ta thu được
xk
Q (x) .2k
x2k
Q 2x2
= 2x3
+ x
k
Q 2x3
+ x
nên
Q (x) .2k
x2k
Q 2x2
= 2x2
+ 1
k
Q 2x3
+ x .
Suy ra Q (0) = 0, vô lý.
+) Trường hợp 2: Khi a0 = 1.
2
2
52
Viết đề tài giá sinh viên – ZALO:0973.287.149-TEAMLUANVAN.COM
0
Σ
n n
.Σ i i
n
| | ≤
n n n
0 0
Giả sả x0 là nghi m của P (x) , suy ra 2x3
+ x0 là nghi m của P (x) .
M t khác, ta có
.2x3
+ x0. = |x0| .2x2
+ 1. ≥ |x0| ,
tà đó suy ra P (x) ∈ R [x] có vô so nghi m thực, vô lý.
V y không ton tại đa thác nào thỏa mãn yêu cau bài toán.
Bài toán 3.17. Tìm tat cả các đa thác P (x) ∈ Z [x], là monic b c 2, sao cho
ton tại đa thác Q (x) ∈ Z [x] mà các h so của đa thác R (x) = P (x) Q (x) đeu
thu®c t p {−1; 1}.
Lài giai. Do P (x) ∈ Z [x] là monic b c 2 nên P (x) = x2
+ ax ± 1.
Giả sả R (x) =
n
i=0
aixi
; ai ∈ {−1; 1}.
Goi z ∈ C là m®t nghi m của R (x) sao cho |z| > 1.
Khi đó
n−1
|z| = |z | = .
ai
zi
. ≤
n−1
|z| =
n−1
|z| = |z| − 1
.
Do đó
.
i=0
an .
n
i=0 i=0 |z| − 1
z n |z| − 1
|z| − 1
⇔ |z| (|z| − 1) ≤ |z| − 1 ⇔ |z| (|z| − 1) ≤ −1
nên |z| < 2.
V y moi nghi m của R (x) đeu có modul nhỏ hơn 2.
Giả sả z1, z2 là 2 nghi m của P (x) ∈ Z [x] thì z1, z2 là 2 nghi m của R (x) .
Suy ra |z1| ; |z2| < 2, lại có theo định lý Viet thì
|z1z2| = |z1| |z2| = 1
nên
|z1| ≥ |z2|
1 ≤ |z1| < 2
0 ≤ |z2| ≤ 1
M t khác
|a| = |z1 + z2| ≤ |z1| + |z2| < 3
nên a ∈ {±2; ± 1; 0}.
+) Trường hợp 1: Với a = 0, ta được
Q (x) = x + 1
P (x) = x2
− 1
Q (x) = 1
P (x) = x2
+ 1
Σ Σ
( (
∨
53
Viết đề tài giá sinh viên – ZALO:0973.287.149-TEAMLUANVAN.COM
"
Σ Σ
(
n n
2
2
+) Trường hợp 2: Với a = ±1, ta được P (x) = x2
± x ± 1 nên Q (x) = 1.
P (x) = x2
± 2x + 1 ⇒ Q (x) = x ± 1
+) Trường hợp 3: Với a = ±2 thì
P (x) = x2
± 2x − 1 ⇒ Q (x) do ∃zo , |zo | > 2
3.2.2 Sfi dnng công thfíc n i suy Lagrange đe xác định h so của đa
thfíc.
Bài toán 3.18. Cho đa thác P (x) = ax2
+bx+c, (a =
/ 0). Cháng minh rang ton
tại không quá 1 đa thác Q (x) b c n thoả mãn đieu ki n P (Q (x)) = Q (P (x)).
Lài giai. Giả sả ton tại Q1 (x) =
Ta có
n
i=1
2
aixi
; Q2 (x) =
n
i=1
bixi
.
Σ
n
i
P (Q1 (x)) = Q1 (P (x)) ⇔ aQ1
P (Q2 (x)) = Q2 (P (x)) ⇔ aQ2
(x) + bQ1 (x) + c =
2
(x) + bQ2 (x) + c =
i=1
Σ
n
aiP
biP
(x), (1)
.
i
(x) , (2)
So sánh h so của x2n
trong (1) , (2) , ta được
i=1
a.a2
= a .an
a.b2
n = bn.an
⇔
an = an−1
bn = an−1
nên an = bn.
Tà đó, ta đ t R (x) = Q1 (x) − Q2 (x) , suy ra deg (R (x)) ≤ n − 1.
Ta có
R (P (x)) = Q1 (P (x)) − Q2 (P (x)) = P (Q1 (x)) − P (Q2 (x))
= aQ1 (x) + bQ1 (x) + c − aQ2 (x) + bQ2 (x) + c
= (Q1 (x) − Q2 (x)) (Q1 (x) + Q2 (x) + b)
= R (x) .T (x) , (∗) .
Nh n thay trong (∗) deg R (x) = k nên deg (V T) = kn, deg (V P) = k +
n; (k < n) không xảy ra.
V y ta có đieu phải cháng minh.
Bài toán 3.19. Cho 4 so nguyên to khác nhau p1, p2, p3, p4. Cháng minh rang
không ton tại đa thác Q (x) b c 3 có h so nguyên thỏa mãn đieu ki n
|Q (p1)| = |Q (p2)| = |Q (p3)| = |Q (p4)| = 3.
(
54
Viết đề tài giá sinh viên – ZALO:0973.287.149-TEAMLUANVAN.COM
"
⇔
Lài giai. Giả sả ton tại đa thác Q (x) b c 3 có h so nguyên thỏa mãn |Q (p1)| =
|Q (p2)| = |Q (p3)| = |Q (p4)| = 3.
Ta xét các trường hợp sau
Trường hợp 1: Q (pi) = 3, i = 1, 2, 3, 4
Xét Q (x) = P (x) − 3 ta có deg (P (x)) = 3. Suy ra P (pi) = 6, i = 1, 2, 3, 4
v y P (x) ≡ 0. Vô lý
Trường hợp 2: Khi 3 trong 4 giá trị Q (pi) , i = 1, 2, 3, 4 bang nhau. Giả sả
Q (pi) = −3, i = 1, 2, 3, Q (p4) = 3
Q (x) = a (x − p1) (x − p2) (x − p3) − 3; a ∈ Z
Ta có ⇒ Q (p4) = 3 = a (p4 − p1) (p4 − p2) (p4 − p3) − 3
⇒ a (p4 − p1) (p4 − p2) (p4 − p3) = 6
Ta thay tích của 4 so nguyên khác nhau=6, vô lý.
Trường hợp 3: Trong 4 giá trị Q (pi) , i = 1, 2, 3, 4 có 2 giá trị bang 3 và 2 giá
trị bang -3
Giả sả Q (pi) = −3, i = 1, 2, Q (p4) = Q (p3) = 3
Ta có Q (x) = (x − p1) (x − p2) G (x)−3 ⇒ Q (p3) = (p3 − p1) (p3 − p2) G (p3)−
3 = 3,
suy ra (p3 − p1) (p3 − p2) G (p3) = 6, vô lý.
Vì neu p1 = 2 thì (Q (p3) − Q (2)) . (p3 − 2) |6 suy ra p3 ∈ {3; 5}, tương tự
p4 ∈ {3; 5}. Tà đó suy ra p3 = 3, p4 = 5.
Lại có (p4 − p1) (p4 − p2) |6 nên (5 − p2) |2 suy ra p2 ∈ {3; 7} và p2 = 7. Khi
đó (p3 − p1) (p3 − p2) = −4, vô lý.
Tương tự, với p2 = 2; p3 = 2; p4 = 2, vô lý.
Suy ra các so pi là lẻ nên (p3 − p1) (p3 − p2) .4; (p4 − p1) (p4 − p2) .4, vô lý.
V y ta có đieu phải cháng minh.
Bài toán 3.20. Tìm tat cả các đa thác P(x) với h so thực thoả mãn phương
trình P x2
= P2
(x) với moi x thu®c R.
Lài giai. Với P (x) = C, C là hang so. Khi đó ta có C = C2
C = 0
C = 1
V y P (x) = 0; P (x) = 1.
Với P (x) = anxn
+ · · · + a1x + a0 với các h so thực không đong thời bang 0.
Khi đó ta có P x2
= an x2
+· · ·+a x2
+a ; P2
(x) = [a xn
+ · · · + a x + a ]2
n
0 n 1 0
Giả sả k là so lớn nhat bé hơn n sao cho ak 0. Ta đong nhat h so của xn+k
ta được 0 = 2anak. Tái với giả thiet. Do đó P (x) = anxn
. So sánh h so của x2n
ta nh n được an = 1
1
55
Viết đề tài giá sinh viên – ZALO:0973.287.149-TEAMLUANVAN.COM
2 2
2 2
2 2
Khi đó (∗) trở thành Q x2
= Q2
(x)
2
2
V y P (x) = xn
.
Bài toán 3.21. Tìm tat cả các đa thác P (x) thỏa mãn h thác P x2
− 2x =
[P (x − 2)]2
với moi giá trị thực của x.
Lài giai. Tà giả thiet của bài toán ta được
P (x − 1)2
− 1 = [P ((x − 1) − 1)]2
. (∗)
Đ t Q (x) = P (x − 1) ⇒ Q x2
= P x2
− 1 ; Q2
(x) = P2
(x − 1)
Áp dụng ket quả của Bài toán 3.20 ta được Q (x) = 0, Q (x) = 1, Q (x) = xn
Do đó ta được ket quả P (x) = 0, P (x) = 1, P (x) = (x + 1)n
.
Bài toán 3.22. Tìm tat cả các đa thác P (x) h so thực thỏa mãn P x2
+
x [3P (x) + P (−x)] = P 2
(x) + 2x2
với moi giá trị thực của x.
Lài giai. Tà giả thiet của bài toán tat hay x bởi −x ta nh n được
P x2
− x [3P (−x) + P (x)] = P 2
(−x) + 2x2
.
Trà ve với ve phương trình cho nhau, ta nh n được
[P (x) + P (−x)] [P (x) − P (−x) − 4x] = 0.
Khi P (x) = −P (−x). Thay vào giả thiet ban đau ta được P x2
− x2
=
[P (x) − x] .
Đ t Q (x) = P (x) − x khi đó ta được Q x = Q (x). Theo ket quả của
Bài toán 3.20, ta có Q (x) = 0, Q (x) = 1, Q (x) = xn
hay P (x) = x, P (x) =
x + 1, P (x) = xn
+ x.
So sánh với P (x) − P (−x) = 0 ta nh n được P (x) = x, P (x) = x2k+1
+ x.
Khi P (x)−P (−x)−4x = 0 Thay vào giả thiet ban đau ta được P x −2x =
[P (x) − 2x] .
Đ t Q (x) = P (x) − 2x, khi đó ta được Q x = Q (x).
Theo ket quả của Bài toán 3.20, ta có Q (x) = 0, Q (x) = 1, Q (x) = xn
hay
P (x) = 2x, P (x) = 2x+1, P (x) = xn
+2x. Đoi chieu với P (x)−P (−x)−4x =
0 ta nh n được P (x) = 2x, P (x) = x2k
+ 2x.
56
Viết đề tài giá sinh viên – ZALO:0973.287.149-TEAMLUANVAN.COM
Σ
(
3.2.3 M t so bài toán xác định đa thfíc khác không liên quan đen
các công thfíc n i suy.
Bài toán 3.23. Tìm tat cả các đa thác P (x) ∈ Z [x] thỏa mãn
(P (a) + P (b) + P (c)) . (a + b + c)
với moi so nguyên a, b, c.
Lài giai. Ta luôn có (a + b + c) |(P (a) − P (−b − c)) lại có theo giả thiet
(a + b + c) |(P (a) + P (b) + P (c))
tà đó suy ra (a + b + c) |(P (b) + P (c) + P (−b − c)) đúng với moi so nguyên
a, b, c. Co định so nguyên b, c. Khi đó ta có (a + b + c) |(P (b) + P (c) + P (−b − c))
với moi so nguyên a. Suy ra ton tại so nguyên a đủ lớn đe
(a + b + c) |(P (b) + P (c) + P (−b − c))
và a + b + c > P (b) + P (c) + P (−b − c), (hien nhiên ton tại vô so so nguyên a
như the) tà đó ta suy ra P (b) + P (c) + P (−b − c) ≡ 0 với moi c p so nguyên
b, c. Chon b = c ta được 2P (b) = −P (−2b).
Giả sả P (x) =
n
i=0
aixi
. Đong nhat h so của xn
ta nh n được 2anxn
+
an(−2x)n
= 0 ⇒ n = 1 ⇒ P (x) = mx + k
Theo giả thiet (a + b + c) |(m (a + b + c) + 3k) ⇒ k = 0
V y P (x) = mx, m ∈ Z.
Bài toán 3.24. Tìm tat cả các đa thác P (x) ∈ Z [x] thỏa mãn P (7) .5, P (5) .7
và P (12) không chia het cho 35.
Lài giai. Giả sả ton tại P (x) ∈ Z [x] thỏa mãn P (7) .5, P (5) .7
Ta có
(P (12) − P (7)) .5 ⇒ P (12) .5 .
(P (12) − P (5)) .7 ⇒ P (12) .7
⇒ P (12) .35, trái với giả thiet của bài
toán. V y không ton tại đa thác nào thỏa mãn yêu cau bài toán.
Bài toán 3.25. Tìm tat cả các đa thác P (x) b c n thỏa mãn đieu ki n P x2
− y2
=
P (x + y) .P (x − y) , ∀x, y ∈ R.
Lài giai. Tà giả thiet của bài toán ta có đieu ki n tương đương là P (xy) =
P (x) .P (y) , ∀x, y ∈ R.
57
Viết đề tài giá sinh viên – ZALO:0973.287.149-TEAMLUANVAN.COM
x
x
P
Cho x = y = 0 ⇒ P (0) = 0 ∨ P (0) = 1.
+) Trường hợp 1. Neu P (0) = 0 khi đó P (x) = xQ (x) , deg Q = n − 1 thay
vào P (xy) = P (x) .P (y) , ∀x, y ∈ R. ta nh n được Q (xy) = Q (x) .Q (y) , ∀x, y ∈
R. suy ra Q (x) = 1 ∨ Q (x) = x.Q1 (x) , deg Q1 = n − 2
Ta nh n được P (x) = 1 ∨ P (x) = xn
.
+) Trường hợp 2. Neu P (0) = 1, cho y = 0 thay vào P (xy) = P (x) .P (y) , ∀x, y ∈
R. ta được P (x) = 1.
Thả lại các ket quả ta thay đa thác này thỏa mãn yêu cau bài ra.
Bài toán 3.26. Tìm tat cả các đa thác P (x) ∈ R [x] thỏa mãn đieu ki n
P (x + y) = P (x) + P (y) + 2xy, ∀x, y ∈ R.
Lài giai. Xét x = y = 0 ⇒ P (0) = 0, tà giả thiet của bài toán ta có
P′
(x) = lim
P (x + y) − P (x) P (y) + 2xy
= lim = 2x + P′ (0) .
y→0 y
∫x
′
y→0 y
2
Thả lại ta thay đa thác P (x) = x2
+ ax thỏa mãn yêu cau bài toán.
Bài toán 3.27. Tìm tat cả các đa thác P1 (x) , P2 (x) , P3 (x) , P4 (x) sao cho với
moi x, y, z, t ∈ R thỏa mãn xy − zt = 1 thì P1 (x) P2 (y) − P3 (z) P4 (t) = 1.
Lài giai. Giả sả deg P1 = n1, deg P2 = n2. Goi N ∈ N là so tự nhiên có so các
ước ϕ (N) lớn hơn n1 + n2.
N
Chon y =
được x
, (x |N ) ; z = 1; t = N − 1 thay vào giả thiet của bài toán ta
Nh n thay
P1 (x) P2
N
− P (1) P4 (N − 1) = 1. (∗)
deg P1 (x) P2
N
≤ n + n2 < ϕ (N) . (∗∗)
M t khác tà (∗) ta suy ra Q (x) = P1 (x) P2 có ϕ (N) > n + n
N
2 nghi m.
V y deg Q (x) = 0 tà đó suy ra P1 (x) = axn
; P2 (x) = bxn
.
Hoàn toàn tương tự ta cũng có ket quả P3 (x) = cxm
; P4 (x) = dxm
.
+) Cho x = y = 1; z = t = 0 ta nh n được ab = 1.
0
x
Do đó P (x) = P (x) − P (0) = (t) dt = x + ax.
3
1
1
58
Viết đề tài giá sinh viên – ZALO:0973.287.149-TEAMLUANVAN.COM
+) Cho x = y = 0; z = t = 1 ta nh n được cd = 1.
+) Cho x = z = 1; t = y − 1 ta nh n được (yn
− (y − 1)m
= 0, ∀y ∈ R) ⇔
m = n = 1.
V y các đa thác nh n được là: P1 (x) = ax; P2 (x) =
x
x
a
, (a /= 0) ; P3 (x) =
cx; P4 (x) =
c
, (c /= 0). Thả lại thay thỏa mãn giả thiet của bài toán.
59
Viết đề tài giá sinh viên – ZALO:0973.287.149-TEAMLUANVAN.COM
KET LU N
Lu n văn ”M®t so áng dụng của công thác n®i suy Lagrange và Hermite”
đã trình bày được nhǎng van đe sau:
1. Lu n văn trình bày các bài toán n®i suy Lagrange và n®i suy Hermite
2. Tiep theo, trình bày m®t so áng dụng của công thác n®i suy Lagrange
và Hermite đe tìm nguyên hàm của các hàm phân thác.
3. Xét các bài toán ve đa thác nh n giá trị nguyên, các bài toán ve xác
định đa thác.
60
Viết đề tài giá sinh viên – ZALO:0973.287.149-TEAMLUANVAN.COM
TÀI LI U THAM KHẢO
[A] Tieng Vi t
[1] Nguyen Văn M u (2002), Đa thúc đại so và phân thúc hũu ty, NXB Giáo
dục.
[2] Nguyen Văn M u (2016), N®i suy đa thúc, NXB ĐHQG Hà N®i.
[3] Nguyen Văn M u, Lê Ngoc Lăng, Phạm the Long, Nguyen Minh Tuan (2006),
Các đe thi olympic Toán sinh viên toàn quoc, NXB Giáo dục.
[4] Nguyen Thúy Vân (2014), M®t so thu¾t toán n®i suy đe xác đ nh các nguyên
hàm sơ cap của hàm hũu ty, Lu n văn Thạc sy, ĐH Thái Nguyên.
[B] Tieng Anh
[5] Paulo Ney de Sausa, Jorge- Nume Silva (1998), Berkeley Problems in Math-
ematics, Springer.

More Related Content

Similar to M T So Ứng Dụng Của Công Thức N I Suy Lagrange Và Hermite.docx

Tich phan %28 nguyen duy khoi%29
Tich phan %28 nguyen duy khoi%29Tich phan %28 nguyen duy khoi%29
Tich phan %28 nguyen duy khoi%29
trongphuckhtn
 
Tich phan (nguyen duy khoi)
Tich phan (nguyen duy khoi)Tich phan (nguyen duy khoi)
Tich phan (nguyen duy khoi)
roggerbob
 
Chuyen de tich phan cuc hay
Chuyen de tich phan cuc hayChuyen de tich phan cuc hay
Chuyen de tich phan cuc hay
Oanh MJ
 

Similar to M T So Ứng Dụng Của Công Thức N I Suy Lagrange Và Hermite.docx (20)

M T So Dạng Toán Cực Tr± Trong L P Hàm Mũ Và Hàm Hyperbolic.docx
M T So Dạng Toán Cực Tr± Trong L P Hàm Mũ Và Hàm Hyperbolic.docxM T So Dạng Toán Cực Tr± Trong L P Hàm Mũ Và Hàm Hyperbolic.docx
M T So Dạng Toán Cực Tr± Trong L P Hàm Mũ Và Hàm Hyperbolic.docx
 
Khảo sát nghi m của các phương trình sinh b i đạo hàm và nguyên hàm Của m t đ...
Khảo sát nghi m của các phương trình sinh b i đạo hàm và nguyên hàm Của m t đ...Khảo sát nghi m của các phương trình sinh b i đạo hàm và nguyên hàm Của m t đ...
Khảo sát nghi m của các phương trình sinh b i đạo hàm và nguyên hàm Của m t đ...
 
Bất đẳng thức Trong lớp các hàm lượng giác và lượng giác ngược.docx
Bất đẳng thức Trong lớp các hàm lượng giác và lượng giác ngược.docxBất đẳng thức Trong lớp các hàm lượng giác và lượng giác ngược.docx
Bất đẳng thức Trong lớp các hàm lượng giác và lượng giác ngược.docx
 
Phương Pháp Lư Ng Giác Giải Phương Trình Đa Thức Và M T So Dạng Toán.docx
Phương Pháp Lư Ng Giác Giải Phương Trình Đa Thức Và M T So Dạng Toán.docxPhương Pháp Lư Ng Giác Giải Phương Trình Đa Thức Và M T So Dạng Toán.docx
Phương Pháp Lư Ng Giác Giải Phương Trình Đa Thức Và M T So Dạng Toán.docx
 
Một số phương pháp giải các đề thi olympic Về phương trình diophant.docx
Một số phương pháp giải các đề thi olympic Về phương trình diophant.docxMột số phương pháp giải các đề thi olympic Về phương trình diophant.docx
Một số phương pháp giải các đề thi olympic Về phương trình diophant.docx
 
Về Tổng Gauss Và Một Số Ứng Dụng.docx
Về Tổng Gauss Và Một Số Ứng Dụng.docxVề Tổng Gauss Và Một Số Ứng Dụng.docx
Về Tổng Gauss Và Một Số Ứng Dụng.docx
 
Kĩ thuật tổng hợp Giải bất phương trình hỗn hợp.docx
Kĩ thuật tổng hợp Giải bất phương trình hỗn hợp.docxKĩ thuật tổng hợp Giải bất phương trình hỗn hợp.docx
Kĩ thuật tổng hợp Giải bất phương trình hỗn hợp.docx
 
Hàm Đơn Đi›U, Tựa Đơn Đi›U Và Một Số Ứng Dụng Của Phép Đơn Đi›U Hóa Hàm Số.docx
Hàm Đơn Đi›U, Tựa Đơn Đi›U Và Một Số Ứng Dụng Của Phép Đơn Đi›U Hóa Hàm Số.docxHàm Đơn Đi›U, Tựa Đơn Đi›U Và Một Số Ứng Dụng Của Phép Đơn Đi›U Hóa Hàm Số.docx
Hàm Đơn Đi›U, Tựa Đơn Đi›U Và Một Số Ứng Dụng Của Phép Đơn Đi›U Hóa Hàm Số.docx
 
Một Số Dạng Toán Cực Trị Trong Lîp Hàm Mũ Và Hàm Hyperbolic.docx
Một Số Dạng Toán Cực Trị Trong Lîp Hàm Mũ Và Hàm Hyperbolic.docxMột Số Dạng Toán Cực Trị Trong Lîp Hàm Mũ Và Hàm Hyperbolic.docx
Một Số Dạng Toán Cực Trị Trong Lîp Hàm Mũ Và Hàm Hyperbolic.docx
 
Ve Bat Đang Thức Ho¨ Lder Và Áp Dụng.docx
Ve  Bat  Đang  Thức  Ho¨ Lder  Và  Áp  Dụng.docxVe  Bat  Đang  Thức  Ho¨ Lder  Và  Áp  Dụng.docx
Ve Bat Đang Thức Ho¨ Lder Và Áp Dụng.docx
 
Tich phan %28 nguyen duy khoi%29
Tich phan %28 nguyen duy khoi%29Tich phan %28 nguyen duy khoi%29
Tich phan %28 nguyen duy khoi%29
 
Tich phan (nguyen duy khoi)
Tich phan (nguyen duy khoi)Tich phan (nguyen duy khoi)
Tich phan (nguyen duy khoi)
 
Tổng quát về tích phân
Tổng quát về tích phân Tổng quát về tích phân
Tổng quát về tích phân
 
M T So Dạng Toán Ve Dãy So Sinh B I Các Hàm So Sơ Cap.docx
M T So Dạng Toán Ve Dãy So Sinh B I Các Hàm So Sơ Cap.docxM T So Dạng Toán Ve Dãy So Sinh B I Các Hàm So Sơ Cap.docx
M T So Dạng Toán Ve Dãy So Sinh B I Các Hàm So Sơ Cap.docx
 
Chuyen de tich phan cuc hay
Chuyen de tich phan cuc hayChuyen de tich phan cuc hay
Chuyen de tich phan cuc hay
 
Toán Tử Sai Phân Và Ứng Dụng Vào Giải Toán Sơ Cấp.docx
Toán Tử Sai Phân Và Ứng Dụng Vào Giải Toán Sơ Cấp.docxToán Tử Sai Phân Và Ứng Dụng Vào Giải Toán Sơ Cấp.docx
Toán Tử Sai Phân Và Ứng Dụng Vào Giải Toán Sơ Cấp.docx
 
Bat Phương Trình Hàm Sinh B I Các Đại Lư Ng Trung Bình B C Tùy Ý Và Các Dạng ...
Bat Phương Trình Hàm Sinh B I Các Đại Lư Ng Trung Bình B C Tùy Ý Và Các Dạng ...Bat Phương Trình Hàm Sinh B I Các Đại Lư Ng Trung Bình B C Tùy Ý Và Các Dạng ...
Bat Phương Trình Hàm Sinh B I Các Đại Lư Ng Trung Bình B C Tùy Ý Và Các Dạng ...
 
Luận văn: Bài toán cực trị với đa thức đối xứng ba biến, HAY - Gửi miễn phí q...
Luận văn: Bài toán cực trị với đa thức đối xứng ba biến, HAY - Gửi miễn phí q...Luận văn: Bài toán cực trị với đa thức đối xứng ba biến, HAY - Gửi miễn phí q...
Luận văn: Bài toán cực trị với đa thức đối xứng ba biến, HAY - Gửi miễn phí q...
 
Đề tài: Lớp bất đẳng thức, bài toán cực trị với đa thức đối xứng, 9đ
Đề tài: Lớp bất đẳng thức, bài toán cực trị với đa thức đối xứng, 9đĐề tài: Lớp bất đẳng thức, bài toán cực trị với đa thức đối xứng, 9đ
Đề tài: Lớp bất đẳng thức, bài toán cực trị với đa thức đối xứng, 9đ
 
Bai giang-toan-kinh-te-tin-hoc
Bai giang-toan-kinh-te-tin-hocBai giang-toan-kinh-te-tin-hoc
Bai giang-toan-kinh-te-tin-hoc
 

More from DV Viết Luận văn luanvanmaster.com ZALO 0973287149

More from DV Viết Luận văn luanvanmaster.com ZALO 0973287149 (20)

Ảnh Hưởng Của Marketing Quan Hệ Đến Lòng Trung Thành Của Khách Hàng.Tình Huốn...
Ảnh Hưởng Của Marketing Quan Hệ Đến Lòng Trung Thành Của Khách Hàng.Tình Huốn...Ảnh Hưởng Của Marketing Quan Hệ Đến Lòng Trung Thành Của Khách Hàng.Tình Huốn...
Ảnh Hưởng Của Marketing Quan Hệ Đến Lòng Trung Thành Của Khách Hàng.Tình Huốn...
 
Phát triển nguồn nhân lực tại Uỷ ban nhân dân huyện Trà Bồng, tỉnh Quảng Ngãi...
Phát triển nguồn nhân lực tại Uỷ ban nhân dân huyện Trà Bồng, tỉnh Quảng Ngãi...Phát triển nguồn nhân lực tại Uỷ ban nhân dân huyện Trà Bồng, tỉnh Quảng Ngãi...
Phát triển nguồn nhân lực tại Uỷ ban nhân dân huyện Trà Bồng, tỉnh Quảng Ngãi...
 
Báo cáo tốt Nghiệp tài chính hợp nhất tại tổng công ty Indochina gol...
Báo cáo tốt Nghiệp  tài chính hợp nhất tại tổng công ty Indochina gol...Báo cáo tốt Nghiệp  tài chính hợp nhất tại tổng công ty Indochina gol...
Báo cáo tốt Nghiệp tài chính hợp nhất tại tổng công ty Indochina gol...
 
Tạo động lực thúc đẩy nhân viên làm việc tại ngân hàng TMCP Ngoại Thương Việt...
Tạo động lực thúc đẩy nhân viên làm việc tại ngân hàng TMCP Ngoại Thương Việt...Tạo động lực thúc đẩy nhân viên làm việc tại ngân hàng TMCP Ngoại Thương Việt...
Tạo động lực thúc đẩy nhân viên làm việc tại ngân hàng TMCP Ngoại Thương Việt...
 
Phát triển công nghiệp trên địa bàn Thành phố Tam Kỳ, Tỉnh Quảng Na...
Phát triển công nghiệp trên địa bàn Thành phố Tam Kỳ, Tỉnh Quảng Na...Phát triển công nghiệp trên địa bàn Thành phố Tam Kỳ, Tỉnh Quảng Na...
Phát triển công nghiệp trên địa bàn Thành phố Tam Kỳ, Tỉnh Quảng Na...
 
Giải pháp phát triển cho vay xuất nhập khẩu tại ngân hàng NN&PTNN ch...
Giải pháp phát triển cho vay xuất nhập khẩu tại ngân hàng NN&PTNN ch...Giải pháp phát triển cho vay xuất nhập khẩu tại ngân hàng NN&PTNN ch...
Giải pháp phát triển cho vay xuất nhập khẩu tại ngân hàng NN&PTNN ch...
 
Hoàn thiện công tác lập báo cáo tài chính hợp nhất tại tổng công ...
Hoàn thiện công tác lập báo cáo tài chính hợp nhất tại tổng công ...Hoàn thiện công tác lập báo cáo tài chính hợp nhất tại tổng công ...
Hoàn thiện công tác lập báo cáo tài chính hợp nhất tại tổng công ...
 
Luận Văn Thạc Sĩ Quản trị thành tích nhân viên tại Cục Hải quan TP Đà Nẵng.doc
Luận Văn Thạc Sĩ  Quản trị thành tích nhân viên tại Cục Hải quan TP Đà Nẵng.docLuận Văn Thạc Sĩ  Quản trị thành tích nhân viên tại Cục Hải quan TP Đà Nẵng.doc
Luận Văn Thạc Sĩ Quản trị thành tích nhân viên tại Cục Hải quan TP Đà Nẵng.doc
 
Hoàn thiện công tác quản lý thuế thu nhập cá nhân tại cục thuế Tỉ...
Hoàn thiện công tác quản lý thuế thu nhập cá nhân tại cục thuế Tỉ...Hoàn thiện công tác quản lý thuế thu nhập cá nhân tại cục thuế Tỉ...
Hoàn thiện công tác quản lý thuế thu nhập cá nhân tại cục thuế Tỉ...
 
Đề Tài Phát triển bền vững nông nghiệp Huyện Ba Tơ, Tỉnh Quảng Ngãi....
Đề Tài Phát triển bền vững nông nghiệp Huyện Ba Tơ, Tỉnh Quảng Ngãi....Đề Tài Phát triển bền vững nông nghiệp Huyện Ba Tơ, Tỉnh Quảng Ngãi....
Đề Tài Phát triển bền vững nông nghiệp Huyện Ba Tơ, Tỉnh Quảng Ngãi....
 
Hoàn thiện công tác bảo trợ xã hội trên địa bàn huyện Phong Điền, tỉnh Thừa T...
Hoàn thiện công tác bảo trợ xã hội trên địa bàn huyện Phong Điền, tỉnh Thừa T...Hoàn thiện công tác bảo trợ xã hội trên địa bàn huyện Phong Điền, tỉnh Thừa T...
Hoàn thiện công tác bảo trợ xã hội trên địa bàn huyện Phong Điền, tỉnh Thừa T...
 
Đề Tài Luận VănPhát triển sản phẩm du lịch tại thành phố Đà Nẵng.doc
Đề Tài Luận VănPhát triển sản phẩm du lịch tại thành phố Đà Nẵng.docĐề Tài Luận VănPhát triển sản phẩm du lịch tại thành phố Đà Nẵng.doc
Đề Tài Luận VănPhát triển sản phẩm du lịch tại thành phố Đà Nẵng.doc
 
Đào tạo nghề cho lao động thuộc diện thu hồi đất trên địa bàn Thàn...
Đào tạo nghề cho lao động thuộc diện thu hồi đất trên địa bàn Thàn...Đào tạo nghề cho lao động thuộc diện thu hồi đất trên địa bàn Thàn...
Đào tạo nghề cho lao động thuộc diện thu hồi đất trên địa bàn Thàn...
 
Tóm Tắt Luận Văn Thạc Sĩ Quản Trị Kinh Doanh Xây dựng chính sách Marketing tạ...
Tóm Tắt Luận Văn Thạc Sĩ Quản Trị Kinh Doanh Xây dựng chính sách Marketing tạ...Tóm Tắt Luận Văn Thạc Sĩ Quản Trị Kinh Doanh Xây dựng chính sách Marketing tạ...
Tóm Tắt Luận Văn Thạc Sĩ Quản Trị Kinh Doanh Xây dựng chính sách Marketing tạ...
 
Đề Tài Nghiên cứu rủi ro cảm nhận đối với mua hàng thời trang trực tuyến.docx
Đề Tài Nghiên cứu rủi ro cảm nhận đối với mua hàng thời trang trực tuyến.docxĐề Tài Nghiên cứu rủi ro cảm nhận đối với mua hàng thời trang trực tuyến.docx
Đề Tài Nghiên cứu rủi ro cảm nhận đối với mua hàng thời trang trực tuyến.docx
 
Giải pháp nâng cao động lực thúc đẩy người lao động tại công ty khai...
Giải pháp nâng cao động lực thúc đẩy người lao động tại công ty khai...Giải pháp nâng cao động lực thúc đẩy người lao động tại công ty khai...
Giải pháp nâng cao động lực thúc đẩy người lao động tại công ty khai...
 
Giải pháp phát triển dịch vụ ngân hàng điện tử tại ngân hàng đầu ...
Giải pháp phát triển dịch vụ ngân hàng điện tử tại ngân hàng đầu ...Giải pháp phát triển dịch vụ ngân hàng điện tử tại ngân hàng đầu ...
Giải pháp phát triển dịch vụ ngân hàng điện tử tại ngân hàng đầu ...
 
Giải pháp phát triển dịch vụ ngân hàng điện tử tại ngân hàng đầu ...
Giải pháp phát triển dịch vụ ngân hàng điện tử tại ngân hàng đầu ...Giải pháp phát triển dịch vụ ngân hàng điện tử tại ngân hàng đầu ...
Giải pháp phát triển dịch vụ ngân hàng điện tử tại ngân hàng đầu ...
 
Quản trị quan hệ khách hàng tại Chi nhánh Viettel Đà Nẵng – Tập đoàn Viễn thô...
Quản trị quan hệ khách hàng tại Chi nhánh Viettel Đà Nẵng – Tập đoàn Viễn thô...Quản trị quan hệ khách hàng tại Chi nhánh Viettel Đà Nẵng – Tập đoàn Viễn thô...
Quản trị quan hệ khách hàng tại Chi nhánh Viettel Đà Nẵng – Tập đoàn Viễn thô...
 
Đề Tài Đánh giá thành tích đội ngũ giảng viên trường Đại Học Phạm ...
Đề Tài Đánh giá thành tích đội ngũ giảng viên trường Đại Học Phạm ...Đề Tài Đánh giá thành tích đội ngũ giảng viên trường Đại Học Phạm ...
Đề Tài Đánh giá thành tích đội ngũ giảng viên trường Đại Học Phạm ...
 

Recently uploaded

Logic học và phương pháp nghiên cứu khoa học
Logic học và phương pháp nghiên cứu khoa họcLogic học và phương pháp nghiên cứu khoa học
Logic học và phương pháp nghiên cứu khoa học
K61PHMTHQUNHCHI
 
Everybody Up 1 - Unit 5 - worksheet grade 1
Everybody Up 1 - Unit 5 - worksheet grade 1Everybody Up 1 - Unit 5 - worksheet grade 1
Everybody Up 1 - Unit 5 - worksheet grade 1
mskellyworkmail
 

Recently uploaded (20)

Logic học và phương pháp nghiên cứu khoa học
Logic học và phương pháp nghiên cứu khoa họcLogic học và phương pháp nghiên cứu khoa học
Logic học và phương pháp nghiên cứu khoa học
 
TỔNG HỢP HƠN 100 ĐỀ THI THỬ TỐT NGHIỆP THPT HÓA HỌC 2024 - TỪ CÁC TRƯỜNG, TRƯ...
TỔNG HỢP HƠN 100 ĐỀ THI THỬ TỐT NGHIỆP THPT HÓA HỌC 2024 - TỪ CÁC TRƯỜNG, TRƯ...TỔNG HỢP HƠN 100 ĐỀ THI THỬ TỐT NGHIỆP THPT HÓA HỌC 2024 - TỪ CÁC TRƯỜNG, TRƯ...
TỔNG HỢP HƠN 100 ĐỀ THI THỬ TỐT NGHIỆP THPT HÓA HỌC 2024 - TỪ CÁC TRƯỜNG, TRƯ...
 
Everybody Up 1 - Unit 5 - worksheet grade 1
Everybody Up 1 - Unit 5 - worksheet grade 1Everybody Up 1 - Unit 5 - worksheet grade 1
Everybody Up 1 - Unit 5 - worksheet grade 1
 
Giải pháp nâng cao chất lượng sản phẩm ở Công ty TNHH Sơn Alex Việt Nam
Giải pháp nâng cao chất lượng sản phẩm ở Công ty TNHH Sơn Alex Việt NamGiải pháp nâng cao chất lượng sản phẩm ở Công ty TNHH Sơn Alex Việt Nam
Giải pháp nâng cao chất lượng sản phẩm ở Công ty TNHH Sơn Alex Việt Nam
 
Hoàn thiện công tác kiểm soát chi NSNN qua Kho bạc Nhà nước huyện Tri Tôn – t...
Hoàn thiện công tác kiểm soát chi NSNN qua Kho bạc Nhà nước huyện Tri Tôn – t...Hoàn thiện công tác kiểm soát chi NSNN qua Kho bạc Nhà nước huyện Tri Tôn – t...
Hoàn thiện công tác kiểm soát chi NSNN qua Kho bạc Nhà nước huyện Tri Tôn – t...
 
Mở rộng hoạt động cho vay tiêu dùng tại Ngân hàng TMCP Hàng Hải Việt Nam (Mar...
Mở rộng hoạt động cho vay tiêu dùng tại Ngân hàng TMCP Hàng Hải Việt Nam (Mar...Mở rộng hoạt động cho vay tiêu dùng tại Ngân hàng TMCP Hàng Hải Việt Nam (Mar...
Mở rộng hoạt động cho vay tiêu dùng tại Ngân hàng TMCP Hàng Hải Việt Nam (Mar...
 
Báo cáo thực tập tốt nghiệp Kế toán tiền mặt tại Công ty trách nhiệm hữu hạn ...
Báo cáo thực tập tốt nghiệp Kế toán tiền mặt tại Công ty trách nhiệm hữu hạn ...Báo cáo thực tập tốt nghiệp Kế toán tiền mặt tại Công ty trách nhiệm hữu hạn ...
Báo cáo thực tập tốt nghiệp Kế toán tiền mặt tại Công ty trách nhiệm hữu hạn ...
 
Báo cáo tốt nghiệp Đánh giá công tác đào tạo và phát triển nguồn nhân lực tại...
Báo cáo tốt nghiệp Đánh giá công tác đào tạo và phát triển nguồn nhân lực tại...Báo cáo tốt nghiệp Đánh giá công tác đào tạo và phát triển nguồn nhân lực tại...
Báo cáo tốt nghiệp Đánh giá công tác đào tạo và phát triển nguồn nhân lực tại...
 
Báo cáo tốt nghiệp Đánh giá rủi ro môi trường từ ô nhiễm hữu cơ nước thải các...
Báo cáo tốt nghiệp Đánh giá rủi ro môi trường từ ô nhiễm hữu cơ nước thải các...Báo cáo tốt nghiệp Đánh giá rủi ro môi trường từ ô nhiễm hữu cơ nước thải các...
Báo cáo tốt nghiệp Đánh giá rủi ro môi trường từ ô nhiễm hữu cơ nước thải các...
 
Báo cáo tốt nghiệp Đánh giá rủi ro môi trường ô nhiễm hữu cơ trong nước thải ...
Báo cáo tốt nghiệp Đánh giá rủi ro môi trường ô nhiễm hữu cơ trong nước thải ...Báo cáo tốt nghiệp Đánh giá rủi ro môi trường ô nhiễm hữu cơ trong nước thải ...
Báo cáo tốt nghiệp Đánh giá rủi ro môi trường ô nhiễm hữu cơ trong nước thải ...
 
30 ĐỀ PHÁT TRIỂN THEO CẤU TRÚC ĐỀ MINH HỌA BGD NGÀY 22-3-2024 KỲ THI TỐT NGHI...
30 ĐỀ PHÁT TRIỂN THEO CẤU TRÚC ĐỀ MINH HỌA BGD NGÀY 22-3-2024 KỲ THI TỐT NGHI...30 ĐỀ PHÁT TRIỂN THEO CẤU TRÚC ĐỀ MINH HỌA BGD NGÀY 22-3-2024 KỲ THI TỐT NGHI...
30 ĐỀ PHÁT TRIỂN THEO CẤU TRÚC ĐỀ MINH HỌA BGD NGÀY 22-3-2024 KỲ THI TỐT NGHI...
 
TỔNG HỢP HƠN 100 ĐỀ THI THỬ TỐT NGHIỆP THPT VẬT LÝ 2024 - TỪ CÁC TRƯỜNG, TRƯ...
TỔNG HỢP HƠN 100 ĐỀ THI THỬ TỐT NGHIỆP THPT VẬT LÝ 2024 - TỪ CÁC TRƯỜNG, TRƯ...TỔNG HỢP HƠN 100 ĐỀ THI THỬ TỐT NGHIỆP THPT VẬT LÝ 2024 - TỪ CÁC TRƯỜNG, TRƯ...
TỔNG HỢP HƠN 100 ĐỀ THI THỬ TỐT NGHIỆP THPT VẬT LÝ 2024 - TỪ CÁC TRƯỜNG, TRƯ...
 
Quản lý dạy học phân hóa môn Toán tại các trường trung học cơ sở huyện Tam D...
Quản lý dạy học phân hóa môn Toán tại các trường trung học cơ sở huyện Tam D...Quản lý dạy học phân hóa môn Toán tại các trường trung học cơ sở huyện Tam D...
Quản lý dạy học phân hóa môn Toán tại các trường trung học cơ sở huyện Tam D...
 
TỔNG HỢP HƠN 100 ĐỀ THI THỬ TỐT NGHIỆP THPT TOÁN 2024 - TỪ CÁC TRƯỜNG, TRƯỜNG...
TỔNG HỢP HƠN 100 ĐỀ THI THỬ TỐT NGHIỆP THPT TOÁN 2024 - TỪ CÁC TRƯỜNG, TRƯỜNG...TỔNG HỢP HƠN 100 ĐỀ THI THỬ TỐT NGHIỆP THPT TOÁN 2024 - TỪ CÁC TRƯỜNG, TRƯỜNG...
TỔNG HỢP HƠN 100 ĐỀ THI THỬ TỐT NGHIỆP THPT TOÁN 2024 - TỪ CÁC TRƯỜNG, TRƯỜNG...
 
Báo cáo tốt nghiệp Đánh giá rủi ro quá trình xử lí nước cấp tại Chi nhánh Cấp...
Báo cáo tốt nghiệp Đánh giá rủi ro quá trình xử lí nước cấp tại Chi nhánh Cấp...Báo cáo tốt nghiệp Đánh giá rủi ro quá trình xử lí nước cấp tại Chi nhánh Cấp...
Báo cáo tốt nghiệp Đánh giá rủi ro quá trình xử lí nước cấp tại Chi nhánh Cấp...
 
Tóm tắt luận văn Hệ thống ca dao than thân người Việt từ góc nhìn thi pháp
Tóm tắt luận văn Hệ thống ca dao than thân người Việt từ góc nhìn thi phápTóm tắt luận văn Hệ thống ca dao than thân người Việt từ góc nhìn thi pháp
Tóm tắt luận văn Hệ thống ca dao than thân người Việt từ góc nhìn thi pháp
 
4.NGÂN HÀNG KĨ THUẬT SỐ-slide CHƯƠNG 3.pptx
4.NGÂN HÀNG KĨ THUẬT SỐ-slide CHƯƠNG 3.pptx4.NGÂN HÀNG KĨ THUẬT SỐ-slide CHƯƠNG 3.pptx
4.NGÂN HÀNG KĨ THUẬT SỐ-slide CHƯƠNG 3.pptx
 
Vận dụng thi pháp học vào phân tích truyện ngắn Chiếc thuyền ...
Vận dụng thi pháp học vào phân tích truyện ngắn Chiếc thuyền ...Vận dụng thi pháp học vào phân tích truyện ngắn Chiếc thuyền ...
Vận dụng thi pháp học vào phân tích truyện ngắn Chiếc thuyền ...
 
Báo cáo thực tập tốt nghiệp Phân tích thực trạng hoạt động bán hàng tại Công ...
Báo cáo thực tập tốt nghiệp Phân tích thực trạng hoạt động bán hàng tại Công ...Báo cáo thực tập tốt nghiệp Phân tích thực trạng hoạt động bán hàng tại Công ...
Báo cáo thực tập tốt nghiệp Phân tích thực trạng hoạt động bán hàng tại Công ...
 
Bài tập lớn môn Văn hóa kinh doanh và tinh thần khởi nghiệp Trình bày về triế...
Bài tập lớn môn Văn hóa kinh doanh và tinh thần khởi nghiệp Trình bày về triế...Bài tập lớn môn Văn hóa kinh doanh và tinh thần khởi nghiệp Trình bày về triế...
Bài tập lớn môn Văn hóa kinh doanh và tinh thần khởi nghiệp Trình bày về triế...
 

M T So Ứng Dụng Của Công Thức N I Suy Lagrange Và Hermite.docx

  • 1. ĐẠI HOC THÁI NGUYÊN TRƯ NG ĐẠI HOC KHOA HOC Tải tài liệu tại sividoc.com Viết đề tài giá sinh viên – ZALO:0973.287.149-TEAMLUANVAN.COM HOÀNG TH± NGA M T SO ỨNG DỤNG CỦA CÔNG THỨC N I SUY LAGRANGE VÀ HERMITE LU N VĂN THẠC SĨ TOÁN HOC THÁI NGUYÊN - 2018
  • 2. ĐẠI HOC THÁI NGUYÊN TRƯ NG ĐẠI HOC KHOA HOC Tải tài liệu tại sividoc.com Viết đề tài giá sinh viên – ZALO:0973.287.149-TEAMLUANVAN.COM HOÀNG TH± NGA M T SO ỨNG DỤNG CỦA CÔNG THỨC N I SUY LAGRANGE VÀ HERMITE Chuyên ngành: PHƯƠNG PHÁP TOÁN SƠ CAP Mã so: 8 46 01 13 LU N VĂN THẠC SĨ TOÁN HOC Người hướng dan khoa hoc: GS.TSKH. Nguyen Văn M u THÁI NGUYÊN - 2018
  • 3. i Viết đề tài giá sinh viên – ZALO:0973.287.149-TEAMLUANVAN.COM MỤC LỤC M ĐAU ii Chương 1. N i suy Lagrange và n i suy Hermite 1 1.1 Bài toán n®i suy Lagrange...................................................................... 1 1.2 Bài toán n®i suy Hermite........................................................................ 9 1.3 Bài toán n®i suy Lagrange - Newton.....................................................18 Chương 2. Ứng dnng n i suy tính nguyên hàm và tích phân các hàm phân thfíc 21 2.1 Nguyên hàm của hàm phân thác với các cực điem đơn........................21 2.2 Nguyên hàm của hàm phân thác với các cực điem b c tùy ý...............26 Chương 3. M t so dạng toán liên quan 43 3.1 M®t so bài toán ve đa thác nh n giá trị nguyên...................................43 3.2 M®t so bài toán xác định đa thác .........................................................50 3.2.1 Tìm đa thác khi biet các nghi m của nó. ..................................50 3.2.2 Sả dụng công thác n®i suy Lagrange đe xác định h so của đa thác. ......................................................................................53 3.2.3 M®t so bài toán xác định đa thác khác không liên quan đen các công thác n®i suy ..................................................................... 56 KET LU N 59 TÀI LI U THAM KHẢO 59
  • 4. ii Viết đề tài giá sinh viên – ZALO:0973.287.149-TEAMLUANVAN.COM M ĐAU Trong các kì thi hoc sinh giỏi toán các cap, Olympic Toán sinh viên, các bài toán liên quan tới đa thác thường xuyên được đe c p. Nhǎng dạng toán này thường được xem là thu®c loại khó, hơn nǎa phan kien thác ve n®i suy đa thác lại không nam trong chương trình chính thác của giáo trình Đại so và Giải tích b c trung hoc phő thông. Như ta đã biet, công thác n®i suy Lagrange đã được đe c p ở b c phő thông. Tuy nhiên công thác n®i suy Hermite chỉ có trong các tài li u chuyên khảo. Vì v y, tôi chon đe tài lu n văn ”M®t so áng dụng của công thác n®i suy Lagrange và Hermite”. Lu n văn nham cung cap m®t so áng dụng của công thác n®i suy Lagrange và Hermite đe tìm nguyên hàm của hàm phân thác. Lu n văn gom phan mở đau, ket lu n và 3 chương. Chương 1. N®i suy Lagrange và n®i suy Hermite. Chương 2. Úng dụng n®i suy tính nguyên hàm và tích phân các hàm phân thác Chương 3. M®t so dạng toán liên quan. Tiep theo, trong các chương đeu trình bày m®t h thong bài t p áp dụng giải các đe thi HSG quoc gia và Olympic liên quan. Lu n văn được thực hi n và hoàn thành tại Trường Đại hoc Khoa hoc, Đại hoc Thái Nguyên dưới sự hướng dan của GS.TSKH. Nguyen Văn M u. Xin được gải lời cảm ơn chân thành và sâu sac đen Thay, người đã t n tình hướng dan và chỉ đạo tác giả t p dượt nghiên cáu khoa hoc trong suot quá trình tìm hieu tài li u, viet và hoàn thi n Lu n văn. Đong thời em xin chân thành cảm ơn các quý thay cô trong B® môn toán, Khoa Khoa hoc Tự nhiên, các Thay Cô trường Đại hoc Khoa hoc Tự nhiên Hà N®i, các Thay Cô Vi n Toán hoc đã t n tình giảng dạy, quan tâm và tạo moi đieu ki n thu n lợi ve thủ tục hành chính đe em hoàn thành khoá hoc và bảo v lu n văn Thạc sĩ. Tôi cũng chân thành cảm ơn gia đình, bạn bè và cơ quan, đoàn the nơi tôi công tác là Trường Trung hoc Phő thông Thu Sơn, Sở Giáo dục và Đào tạo Hải Phòng, đã tạo moi đieu ki n ve v t chat lan tinh than trong quá trình hoc t p, nghiên cáu và viet lu n văn. Thái Nguyên, tháng 5 năm 2018 Tác giả Hoàng Thị Nga
  • 5. Viết đề tài giá sinh viên – ZALO:0973.287.149-TEAMLUANVAN.COM 1 Chương 1. N i suy Lagrange và n i suy Hermite Chương này được dành đe trình bày ve các bài toán n®i suy Lagrange, bài toán n®i suy Hermite và bài toán n®i suy Lagrange-Newton, tà định lí, h quả cho đen m®t so ví dụ tính toán cụ the. 1.1 Bài toán n i suy Lagrange Trong m®t so trường hợp, đe tính tőng hǎu hạn các phân thác, người ta thường sả dụng m®t so tính chat của đa thác, đ c bi t là công thác n®i suy Lagrange. Dưới đây là m®t so đong nhat thác cơ bản và áp dụng của chúng. Định lj 1.1 (Đong nhat thác Lagrange). Neu x1, x2, . . . , xm là m giá tr tuỳ ý, đôi m®t khác nhau và f(x) là đa thúc b¾c nhó thua m thì ta có đong nhat thúc sau f(x) = f(x1 ) (x − x2)(x − x3) . . . (x − xm) + (x1 − x2)(x1 − x3) . . . (x1 − xm) +f(x2) (x − x1)(x − x3) . . . (x − xm) (x2 − x1)(x2 − x3) . . . (x2 − xm) + · · · + f(xm ) (x − x1)(x − x2) . . . (x − xm−1) . (1.1) (xm − x1)(xm − x2) . . . (xm − xm−1) ChGng minh. Ta can cháng minh công thác f(x) − f(x1 ) (x − x2)(x − x3) . . . (x − xm) (x1 − x2)(x1 − x3) . . . (x1 − xm) −f(x2) (x − x1)(x − x3) . . . (x − xm) (x2 − x1)(x2 − x3) . . . (x2 − xm) — · · · − f(xm ) (x − x1)(x − x2) . . . (x − xm−1) (xm − x1)(xm − x2) . . . (xm − xm−1) ≡ 0. Nh n xét rang ve trái của công thác là m®t đa thác b c không quá m − 1 và có ít nhat m nghi m phân bi t là x1, x2, . . . , xm. V y đa thác trên phải đong nhat bang 0. H quả 1.1. Tà Định lý 1.1, ta thu được các đong nhat thác sau đây. −
  • 6. 2 Viết đề tài giá sinh viên – ZALO:0973.287.149-TEAMLUANVAN.COM 2 m 1 m 2 m m−1 (x − √ 3)(x − √ 5)(x − √ 7) (x − √ 2)(x − √ 5)(x − √ 7) ( √ 2 − √ 3)( √ 2 − √ 5)( √ 2 − √ 7) + ( √ 3 − √ 2)( √ 3 − √ 5)( √ 3 − √ 7) (x − √ 2)(x − √ 3)(x − √ 7) (x − √ 2)(x − √ 3)(x − √ 5) + ( √ 5 − √ 2)( √ 5 − √ 3)( √ 5 − √ 7) + ( √ 7 − √ 2)( √ 7 − √ 3)( √ 7 − √ 5) ≡ 1, a2 (x − b)(x − c) + b2 (x − c)(x − a) + c2 (x − a)(x − b) ≡ x (a < b < c). (a − b)(a − c) (b − c)(b − a) (c − a)(c − b) Định lj 1.2. Giả sả f (x) là m®t đa thác b c nhỏ thua ho c bang m − 2 (m > 2) và x1, x2, . . . , xm là m giá trị đôi m®t khác nhau cho trước tuỳ ý. Khi đó, ta có đong nhat thác f(x1) f(x2) + (x1 − x2)(x1 − x3) . . . (x1 − xm) (x2 − x1)(x2 − x3) . . . (x2 − xm) f (xm) + · · · + (x − x )(x − x ) . . . (x − x ) ≡ 0. ChGng minh. Nh n xét rang ve trái của đȁng thác đã cho chính là h so của hạng tả áng với b c m − 1 trong cách viet chính tac của đa thác f (x). Đong nhat các h so đong b c ta có ngay đieu phải cháng minh. Dưới đây, ta xét m®t so áng dụng trực tiep của đong nhat thác Lagrange. Ví dn 1.1. Tính tőng cos 1o cos 2o S = (cos 1o − cos 2o)(cos 1o − cos 3o) + (cos 2o − cos 1o)(cos 2o − cos 3o) cos 3o + (cos 3o − cos 1o)(cos 3o − cos 2o) L i giải. Áp dụng Định lý 1.2, với f(x) = x, x1 = cos 1o , x2 = cos 2o , x3 = cos 3o , ta thu được S = 0. Ví dn 1.2. Ta có các đong nhat thác b + c + d (b − a)(c − a)(d − a)(x − a) d + a + b c + d + a + + (c − b)(d − b)(a − b)(x − b) a + b + c + (d − c)(a − c)(b − c)(x − c) (a − d)(b − d)(c − d)(x − d) x − a − b − c − d . (x − a)(x − b)(x − c)(x − d) ≡
  • 7. 3 Viết đề tài giá sinh viên – ZALO:0973.287.149-TEAMLUANVAN.COM · · · L i giải. Th t v y, ta can cháng minh (a + b + c + d) − a + (a + b + c + d) − b + (a − b)(a − c)(a − d)(a − x) (b − a)(b − c)(b − d)(b − x) + (a + b + c + d) − c + (a + b + c + d) − d + (c − a)(c − b)(c − d)(c − x) (d − a)(d − b)(d − c)(d − x) + (a + b + c + d) − x (x − a)(x − b)(x − c)(x − d) Ta có, với đa thác b c nhat = 0. f(y) = a + b + c + d − y, y1 = a, y2 = b, y3 = c, y4 = d, y5 = x, theo Định lý 1.2 ta sě thu được ngay đieu phải cháng minh. Định lj 1.3. Cho x1, x2, . . . , xm là m giá trị tuỳ ý đôi m®t khác nhau. Đ t xn xn Sn = 1 + 2 (x1 − x2)(x1 − x3) . . . (x1 − xm) (x2 − x1)(x2 − x3) . . . (x2 − xm) xn Khi đó + + m . (xm − x1)(xm − x2) . . . (xm − xm−1) a) Sn = 0 neu 0 ≤ n < m − 1, b) Sm−1 = 1, c) Sm+k bang tőng các tích, moi tích có k + 1 thàa so (giong nhau ho c khác nhau) lay trong các so x1, x2, . . . , xm. ChGng minh. a) Theo Định lý 1.2, với f(x) = 1, x, x2 , . . . , xm−2 , ta được ngay S0 = S1 = . . . = Sm−2 = 0. b) Đe cháng minh Sm−1 = 1, ta chỉ can thay f (x) trong Định lý 1.2 bởi xm−1 , roi so sánh h so của hạng tả b c m − 1 ở hai ve của đong nhat thác vàa thu được. c) Đe tính Sn khi n > m − 1 ta làm như sau: Giả sả x1, x2, . . . , xm thoả mãn phương trình b c m αm + p1.αm−1 + p2.αm−2 + · · · + pm−1.α + pm = 0,
  • 8. 4 Viết đề tài giá sinh viên – ZALO:0973.287.149-TEAMLUANVAN.COM trong đó −p1 = x1 + x2 + · · · + xm p2 = x1x2 + x1x3 + · · · + xm−1xm . . . . . . . . . (−1)k .pk = x1x2x3 . . . xk + · · · Nhân cả hai ve của phương trình trên với αk , ta được αm+k + p1.αm+k−1 + p2αm+k−2 + · · · + pm−1.αk+1 + pm.αk = 0. Thay α trong đȁng thác này lan lượt bởi x1, x2, . . . , xm; và lan lượt chia đȁng thác thá nhat cho đȁng thác thá hai cho (x1 − x2)(x1 − x3) . . . (x1 − xm), (x2 − x1)(x2 − x3) . . . (x2 − xm) . . ., roi c®ng ve với ve các đȁng thác mới vàa nh n được, ta thu được Sm+k + p1.Sm+k−1 + · · · + pm−1.Sk+1 + pm.Sk = 0. (1.2) Đ t k = 0, ta thu được Sm + p1Sm−1 = 0. Do đó Sm = −p1 = x1 + x2 + · · · + xm. Nhờ đȁng thác (1.2) ta sě lan lượt tính tiep các bieu thác Sm+1, Sm+2, . . . Ta đ t lan lượt (x1 (x2 — x2 — x1 )(x1 )(x2 1 — x3 1 — x3 1 ) . . . (x1 ) . . . (x2 . — xm — xm ) = α1; ) = α2; Khi đó ta có (xm — x1)(xm — x2) . . . (xm — xm−1 ) = αm. Sn = xn α1 + xn α2 + · · · + xn αm. Xét 1 2 m α1 α2 αm P = + + · · · + . 1 − x1z 1 − x2z 1 − xmz Dùng công thác của cap so nhân với giả thiet rang z được chon sao cho |x1z| < 1, |x2z| < 1, . . . , |xmz| < 1,
  • 9. 5 Viết đề tài giá sinh viên – ZALO:0973.287.149-TEAMLUANVAN.COM m ta khai trien tőng P thành chuoi vô hạn như sau: P = α1(1 + x1z + x2 z2 + · · · ) + α2(1 + x2z + x2 z2 + · · · )+ hay 1 2 + · · · + αm(1 + xm.z + x2 z2 + · · · ) P = (α1 + α2 + · · · + αm) + (x1α1 + x2α2 + · · · + xmαm)z + +(x2 α1 + x2 α2 + · · · + x2 αm)z2 + · · · tác là 1 2 m Đe cho gon, ta đ t P = S0 + S1z + S2z2 + S3z3 + · · · . (1 − x1z)(1 − x2z) . . . (1 − xmz) = Q. Khai trien Q theo luy thàa của z, ta có the viet Q = 1 − δ1z + δ2z2 + · · · + (−1)m δmzm , trong đó δ1 = x1 + x2 + · · · + xm, δ2 = x1x2 + x1x3 + · · · + xm−1xm.. . . . . . . . . . Tiep theo, nhân cả hai ve của đȁng thác thá hai với (1 − x1z)(1 − x2z) . . . (1 − xm.z), ta có PQ = α1(1 − x2z)(1 − x3z) . . . (1 − xmz)+ α2(1 − x1z)(1 − x3z) . . . (1 − xm.z)+ α3(1 − x1z)(1 − x2z)(1 − x4z) . . . (1 − xmz) + · · · + αm(1 − x1z)(1 − x2z) . . . (1 − xm−1z). Như v y PQ là m®t đa thác b c m − 1 đoi với z. Ta sě cháng minh rang nó chính là zm−1 , tác là có đong nhat thác PQ = zm−1 .
  • 10. 6 Viết đề tài giá sinh viên – ZALO:0973.287.149-TEAMLUANVAN.COM 1 x1 x1 x1 xm−1 xm−1 xm−1 Th t v y, bieu thác PQ − zm−1 tri t tiêu khi vì, chȁng hạn, với 1 z = , x1 1 1 , . . . , x2 xm 1 z = x1 thì α 1 − x2 1 − x3 · · · 1 − xm − 1 1 1 = − = 0. V y nên PQ − zm−1 = 0. Do đó hay zm−1 = P Q zm−1 1 1 − δ1z + δ2z2 − . . . + (−1)mδmzm = S0 + S1 z + · · · + Sm−1zm−1 + · · · Neu khai trien ve trái thành chuoi vô hạn theo luy thàa của z thì chuoi này bat đau bang hạng tả cháa zm−1 . Vì v y, h so của các hạng tả b c 0, 1, 2, . . . , m − 2 trong ve phải bang không, tác là S0 = S1 = . . . = Sm−2 = 0. Ngoài ra, h so của hạng tả áng với b c m = 1 ở ve trái bang 1. V y Sm−1 = 1. Bây giờ đȁng thác can cháng minh có dạng sau: 1 − δ1 .z + δ2 zm−1 .z2 − · · · (−1)mδm .zm = zm−1 + Sm.zm + Sm+1.zm+1 + · · · Uớc lượng cả hai ve cho zm−1 , ta thu được ho c 1 1 − δ1z + δ2z2 − · · · + (−1)mδmzm = 1 + Sm z + Sm+1z2 + · · · 1 = (1 − δ1z + δ2z2 − · · · + (−1)m δm.zm )(1 + Smz + Sm+1z2 + · · · ) 1 1 1
  • 11. 7 Viết đề tài giá sinh viên – ZALO:0973.287.149-TEAMLUANVAN.COM − − − 1 2 m 1 2 m 1 2 m p=0 q=0 s=0 Khai trien ve phải theo lũy thàa của z và so sánh các h so ở hai ve, ta được Sm − δ1 = 0, δ2 − δ1.Sm + Sm+1 = 0, . . . . . . . . . Như v y, ta có the tính được Sm, Sm+1, Sm+2, . . . Nham thiet l p được m nh đe mở r®ng cau trúc của Sm+k, ta xét 1 1 = Q 1 x1.z ∞ 1 . 1 x2.z ∞ .. . 1 1 xm.z ∞ = Σ xp .zp . Σ xq .zq · · · Σ xs zs = Σ xp xq . . . xs zp+q+···+s . M t khác 1 = 1 + Sm Q z + Sm+1z2 + · · · + Sm+k zk+1 + · · · , nên ta được Sm+k = Σ xp xq . . . xs Vì v y, ta thu được ket quả cuoi cùng Sm+k bang tőng các tích, moi tích có k + 1 thàa so (giong nhau ho c khác nhau) lay trong các so x1, x2, . . . , xm. Nói riêng Sm+1 = x Sm+2 = x −1xm −1xm +x1x2x3 + · · · + xm−2xm−1xm, (đieu phải cháng minh). H quả 1.2. Giả sả ak bk ck Khi đó Sk = + + . (a − b)(a − c) (b − a)(b − c) (c − a)(c − b) S0 = S1 = 0 , S2 = 1 , S3 = a + b + c, S4 = a2 + b2 + c2 + ab + bc + ca, S5 = a3 + b3 + c3 + a2 b + b2 c + c2 a + ab2 + bc2 + ca2 + abc. p+q+···+s=k+1 2 + x2 + · · · + x2 + x1x2 + x1x3 + · · · + xm 1 2 m 3 + x3 + · · · + x3 + x2 x2 + x2 x3 + · · · + x2 1 2 m 1 1 m
  • 12. 8 Viết đề tài giá sinh viên – ZALO:0973.287.149-TEAMLUANVAN.COM N Σ j=1 j=1 H quả 1.3. Giả sả ak bk ck Tk = + + + (a − b)(a − c)(a − d) (b − a)(b − c)(b − d) (c − a)(c − b)(c − d) dk Khi đó + . (d − a)(d − b)(d − c) T0 = T1 = T2 = 0 , T3 = 1 , T4 = a + b + c + d. Bây giờ ta chuyen sang khảo sát bài toán n®i suy Lagrange dưới ngôn ngǎ tőng quát. Bài toán 1.1 (Bài toán n®i suy Lagrange). Cho xi, ai ∈ R, với xi /= xj ∀i j, (i, j = 1, 2, . . . , N). Hãy xác định đa thác L(x) có b c deg L(x) ≤ N − 1 thỏa mãn đieu ki n L(xi) = ai, ∀i = 1, 2, . . . , N. (1.3) L i giải. Đe đơn giản, ký hi u Li(x) = Khi đó, de thay rang j= Y 1, j= / x − xj , (i = 1, 2, . . . , N). i xi − xj hay Li(xj) = δij. Li(xj ) = 1 khi i = j 0 khi i /= j Tiep theo, ta cháng minh rang đa thác N L(x) = aiLi(x) (1.4) i=1 là đa thác duy nhat thỏa mãn đieu ki n của bài toán n®i suy Lagrange (1.3), và ta goi đa thác này là đa thúc n®i suy Lagrange. Th t v y, de thay rang Ngoài ra, ta có deg L(x) ≤ N − 1. N N L(xi) = Σ ajLj(xi) = Σ ajδij
  • 13. 9 Viết đề tài giá sinh viên – ZALO:0973.287.149-TEAMLUANVAN.COM Y Y hay L(xi) = ai, ∀i = 1, 2, . . . , N. Cuoi cùng, neu có đa thác L∗(x), có b c deg L∗(x) với deg L∗(x) ≤ N − 1 cũng thỏa mãn đieu ki n của bài toán (1.3) thì khi đó, đa thác P(x) = L(x) − L∗(x) cũng có b c deg P(x) ≤ N − 1 và thỏa mãn P(xi) = 0, ∀i = 1, 2, . . . , N. Tác là P (x) là đa thác có b c deg P (x) với deg P (x) ≤ N − 1 mà lại có ít nhat N nghi m phân bi t x1, x2, . . . , xN , nên P(x) ≡ 0, và do đó L(x) = L∗(x). Tà bài toán n®i suy Lagrange ta có nh n xét sau: Nh n xét 1.1. Ve m t hình hoc, vi c xây dựng đa thác n®i suy Lagrange (1.3) có nghĩa là xây dựng m®t đa thác m®t bien b c không quá N − 1 đi qua tat cả các điem Mi(xi, yi), ∀i = 1, 2, . . . , N cho trước. 1.2 Bài toán n i suy Hermite Bài toán n®i suy Newton là m®t mở r®ng tự nhiên của đong nhat thác Taylor và tương áng, khai trien Taylor - Gontcharov là mở r®ng khai trien Taylor cő đien. Bây giờ ta chuyen sang xét bài toán n®i suy Hermite là m®t mở r®ng tự nhiên của bài toán n®i suy Lagrange và Taylor. Với đa thác n L(x) = (x − xj), xi j=1 xj khi i j, i, j = 1, 2, . . . , n thì đieu ki n Lagrange L(xj) = 0, j = 1, 2, . . . , n là đieu ki n tự nhiên đe xác định đa thác đơn P (x) ( đa thác có các nghi m đơn). Khi P (x) có nghi m b®i thì đieu ki n Lagrange không đủ đe xác định P (x). Vì the, ta can các đieu ki n tőng quát hơn đe đảm bảo ton tại duy nhat m®t đa thác dạng n H(x) = (x − xj)αj , xi j=1 xj khi i j, i, j = 1, 2, . . . , n.
  • 14. 10 Viết đề tài giá sinh viên – ZALO:0973.287.149-TEAMLUANVAN.COM Y n Σ Σ Σ n p −1 j l i (x − xi)pi i W(x) i (x − xi)pi−l i i (x − xj)pj−l Rõ ràng đa thác H(x) có deg H(x) = α1 + α2 + · · · + αn và H(k) (xj) = 0, k = 0, 1, . . . , αj − 1; j = 1, 2, . . . , n. Vì the, ta có the phát bieu bài toán n®i suy Hermite dưới dạng sau. Bài toán 1.2 (N®i suy Hermite). Cho xi, aki ∈ R, với i = 1, 2, . . . , n; k = 0, 1, . . . , pi − 1 và xi /= xj ∀i j, trong đó p1 + p2 + · · · + pn = N. Hãy xác định đa thác H(x) có b c deg H(x) ≤ N − 1 thỏa mãn đieu ki n H(k) (xi) = aki, ∀i = 1, 2, . . . , n; ∀k = 0, 1, . . . , pi − 1. (1.5) L i giải. Ký hi u n W(x) = (x − xj)pj j=1 W (x) W (x) = = Y (x − x )pj . Tiep theo, giả sả H(x) là đa thác có b c deg H(x) với deg H(x) ≤ N − 1 và thỏa mãn đieu ki n bài toán. Ta can xác định các h so αli ∈ R sao cho đȁng thác sau được thoả mãn n pi−1 H(x) W (x) = αli . (x − xi)pi−l Ta có i=1 l=0 n pi−1 H(x) = (x − x )pi H(x) = (x − x )pi Σ Σ αli pi−1 = α (x − x )l + (x − x )pi Σ Σ αlj . Trong các phép bien đői tiep theo, đe ý rang l p l p p (fg)(l) = Σ Ck f (k) g(l−k) và Σ Σ Aki = Σ Σ Aki, ta thu được k=0 l=0 k=0 k=0 l=k h H(x) i(l) = l!αli. Wi(x) (x=xi) l=0 i j=1, j l=0 l=0 i=1 i j=1, j j Wi(x) li
  • 15. 11 Viết đề tài giá sinh viên – ZALO:0973.287.149-TEAMLUANVAN.COM Σ Σ n ki ki Σ Σ Σ i Σ Suy ra αli = Σ aki h 1 i(l−k) . Do đó k=0 k!(l − k)! Wi(x) (x=xi) H(x) = pi−1 l aki h 1 i(l−k) 1 W (x) i=1 l=0 k=0 k!(l − k)! Wi(x) (x=xi) (x − xi)pi−l Σ p Σ i−1 p Σ i−1 a h 1 i(l−k) 1 Suy ra i=1 k=0 l=k k!(l − k)! Wi(x) (x=xi) (x − xi)pi−l H(x) = W (x) Σ p Σ i−1 p Σ i−1 a h 1 i(l−k) 1 hay i=1 k=0 l=k k!(l − k)! Wi(x) (x=xi) (x − xi)pi−l H(x) = Σ pi−1 aki (x − xi)k Wi(x) p Σ i−1 h 1 i(l−k) (x − xi)l−k i=1 k! k=0 l=k Wi(x) (x=xi) (l − k)! Đői chỉ so ở tőng cuoi cùng của đȁng thác trên, ta thu được H(x) = Σ pi−1 aki (x − xi)k Wi(x) pi Σ −1−k h 1 i(l) (x − xi)l hay i=1 k! k=0 l=0 Wi(x) (x=xi) l! H(x) = Σ pi−1 aki (x − xi)k Wi(x) T , 1 ,(pi−1−k) , trong đó i=1 k! k=0 Wi(x) (x=xi) T , 1 ,(pi−1−k) pi Σ −1−k h 1 i(l) (x − x )l Wi(x) (x=xi) l=0 Wi(x) (x=xi) l! 1 là đoạn khai trien Taylor đen cap thá (pi − 1 − k) tại x = xi của hàm so Ký hi u . Wi(x) Hki(x) = (x − xi)k Wi(x) T , 1 ,(pi−1−k) . k! Khi đó, de thay rang Wi(x) (x=xi) deg Hki(x) ≤ k + (N − pi) + (pi − 1 − k) = N − 1, = l n n n n n = . .
  • 16. 12 Viết đề tài giá sinh viên – ZALO:0973.287.149-TEAMLUANVAN.COM ki ki Σ Σ Σ lj Σ H(l) (xj hay H(l) (xj) = δklδij. ) = 1 neu k = l và i = j 0 neu k /= l ho c i /= j Bây giờ, ta sě cháng minh rang pi−1 akiHki(x) = Σ pi−1 aki (x − xi)k Wi(x) T , 1 ,(pi−1−k) (1.6) hay i=1 k=0 i=1 k! k=0 wi(x) (x=xi) H(x) = Σ pi−1 aki (x − xi)k Wi(x) T , 1 ,(pi−1−k) i=1 k! k=0 wi(x) (x=xi) là đa thác duy nhat thỏa mãn đieu ki n của bài toán n®i suy Hermite 1.5 và ta goi đa thác (1.6) là đa thúc n®i suy Hermite. Th t v y, de thay rang deg H(x) ≤ N − 1 và n pj −1 n pj −1 H(k) (xi) = Σ Σ aljH(k) (xi) = Σ Σ aljδklδij. Suy ra j=1 l=0 j=1 l=0 H(k) (xi) = aki, (∀i = 1, 2, . . . , n; ∀k = 0, 1, . . . , pi − 1). Cuoi cùng, ta cháng minh tính duy nhat nghi m của bài toán n®i suy Hermite. Giả sả ton tại đa thác H∗(x), có b c deg H∗(x) với deg H∗(x) ≤ N − 1, cũng thỏa mãn đieu ki n của bài toán n®i suy Hermite. Khi đó, đa thác P (x) = H(x) − H∗(x) cũng có b c deg P(x) ≤ N − 1 và thỏa mãn đieu ki n P(k) (xi) = 0, (∀i = 1, 2, . . . , n; ∀k = 0, 1, . . . , pi − 1). Khi đó, theo như cách xây dựng đa thác H(x) ở phan trên, áng với trường hợp aki = 0, ∀i = 1, 2, . . . , n; ∀k = 0, 1, . . . , pi − 1, ta suy ra P(x) ≡ 0 và do đó H(x) = H∗(x). Chú ý rang đa thác n®i suy Taylor và đa thác n®i suy Lagrange là nhǎng trường hợp riêng của đa thác n®i suy Hermite. Th t v y, đa thác n®i suy Taylor là trường hợp riêng của đa thác n®i suy Hermite vì trong đa thác n®i suy Hermite (1.5) áng với n = 1 thì ta có p1 = N, W(x) = (x − x1)p1 , W1(x) ≡ 1. n n n
  • 17. 13 Viết đề tài giá sinh viên – ZALO:0973.287.149-TEAMLUANVAN.COM Σ N Σ Σ i . 1 n Wi(x) a0i T (x=x ) + a1i(x − xi) T Suy ra V y nên Hk1(x) = N−1 (x − x1)k k! k H(x) = ak1 k=0 (x − x1) k! ≡ T(x). Tương tự, đa thác n®i suy Lagrange là trường hợp riêng của đa thác n®i suy Hermite vì trong đa thác n®i suy Hermite (1.5), áng với k = 0, ta có pi = 1 ∀i = 1, 2, . . . , n, và n = N. Suy ra V y nên H0i (x) = Wi(x) = Wi(xi) N j= Y 1, j= / x − xj i xi − xj = Li(x). H(x) = a0iLi(x) ≡ L(x). i=1 Tiep theo, ta xét m®t so trường hợp riêng đơn giản của đa thác n®i suy Hermite. Xét đa thác n®i suy Hermite (1.5) H(x) = Σ pi−1 aki (x − xi)k Wi(x) T , 1 ,(pi−1−k) , trong đó i=1 k! k=0 Wi(x) (x=xi) T , 1 ,(pi−1−k) pi Σ −1−k h 1 i(l) (x − x )l Wi(x) (x=xi) l=0 Wi(x) (x=xi) l! Ta xét m®t trường hợp khi h đieu ki n chỉ cháa đạo hàm b c nhat. Trong đa thác n®i suy Hermite (1.5), neu pi = 2, ∀i = 1, 2, . . . , n thì khi đó k = 0, 1 và ta có H(x) = Σ Σ aki (x − xi)k Wi(x) T , 1 ,(1−k) . Suy ra i=1 k! k=0 Wi(x) (x=xi) Σ h , 1 ,(1) , 1 ,(0) i (x=xi) Wi(x) i Wi(x) i=1 = . n n H(x) =
  • 18. 14 Viết đề tài giá sinh viên – ZALO:0973.287.149-TEAMLUANVAN.COM n i n i i 0 0 0 0 0 0 0 Y i i i hay H(x) = Σ Wi(x) h a0i 1 − Wi ′ (xi) (x − xi) + a1i(x − xi) i = i=1 Wi(xi) Wi(xi) = Σ Wi(x) h a0i − Wi a0i ′ (xi) — a1i (x − xi) i . i=1 Wi(xi) Wi(xi) Cuoi cùng, đe ý rang ta có các đong nhat thác n 2 Wi(x) Wi(xi) = j=1, j= / (x − xj) (xi − xj)2 = L2 (x), Wi ′ (xi) = 2 h Y x − xj i′ = 2L ′ (xi). W (x ) x − x x=x i j=1, j/=i V y, ta đã thu được đa thác n®i suy Hermite trong trường hợp này như sau H(x) = Σ i=1 L2 (x) h a0i − 2a0iL ′ (xi) − a1i (x − xi) i . Tiep theo ta xét trường hợp khi h đieu ki n cháa các giá trị đạo hàm các cap tại các nút n®i suy đeu bang 0 trà ra m®t điem. Trong đa thác n®i suy Hermite (1.5), ta xét trường hợp đ c bi t, chȁng hạn ∃k0, i0 sao cho ak0i0 = a = / 0 v à aki = 0, ∀(k, i) Khi đó, ta có (k0, i0). (x − xi )k0 , 1 ,(pi0 −1−k0) Neu k0 = p0 − 1 và chon ak0i0 = (pi0 − 1)!Wi0(xi0) thì ta có W (x) V y, trong trường hợp đ c bi t H(x) = . x − xi0 H(k) (xi ) = 0, ∀k = 0, 1, . . . , pi − 2; H(pi0 −1) (xi ) = (pi − 1)!Wi (xi ); H(k) (xi) = 0, ∀k = 0, 1, . . . , pi − 1, ∀i = 1, 2, . . . , n, i = / i0 thì đa thác n®i suy Hermite có dạng W (x) H(x) = x − xi0 = (x−x1 )p1 · · · (x−x i0−1 )pi0 (x−xi )pi0−1 (x−x i0+1 )pi0 . . . (x−xn )pn . (x=xi0) Wi0(x) k0! j i n n H(x) = ak0i0 Wi0(x) T . i 0
  • 19. 15 Viết đề tài giá sinh viên – ZALO:0973.287.149-TEAMLUANVAN.COM Y 0 0 0 0 Ngược lại, moi đa thác có dạng H(x) = (x − x1)α1 (x − x2)α2 · · · (x − xn)αn , xi đeu là nghi m của các bài toán Hermite xj ∀i = / j, i, j = 1, 2, . . . , n H(k) (xi) = 0, ∀k = 0, 1, . . . , αi − 1, i = 1, 2, . . . , n H(αi0 ) (xi ) = αi !Wi (xi ). Chȁng hạn, neu ta viet α α α (x − x1)α1+1 (x − x2)α2 ...(x − xn)αn H(x) = (x − x1) 1 (x − x2) 2 ...(x − xn) n = x − x1 thì H(x) là nghi m của bài toán Hermite sau đây H(k) (x1) = 0, ∀k = 0, 1, . . . , α1 − 1, H(α1) (x1) = α1!W1(x1) H(k) (xi) = 0, ∀k = 0, 1, . . . , αi − 1, ∀i = 1, 2, . . . , n, i 1. Tiep theo, trong phan này ta sě nêu m®t so ví dụ áp dụng các ky thu t cơ bản đe xác định các đa thác khi biet m®t so đ c trưng của chúng dưới dạng nút n®i suy. Bài toán 1.3. Cho 0 < α < 1. Xác định tat cả các đa thác f (x) b c n (n ≥ 2) sao cho ton tại dãy so r1, r2, . . . , rn (r1 < r2 < . . . < rn) thoả mãn các đieu ki n sau f(ri) = 0, f′(αri + (1 − α)ri+1) = 0 (i = 1, 2, . . . , n). L i giải. Nh n xét rang với a < b và x = αa +(1 − α)b ; α ∈ (0, 1) thì x ∈ (a, b). Khi đó 1 p = x − a 1 + x − b 1 = 2α − 1 . α(1 − α)(b − a) 1 Do v y p > 0 khi và chỉ khi α > 1 , p < 0 khi và chỉ khi α < 2 2 và p = 0 khi và chỉ khi α = nên . Theo giả thiet thì 2 n f(x) = c (x − ri) i=1 f′(x) f(x) = i=1 1 . x − ri n Σ
  • 20. 16 Viết đề tài giá sinh viên – ZALO:0973.287.149-TEAMLUANVAN.COM Σ (−1) C P(k) = 0. Σ Σ n−1 P (x) = Σ P (k) (x − 0) · · · (x − (k − 1))(x − (k + 1)) · · · (x − (n − 1)) . (−1)n Cn P(n) = Σ (−1)k Ck P(k). (k − 0) · · · (k − (k − 1))(k − (k + 1)) · · · (k − (n − 1)) k=0 Với n ≥ 3 và 0 < α ≤ 1 2 ta đ t x = αr1 + (1 − α)r2. Khi đó theo giả thiet thì f′(x) = 0 và đong thời f(x) = 0. M t khác mâu thuan. f′(x) f(x) 1 = + x − r1 1 1 x − r2 + i=3 1 x − ri < 0, Tương tự với n ≥ 3 và 1 < α < 1 ta cũng nh n được đieu vô lý. 2 Neu n = 2 và α / = , thì tương tự như trên cũng dan đen đieu mâu thuan. Do 2 1 v y chỉ can xét trường hợp n = 2 và α = . Khi đó moi tam thác b c hai có 2 2 nghi m phân bi t đeu thoả mãn bài toán đã cho. Bài toán 1.4. Xác định tat cả các đa thác P(x) b c nhỏ thua n và thoả mãn đieu ki n n k k n k=0 L i giải. Áp dụng công thác n®i suy Lagrange với nút n®i suy xk = k, ta được moi đa thác P(x) b c nhỏ thua n đeu có dạng n−1 nên P(x) = P(xk k=0 n−1 ) (x − x0) · · · (x − xk−1)(x − xk+1) · · · (x − xn−1) (xk − x0) · · · (xk − xk−1)(xk − xk+1) · · · (xk − xn−1) Ta có k=0 (k − 0) · · · (k − (k − 1))(k − (k + 1)) · · · (k − (n − 1)) n−1 P (n) = P(k) (n − 0) · · · (n − k + 1)(n − k − 1) · · · 1 . k!(−1)n−k(n − k − 1)! Suy ra k=0 n−1 n V y đieu ki n bài toán được thỏa mãn. n k=0 Tóm lại, các đa thác can tìm có dạng P (x) = Σ P (k) (x − 0) · · · (x − (k − 1))(x − (k + 1)) · · · (x − (n − 1)) , n Σ
  • 21. 17 Viết đề tài giá sinh viên – ZALO:0973.287.149-TEAMLUANVAN.COM − P . ≡ 2q q p p − n 2 2 2 2q 2q 2q 2q 2q q n n 2 1000 10 10 L i giải. Xét đa thác trong đó P(0), P(1), . . . , P(n − 1) là các giá trị tùy ý. Tiep theo ta xét m®t so bài toán liên quan đen n®i suy theo xap xỉ Diophane. Bài toán 1.5. Cháng minh rang ton tại đa thác Pn(x) b c n (n ≥ 1) với h so nguyên sao cho . (x) − . < ∀x ∈ h , i . 1 1 1 9 P (x) = 1 [(2x 1)n + 1]. n 2 Ta thay ngay rang các h so của P (x) là các so nguyên. Khi đó với x ∈ h 1 , 9 i ta có n . (x) − 1 . = 1 |2x − 1|n ≤ 1 (0, 8)n ∀n ∈ N∗. 10 10 Chon n đe (0, 8)n < 0, 002. Chȁng hạn n > 28 thì ta có ngay Pn(x) chính là đa thác can tìm. Bài toán 1.6. Cho hai so nguyên dương p, q. Cháng minh rang ton tại đa thác Pn(x) b c n với h so nguyên sao cho P p 1 (x) − . < n với moi x thu®c khoảng 1 , 3 . q q2 L i giải. Với q = 1 thì ta chon P(x) p. Với q > 1 thì ta thay khoảng I = 1 , 3 có đ® dài bang 1/q < 1. Chon m ∈ N sao cho 3 m < 1 . Chon n đủ lớn đe và đ t an < 1 pq với a = 1 − 1 m P (x) = p [1 − (1 − qxm )n ]. Khi đó rõ ràng Pn(x) là đa thác với h so nguyên và với x ∈ I thì . (x) − . = . (1 − qxm )n . < p an < 1 n q q và q q2 P (x) = 1 [(2x 1)n + 1]. n 2 Ta thay ngay rang các h so của Pn(x) là các so nguyên. . . P P
  • 22. 18 Viết đề tài giá sinh viên – ZALO:0973.287.149-TEAMLUANVAN.COM Bài toán 1.7. Cháng minh rang không ton tại đa thác f(x) ∈ Z[x] mà f(2005) = 2005 và f(2007) = 2008. L i giải. Giả sả ton tại đa thác f(x) = anxn + an−1xn−1 + · · · + a0, ai ∈ Z ∀i ∈ {0, 1, . . . , n} thỏa mãn đieu ki n của bài toán. Khi đó, ta có f(2007) − f(2005) = an(2007n − 2005n ) + an−1(2007n−1 − 2005n−1 ) + · · · + a1(2007 − 2005) chia het cho 2. M t khác f(2007) − f(2005) = 2008 − 2005 = 3 và không chia het cho 2. V y không ton tại đa thác f(x) thỏa mãn đieu ki n đe bài. 1.3 Bài toán n i suy Lagrange - Newton Trước het ta xét bài toán n®i suy hon hợp Lagrange - Newton. Bài toán 1.8 (N®i suy Lagrange - Newton). Cho xki, aki ∈ R, với xki /= j; k = 0, 1, . . . , n − 1; i, j = 1, . . . , rk+1; trong đó r0 = 0, r0 + r1 + · · · + rk = sk, r0 + r1 + · · · + rn = sn = N. xkj ∀i Hãy xác định đa thác f(x) có b c deg f(x) ≤ N − 1 và thỏa mãn đieu ki n f(sk) (xki) = aki, ∀k = 0, 1, . . . , n − 1, ∀i = 1, . . . , rk+1. (1.7) L i giải. Ký hi u rk+1 Lki(x) = j= Y 1, j/=i x − xkj xki − xkj , (k = 0, 1, . . . , n − 1) và phép lay nguyên hàm . Khi đó, de thay rang deg Lki(x) ≤ rk+1 − 1, Lki(xkj ) = 1 neu i = j hay Lki(xkj) = δij. 0 neu i j
  • 23. 19 Viết đề tài giá sinh viên – ZALO:0973.287.149-TEAMLUANVAN.COM k+1 Σ Σ Σ Σ n−1 n−1 n−1 Tiep theo, ta đ t yn(x) ≡ 0 và xây dựng dãy hàm rk+1 yk(x) = Rsk [aki − y(sk) (xki)]Lki(x), (k = n − 1, . . . , 1, 0). De thay rang i=1 deg yk(x) ≤ N − 1 và deg yk(x) < sk+1. Bây giờ, ta sě cháng minh rang đa thác f(x) = y0(x) + y1(x) + · · · + yn−1(x) chính là nghi m duy nhat của bài toán n®i suy Lagrange - Newton (1.7). Th t v y, de thay rang deg f(x) ≤ N − 1. Ngoài ra, ta cũng có f(sn−1) (x(n−1)i) = (y0 + · · · + yn−2 + yn−1)(sn−1) (x(n−1)i) = y(sn−1) (x n−1 rn (n−1)i) = a(n−1)j.L(n−1)j(x(n−1)i) j=1 rn = a(n−1)j.δij = a(n−1)i j=1 f(sn−2) (x(n−2)i) = (y0 + · · · + yn−3 + yn−2 + yn−1)(sn−2) (x(n−2)i) = y (sn−2) (x(n−2)i) + y (sn−2) (x(n−2)i), trong đó y(sn−2) (x n−2 (n−2)i rn−1 n−2 rn−1 ) = [a j=1 (n−2)j n−1 — y(sn−2) (x(n−2)j )].L(n−2)j (x(n−2)i) = Σ [a(n−2)j − y (sn−2) (x(n−2)j)].δij V y nên j=1 = a(n−2)i − y (sn−2) (x(n−2)i). f (sn−2) (x(n−2)i) = a(n−2)i. Bang cách tương tự, ta cháng minh được rang f(sk) (xki) = aki, ∀k = 0, 1, . . . , n − 1, ∀i = 1, . . . , rk+1.
  • 24. 20 Viết đề tài giá sinh viên – ZALO:0973.287.149-TEAMLUANVAN.COM Cuoi cùng, neu có đa thác f∗(x) với b c deg f∗(x) ≤ N − 1 và thỏa mãn đieu ki n của bài toán (1.7), thì khi đó đa thác P(x) = f(x) − f∗(x) cũng có b c deg P(x) ≤ N − 1 và thỏa mãn đieu ki n P(sk) (xki) = 0, ∀k = 0, 1, . . . , n − 1, ∀i = 1, . . . , rk+1. Khi đó, theo như cách xây dựng đa thác f(x) ở phan trên, áng với trường hợp aki = 0, ∀k = 0, 1, . . . , n − 1, ∀i = 1, . . . , rk+1, ta có yk = 0, ∀k = 0, 1, . . . , n − 1. Suy ra P(x) ≡ 0, và do đó f(x) = f∗(x). Tà bài toán trên ta có nh n xét sau: Nh n xét 1.2. Neu k = 0, tương áng r2 = r3 = · · · = rn = 0, thì sk = 0. Khi đó bài toán n®i suy Lagrange - Newton chính là bài toán n®i suy Lagrange quen biet. Neu i = 1, tương áng r1 = r2 = · · · = rn = 1, thì sk = k. Khi đó bài toán n®i suy Lagrange - Newton chính là bài toán n®i suy Newton đã biet.
  • 25. Viết đề tài giá sinh viên – ZALO:0973.287.149-TEAMLUANVAN.COM Σ 0 p + 1 ( — m 1 ) 21 Chương 2. Ứng dnng n i suy tính nguyên hàm và tích phân các hàm phân thfíc Chương này sě giới thi u cho chúng ta m®t so thu t toán tìm nguyên hàm của m®t hàm so hǎu tỉ cho trước theo thá tự tà đơn giản đen phác tạp. 2.1 Nguyên hàm của hàm phân thfíc v i các cfic điem đơn Lớp hàm quen thu®c có nguyên hàm sơ cap là lớp các hàm so hǎu tỉ. Ket lu n này được Laplace cháng minh lan đau tiên vào năm 1812 bang cách phân tích m®t hàm so hǎu tỉ ra thành tőng của các phân thác đơn giản. Định lj 2.1 (Laplace, 1812). Nguyên hàm của m®t hàm so hǎu tỉ là m®t hàm sơ cap (đó là hàm so hǎu tỉ, ho c là tőng của m®t hàm so hǎu tỉ và m®t so hǎu hạn của các logarit của nhǎng hàm so hǎu tỉ). ChGng minh. Nh n xét rang moi đa thác Q(x) = b0xn + b1xn−1 + · · · + bn đeu có the được bieu dien được như sau Q(x) = b0(x − a1)m1 (x − a2)m2 . . . (x − ar)mr , trong đó m1, . . . , mr là nhǎng so nguyên dương có tőng bang n và a1, . . . , ar là nhǎng so thực ho c so phác. Do đó moi hàm so hǎu tỉ R(x) có mau là đa thác Q(x) đeu có the bieu dien dưới dạng A0xp + A1xp−1 + · · · + Ap + Σ ( βs,1 + βs,2 + · · · + βs,ms ) . Tà đó, suy ra s=1 x − as (x − as) (x − as)ms ∫ R (x) dx = A xp+1 + A xp + · · · + A x + C r + s=1 βs,2 βs,1 ln (x − as) − x − a — · · · βs,ms . (ms − 1) (x − as) s− 2 r s 1 p p
  • 26. 22 Viết đề tài giá sinh viên – ZALO:0973.287.149-TEAMLUANVAN.COM m ∫ n−1 m 2 = sin x 1 − sin x cos xdx ∫ m n 2 2 Q = − 4 + 2 ln .x − 1. + C 2 là hàm so sơ cap. Xét các dạng toán minh hoa sau. 2x + 2 Bài toán 2.1. Ta có f (x) = là m®t hàm so hǎu tỉ. Nguyên hàm x3 − 3x + 2 của nó có dạng là tőng của m®t hàm so hǎu tỉ và các logarit của nhǎng hàm so hǎu tỉ.Th t v y, ta có ∫ 2x + 2 dx = ∫ 2x + 2 dx x3 − 3x + 2 (x − 1)2 (x + 2) = ∫ 4 dx + ∫ 2 dx − ∫ 2 dx 3(x − 1)2 9 (x − 1) 9 (x + 2) 3 (x − 1) trong đó C là hang so. 9 .x + 2. Nh n xét 2.3. Neu m và n là các so nguyên, thì ta có các ket lu n sau. i) Tích phân ∫ (1 − xn ) 1 dx là hàm so sơ cap neu và chỉ neu m = ±1 ho c n = ±1 ho c m = −n. ii) Tích phân sin xcos xdx là hàm so sơ cap với moi giá trị của m và n. Th t v y, vì m, n là các so nguyên nên ta có ∫ sinm x.cosn xdx = ∫ sinm x cos2 x n−1 cos xdx = ∫ um 1 − u2 n−1 du, (u = sin x) . Tích phân trên là hàm so sơ cap neu và chỉ neu m lẻ ho c n lẻ ho c cả hai n, m chȁn. Định lý Laplace 2.1 cho ta ket lu n rang moi hàm hǎu tỉ đeu có nguyên hàm sơ cap. Tuy nhiên, vi c tìm các nguyên hàm này không phải là m®t công vi c đơn giản. Sau đây là m®t so thu t toán giải bài toán trên. Trong mục này, ta xét m®t thu t toán tìm nguyên hàm của hàm so hǎu tỉ có P (x) dạng f(x) = n i=1 (x − xi) trong đó P (x) là m®t đa thác, xi xj với moi i j. Tà công thác trên, ta có m®t phương pháp tìm nguyên hàm của hàm so hǎu tỉ mà mau thác b c n có n nghi m đơn phân bi t.
  • 27. 23 Viết đề tài giá sinh viên – ZALO:0973.287.149-TEAMLUANVAN.COM Q − i=1 Đe tìm nguyên hàm của hàm so hǎu tỉ có dạng P (x) f(x) = n , (x xi) i=1 trong đó P(x) là m®t đa thác và xi tự như sau. xj với moi i /= j. Ta tien hành theo trình Bước 1. Áp dụng công thác n®i suy Lagrange ta phân tích P(x) thành dạng Σ n Q x − xi tőng aj j=1 i=1,i/=j x j — xi với aj = P(xj), với ∀j = 1, 2, . . . , n. Q n Sau đây, xét m®t so ví dụ minh hoa. Bài toán 2.2. Tìm nguyên hàm của hàm so f (x) = 2x2 . (x + 1) (x − 1) (x − 2) Lài giai. Đ t P (x) = 2x2 và xét ba nút n®i suy là x1 = −1; x2 = 1; x3 = 2. Ta có a1 = 2, a2 = 2, a3 = 8. Áp dụng công thác n®i suy Lagrange, ta có f (x) = −1 − 1 + 8 . 3 (x + 1) x − 1 3 (x − 2) Tà đây, ta suy ra nguyên hàm của hàm so can tìm là ∫ f (x) dx = −1 ln |x + 1| − ln |x − 1| + 8 ln |x − 2| + C, 3 trong đó C là hang so. Bài toán 2.3. Tìm nguyên hàm của hàm so f (x) = 3 x2 + 3x + 8 . (x + 1) (x − 1) (x − 2) (x + 3) Lài giai. Đ t P (x) = x2 + 3x + 8 và xét bon nút n®i suy là x1 = −1; x2 = 1; x3 = 2; x4 = −3. Ta có a1 = 6, a2 = 12, a3 = 18, a4 = 8. Áp dụng công thác n®i suy Lagrange, ta có 1 3 6 1 f (x) = 2 (x + 1) − 2 (x − 1) + 5 (x − 2) − 5 (x + 3) . Tà đây, ta suy ra nguyên hàm của hàm so can tìm là ∫ f (x) dx = 1 ln |x + 1| − 3 ln |x − 1| + 6 ln |x − 2| − 1 ln |x + 3| + C, 2 2 5 5 trong đó C là hang so. Bước 2. Thực hi n phép chia P(x) cho định dạng đơn giản. (x − xi) ta sě được tích phân bat
  • 28. 24 Viết đề tài giá sinh viên – ZALO:0973.287.149-TEAMLUANVAN.COM 3 − Bài toán 2.4. Tìm nguyên hàm của hàm so x2 f(x) = . (x − 1) (x − 2) (x − 3) Lài giai. Đ t P(x) = x2 và xét ba nút n®i suy là x1 = 1; x2 = 2; x3 = 3. Ta có a1 = 1, a2 = 4, a3 = 9. Áp dụng công thác n®i suy Lagrange ta được 9 P (x) = − (x − 2) (x − 3) − 4 (x − 1) (x − 2) + Rút gon f(x) ta được hàm so sau. f(x) = −1 − 4 + 9 . 2 (x − 1) (x − 2) . 2(x − 1) (x − 2) 2(x − 3) Tà đây, ta suy ra nguyên hàm của hàm so can tìm là ∫ f (x) dx = − ln |x − 1| − 4 ln |x − 2| + 9 ln |x − 3| + C, trong đó C là m®t hang so. Bài toán 2.5. Tìm nguyên hàm của hàm so x2 5x + 10 f(x) = . (x − 1) (x − 2) (x + 1) (x + 2) Lài giai. Đ t P(x) = x2 −5x+10 và xét bon nút n®i suy là x1 = 1; x2 = 2; x3 = −1; x4 = −2. Ta có a1 = 6, a2 = 4, a3 = 16, a4 = 24. Áp dụng công thác n®i suy Lagrange ta được P (x) = − (x − 2) (x + 1) (x + 2) 1 + 3 (x − 1) (x + 1) (x + 2) 8 − 3 (x − 1) (x − 2) (x + 2) — 2 (x − 1) (x − 2) (x − 3) . Rút gon f(x) ta được hàm so sau. f (x) = −1 + 1 + 8 − 2 . (x − 1) 3 (x − 2) 3 (x + 1) (x + 2) Tà đây, ta suy ra nguyên hàm của hàm so can tìm là ∫ f (x) dx = − ln |x − 1| + 1 ln |x − 2| + 8 ln |x + 1| − 2 ln |x − 4| + C, 3 3 trong đó C là m®t hang so.
  • 29. 25 Viết đề tài giá sinh viên – ZALO:0973.287.149-TEAMLUANVAN.COM 5 − 5 3 4 35 2 4 7 2 3 Bài toán 2.6. Tìm nguyên hàm của hàm so x2 − 2x + 3 Lài giai. Đ t f(x) = 24x3 − 10x2 − 3x + 1 . Q(x) = 24x3 − 10x2 − 3x + 1 = 24(x − 1 )(x + 1 )(x − 1 ). 2 Khi đó ta đ t P(x) = x2 và xét ba nút n®i suy là x1 = 3 1 2 ; x2 4 = −1 ; x 3 1 = . Ta 4 có a1 = 9 4 , a2 = 34 41 9 , a3 = 16 . Áp dụng công thác n®i suy Lagrange ta được P (x) = 54 x + 1 x − 1 + 272 x − 1 x − 1 − 123 x − 1 x + 1 . Rút gon f(x) ta được hàm so sau. 54 272 123 f(x) = 1 5(x − 2 ) + 1 + x + 7 3 1 . x − 4 Tà đây, ta suy ra nguyên hàm của hàm so can tìm là ∫ f (x) dx = 54 ln |x trong đó C là m®t hang so. 1 — 2 | 272 + 35 ln |x 1 — 3 | − 123 7 ln |x x3 1 — 4 | + C, Bài toán 2.7. Tìm nguyên hàm của hàm so f(x) = x4 − 5x2 + 4 . Lài giai. Ta phân tích x4 − 5x2 + 4 = (x − 1)(x − 2)(x + 1)(x + 2). Khi đó ta đ t P (x) = x3 và xét bon nút n®i suy là x1 = 1; x2 = 2; x3 = −1; x4 = −2. Ta có a1 = 1, a2 = 8, a3 = −1, a4 = −8. Áp dụng công thác n®i suy Lagrange ta được P(x) = −1 (x 2)(x + 1)(x 6 1 + 2) + 2 2 3 (x − 1)(x + 1)(x + 2) — 6 (x − 1)(x − 2)(x + 2) + 3 (x − 1)(x − 2)(x + 1). Rút gon f(x) ta được hàm so sau. f(x) = −1 + 2 − 1 + 2 . 6(x − 1) 3(x − 2) 6(x + 1) 3(x + 2) 3
  • 30. 26 Viết đề tài giá sinh viên – ZALO:0973.287.149-TEAMLUANVAN.COM Q Y Y 1 1 · · · h Q(x) i(k) 1 Tà đây, ta suy ra nguyên hàm của hàm so can tìm là ∫ f (x) dx = −1 ln |x − 1| + 2 ln |x − 2| − 1 ln |x + 1| + 2 ln |x + 2| + C, 6 3 6 3 trong đó C là m®t hang so. Như v y, chúng ta thay rang đe tính nguyên hàm của hàm phân thác với các cực điem đơn thì ta sě sả dụng công thác n®i suy Lagrange đe tính. V y với hàm phân thác có các cực điem b c tùy ý ta sě tính nguyên hàm như the nào. Câu trả lời sě được trình bày trong phan tiep theo. 2.2 Nguyên hàm của hàm phân thfíc v i các cfic điem b c tùy j Ta sả dụng công thác n®i suy Hermite đe tính nguyên hàm với hàm phân thác với các cực điem b c tùy ý. Trong mục này, ta sě giới thi u phương pháp tìm nguyên hàm của hàm so hǎu tỉ có dạng Ta có Q(x) W(x) dx, với W(x) = n j=1 (x − xj)rj với deg Q < r1 +r2 +· · ·+rn = N. n đ t W(x) = (x − xj)rj , j=1 n W1(x) = (x − xj)rj j=2 với ∀i, j = 1, 2, . . . , n. Ta de thay, Q(x) = α0 + α1 + · · · + αr1−1 + Q (x), W (x) và (x − x1)r (x − x1)r1−1 (x − x1) Q(x) W1(x) = α0 α1 + x − x1 α2 + (x − x )2 + + α{r1 − 1} (x − x1)r1−1 + Q1(x)(x − x1 )r1 với k!αk = W1(x) x=x1 ; k = 0, 1, . . . , r1 − 1, hay 1 h Q(x) i(k) x=x1 (x) 1 W k! ∫ αk = ; k = 0, 1, . . . , r1 − 1. (2.1)
  • 31. 27 Viết đề tài giá sinh viên – ZALO:0973.287.149-TEAMLUANVAN.COM Q n ∫ Σ − (x − x1)r (x − x1)r1−1 (x − x1) (x − xn)r (x − xn)rn−1 (x − xn) N®i dung phương pháp: - Bước 1: Xác định các nút n®i suy là nghi m đơn ho c nghi m b®i của mau thác. - Bước 2: Bieu dien đa thác đã cho theo các nút n®i suy theo công thác Hermite. - Bước 3: Tính các h so bang công cụ đạo hàm và bieu dien. ∫ Q(x) dx = n j=1 (x − xj)rj = ∫ α0 + α1 + · · · + αr1−1 dx + · · · + + ∫ β0 + β1 + · · · + βrn−1 dx. Sau đó, nguyên hàm này có the tính được nhờ bảng nguyên hàm của các hàm so thường g p. Tiep theo, ta xét m®t vài ví dụ minh hoa. Bài toán 2.8. Tìm nguyên hàm của hàm so f (x) = Lài giai. Áp dụng công thác n®i suy Hermite, ta có x2 + x + 1 (x − 1)3 (x + 2)2 . f (x) = −1 + 1 1 2 + + . 9(x + 2)2 3(x − 1)3 (x − 1)2 3 (x − 1) V y nguyên hàm của hàm so can tìm là ∫ f (x) dx = 1 − 1 − 1 + 2 ln |x − 1| + C, 9 (x + 2) trong đó C là hang so. Bài toán 2.9. Tính tích phân 6(x − 1)2 x − 1 3 I = Pn(x)dx , (x − a)n+1 trong đó Pn(x) là đa thác b c n của x. Lài giai. Áp dụng khai trien Taylor cho đa thác Pn(x) tại x = a : n (k) P (x) = Pn (a) (x a)k . n k! k=0 1
  • 32. 28 Viết đề tài giá sinh viên – ZALO:0973.287.149-TEAMLUANVAN.COM Σ − k!(n − k)(x − a)n−k n! k!(2017 − k)(x − 1)2017−k 2017! k=0 k=0 Khi đó I = ∫ Pn(x) dx = Σ Pn (k) (a) ∫ dx n−1 (x − a)n+1 (k) k=0 k! (n) (x − a)n−k+1 = Σ Pn (a) + Pn (a) ln |x − a| + C. Bài toán 2.10. Tính tích phân I = x2017dx (x − 1)2018 . Lài giai. Áp dụng khai trien Taylor cho đa thác P(x) = x2017 tại x = 1 : 2017 (k) P (x) = Pn (1) (x 1)k . k! Khi đó k=0 ∫ P(x) Σ 2017 P(k) (1) ∫ dx I = 2016 (x − 1)2018 dx = (k) k=0 k! (2017) (x − 1)2018−k = Σ P (1) + P (1) ln |x − 1| + C, với C là hang so tuỳ ý. Bài toán 2.11. Tìm nguyên hàm của hàm so 1 Lài giai. f(x) = (x2 − 5x + 6)4 . Bien đői (x2 − 5x + 6)4 = (x − 3)4 (x − 2)4 . Theo thu t toán Hermite áp dụng với mau so là các nghi m b®i ta có: 1 (x − 3)4(x − 2)4 = α0 (x − 3)4 + α1 (x − 3)3 + α2 (x − 3)2 + α3 (x − 3) β0 β1 β2 β3 + + + + . Ta có (x − 2)4 (x − 2)3 (x − 2)2 (x − 2) 1 (x − 2)4 = α0 + α1 (x − 3) + α2 (x − 3)2 + α3 (x − 3)3 n ∫
  • 33. 29 Viết đề tài giá sinh viên – ZALO:0973.287.149-TEAMLUANVAN.COM − − − − + + β0 + β1 + β2 + β3 (x − 3)4 . + α0 + α1 + α2 + α3 (x − 2)4 , 1 2 3 0 1 1 2 3 Trong đó (x − 2)4 (x − 2)3 (x − 2)2 1 (x − 2) α0 = (3 − 2)4 = 1; α = 1 h (x − 2)−4 i(1) = −4; 1! x=3 α = 1 h (x − 2)−4 i(2) = 10; 2! x=3 α = 1 h (x − 2)−4 i(3) = −20. Ta lại có 1 = β + β 3! (x − 2) + β x=3 (x − 2)2 + β (x − 2)3 + (x − 1)3 với (x − 3)4 (x − 3)3 (x − 3)2 1 (x − 3) β0 = (2 − 1)3 = 1; β = 1 h (x − 1)−4 i(1) = 4; 1! x=2 β = 1 h (x − 1)−4 i(2) = 10; 2! x=2 β = 1 h (x − 1)−4 i(3) = 20. 3! x=2 V y ta có the viet 1 (x − 3)4(x − 2)4 1 = (x 3)4 1 + −4 (x 3)3 4 10 + (x 3)2 10 + −20 (x 3) 20 Tà đó, suy ra + (x − 2)4 + (x − 2)3 + (x − 2)2 + (x − 2) . ∫ 1 dx = ∫ 1 dx (x2 − 5x + 6)4 (x − 3)4(x − 2)4 = ∫ ( 1 + −4 + 10 + −20 )dx (x − 3)4 (x − 3)3 (x − 3)2 (x − 3) 2 3
  • 34. 30 Viết đề tài giá sinh viên – ZALO:0973.287.149-TEAMLUANVAN.COM 1 2 0 1 + ∫ ( 1 + 4 + 10 + 20 )dx (x − 2)4 (x − 2)3 (x − 2)2 (x − 2) = −1 + 2 + −10 − 20 ln |x − 3| 3(x − 3)3 (x − 3)2 (x − 3) + −1 + −2 + −10 + 20 ln |x − 2| + C, 3(x − 2)3 (x − 2)2 (x − 2) = −1 + 2 + −10 + 20 ln |x − 2| 3(x − 3)3 (x − 3)2 (x − 3) |x − 3| + −1 + −2 + −10 + C, 3(x − 2)3 (x − 2)2 (x − 2) trong đó C là hang so. Bài toán 2.12. Tìm nguyên hàm của hàm so 1 f(x) = (x − 1)3(x − 2)4 . Lài giai. Theo thu t toán Hermite cho mau so có các nghi m b®i ta có: 1 (x − 1)3(x − 2)4 = α0 (x − 1)3 + α1 (x − 1)2 + α2 + (x − 1) β0 β1 β2 β3 + + + + . (x − 2)4 (x − 2)3 (x − 2)2 (x − 2) và 1 (x − 2)4 = α0 + α1 (x − 1) + α2 (x − 1)2 + + β0 + β1 + β2 + β3 (x − 1)3 . Trong đó (x − 2)4 (x − 2)3 (x − 2)2 (x − 2) 1 α0 = (1 − 2)4 = 1; α = 1 h (x − 2)−4 i(1) = 4; 1! x=1 α = 1 h (x − 2)−4 i(2) = 10. Ta lại có 1 = β + β 2! (x − 2) + β x=1 (x − 2)2 + β (x − 2)3 (x − 3)4 2 3
  • 35. 31 Viết đề tài giá sinh viên – ZALO:0973.287.149-TEAMLUANVAN.COM − (x − 1)3(x − 2)4 (x − 1)3 (x − 1)2 (x − 1) (x − 2)4 (x − 2)3 (x − 2)2 (x − 2) 1 2 3 + α0 + α1 + α2 (x − 2)4 , với (x − 1)3 (x − 1)2 (x − 1) 1 β0 = (2 − 1)3 = 1; β = 1 h (x − 1)−3 i(1) = −3; 1! x=2 β = 1 h (x − 1)−3 i(2) = 6; 2! x=2 β = 1 h (x − 1)−3 i(3) = −10. V y ta có the viet 1 3! x=2 1 4 10 (x − 1)3(x − 2)4 = (x − 1)3 + (x − 1)2 + (x − 1) Tà đó suy ra 1 + (x − 2)4 + −3 (x − 2)3 6 + (x − 2)2 + −10 . (x − 2) ∫ 1 dx = ∫ 1 + 4 + 10 + 1 + −3 + 6 + −10 dx, = −1 + −4 + 10 ln |x − 1| + −1 2(x − 1)2 (x − 1) 3(x − 2)3 + 3 + −6 − 10 ln|x − 2| + C, 2(x − 2)2 (x − 2) = −1 + −4 + 10 ln |x − 1| 2(x − 1)2 (x − 1) |x − 2| + −1 + 3 + −6 + C, 3(x − 2)3 2(x − 2)2 trong đó C là hang so. (x − 2) Bài toán 2.13. Tìm nguyên hàm của hàm so r f(x) = = q 2x2 2x + 5 (x − 1)3(x − 2)4 . Lài giai. Theo thu t toán Hermite, ta có 2x2 − 2x + 5 α0 α1 α2 (x − 1)3(x − 2)4 = (x − 1)3 + (x − 1)2 + (x − 1) +
  • 36. 32 Viết đề tài giá sinh viên – ZALO:0973.287.149-TEAMLUANVAN.COM − x=1 x=1 − x=2 x=2 x=2 + β0 + β1 + β2 + β3 (x − 1)3 , + α0 + α1 + α2 (x − 2)4 , 0 1 2 β0 β1 β2 β3 + + + + , (x − 2)4 (x − 2)3 (x − 2)2 (x − 2) và 2x2 2x + 5 2 (x − 2)4 = α0 + α1(x − 1) + α2(x − 1) + trong đó (x − 2)4 (x − 2)3 (x − 2)2 (x − 2) α = 2.1 − 2.1 + 5 (1 − 2)4 = 5; M t khác α = h (2x2 − 2x + 5)(x − 2)−4 i(1) α = h (2x2 − 2x + 5)(x − 2)−4 i(2) = 22; = 60. 2x2 2x + 5 2 (x − 1)3 = β0 + β1(x − 2) + β2(x − 2) + β3(x − 2) trong đó (x − 1)3 (x − 1)2 2.22 − 2.2 + 5 (x − 1) β0 = (2 − 1)3 = 9; β1 β2 β3 V y ta có the viet = h (2x2 − 2x + 5)(x − 1)−3 i(1) = h (2x2 − 2x + 5)(x − 1)−3 i(2) = h (2x2 − 2x + 5)(x − 1)−3 i(3) = 9; = 68; = −360. 2x2 − 2x + 5 5 22 60 (x − 1)3(x − 2)4 = (x − 1)3 + (x − 1)2 + (x − 1) 9 + (x − 2)4 + −21 (x − 2)3 68 + (x − 2)2 + −360 . (x − 2) 3
  • 37. 33 Viết đề tài giá sinh viên – ZALO:0973.287.149-TEAMLUANVAN.COM — − − (x − 1)3(x − 2)4 dx = (x − 1)3 + (x − 1)2 + (x − 1) (x − 2)4 (x − 2)3 (x − 2)2 (x − 2) Tà đó, suy ra ∫ 2x2 − 2x + 5 ∫ 5 22 60 + 9 + −21 + 68 + −360 dx = −5 + −22 + 60 ln |x − 1| 2(x − 1)2 (x − 1) + −3 + 21 + −68 −360 ln |x − 2| + C, (x − 2)3 2(x − 2)2 (x − 2) trong đó C là hang so. Bài toán 2.14 (Olympic sinh viên toàn quoc - 2001). Cháng minh rang ton tại so thực x ∈ (0, 1) sao cho 1 t2000dt (1 + t)(1 + t2) · · · (1 + t2001) = x2000 (1 + x)(1 + x2) · · · (1 + x2001) . Lài giai. Đ t f(t) = t2000 (1 + t)(1 + t2) · · · (1 + t2001) , t ∈ [0, 1]. Bài toán 2.15. Tìm nguyên hàm của hàm so r f(x) = = q 3x3 2x2 4x + 23 (x + 3)(x − 1)3 . Lài giai. Ta có q = (x 1)3 (x + 3) . r Theo thu¾t toán Hermite, ta viet q dưới dạng r a b c b q = x + 3 + x − 1 + (x − 1)2 + (x − 1)3 , trong đó a, b, c, d là các hang so. Đong nhat các h so ta có h phương trình sau a + b = 3 −3a + b + c = −2 3a − 5b − 2c + d = −4 −a + 3b − 3c + 3d = 23. ∫ x
  • 38. 34 Viết đề tài giá sinh viên – ZALO:0973.287.149-TEAMLUANVAN.COM — − − ∫ ∫ Giải h ta có a = 1, b = 2, c = −1 và d = 5. Do đó Tà đó suy ra r 1 = q x + 3 2 + x − 1 + −1 (x − 1)2 5 + (x − 1)3 . ∫ r dx = ∫ 1 dx + ∫ 2 dx + ∫ −1 dx + ∫ 5 dx. q x + 3 x − 1 (x − 1)2 (x − 1)3 V y, nguyên hàm của hàm so can tìm là ∫ r dx = ln |x + 3| + 2ln|x − 1| + 1 + −5 + C, q trong đó C là m®t hang so. Bài toán 2.16. Tìm nguyên hàm của hàm so x − 1 2(x + 1)2 r f(x) = = q 7x3 34x2 + 96x 54 (x − 2)3(3x + 1)2 . Lài giai. Ta có q = (x 2)3 (3x + 1)2 . r Theo thu¾t toán Hermite, ta viet q dưới dạng r a b c d e q = 3x + 1 + (2x + 1)2 + x − 2 + (x − 2)2 + (x − 2)3 , trong đó a, b, c, d, e là các hang so. Đong nhat thác, ta thu được h phương trình 9a + 3d = 0 −30a + 9b − 17d = 7 13a − 12b + 9c + 30d + e = −34 20a + 11b + 6c − 12d − 14e = 96 4a − 2b + c − 8d + 4e = −54. Giải h phương trình này ta được a = 0, b = 0, c = 2; d = 0 và e = 7. Do đó r 2 7 q = (x − 2)3 + (3x + 1)2 . Ta có 2 dx = −1 + C và 7 dx = −7 + C , với (x − 2)3 C1, C2 là các hang so. Tà đó suy ra (x − 2)2 (3x + 1)2 3(3x + 1) 2 ∫ −1 7 −7x2 + 5x − 4 f(x)dx = với C là m®t hang so. (x − 2)2 + 2(3x + 1) = 3(x − 1)2(3x + 1) + C, 1
  • 39. 35 Viết đề tài giá sinh viên – ZALO:0973.287.149-TEAMLUANVAN.COM − ∫ Bài toán 2.17. Tìm nguyên hàm của hàm so x5 − x4 + 3x3 − 3x2 + x + 3 f(x) = x4 − 2x3 + 3x2 − 2x + 1 . Lài giai. Ta có f(x) = x5 − x4 + 3x3 − 3x2 + x + 3 x4 − 2x3 + 3x2 − 2x + 1 2x3 − x2 + 4x + 2 Đ t = x + 1 + r 2x3 x2 + 4x + 2 q = (x2 − x + 1)2 . (x2 − x + 1)2 . r Theo thu¾t toán Hermite, ta viet q dưới dạng r ax + b cx + d q = x2 − x + 1 + (x2 − x + 1)2 , trong đó a, b, c, d là các hang so. Đong nhat hai ve, ta được h phương trình a = 2 −a + b = −1 a + b + c = 4 b + d = 2. Giải h phương trình này ta được a = 2, b = 1, c = 3 và d = 1. Tà đó suy ra r 3x + 1 2x + 1 q = (x2 − x + 1)2 + x2 − x + 1 . 2x + 1 Bây giờ, ta sě tìm x2 − x + 1 dx. Ta có ∫ 2x + 1 dx = ∫ 2x − 1 dx + 2 ∫ 1 dx. x2 − x + 1 x2 − x + 2 (x − 1 )2 + 3 Tà đó suy ra ∫ 2x + 1 2 2 4 4 2x − 1 x2 − x + 1 dx = ln(x với C1 là hang so. — x + 1) + √ 3 arctan √ 3 + C1,
  • 40. 36 Viết đề tài giá sinh viên – ZALO:0973.287.149-TEAMLUANVAN.COM ∫ ∫ ∫ 3 3 − − − ∫ 2 ∫ 2 3x + 1 Tiep sau đây chúng ta sě tìm (x2 x + 1)2 dx. Theo thu¾t toán Hermite ta có 3x + 1 = t(x2 − x + 1) + (ux + v)(2x − 1), với t, u, v là các hang so. Đong nhat hai ve, ta được Tà đó suy ra 2 t = ; u = 3 5 ; v = −1 . 3 3 3x + 1 dx = (x2 − x + 1)2 2 5 1 = 3 dx + x − x + 1 ( x − )(2x − 1) (x2 − x + 1)2 dx. Ta de dàng tìm được nguyên hàm 2 3 dx = x − x + 2 4 3 √ 3 arctan 2x 1 √ 3 + C2, với C2 hang so. Áp dụng phương pháp tích phân tàng phan 5 1 2x − 1 (đ t u = x − và dv = dx) ta được 3 3 (x2 − x + 1)2 5 1 5 1 5 ∫ ( 3 x − 3 )(2x − 1) ( 3 x − 3 ) ∫ 3 (x2 − x + 1)2 dx = − x2 − x + 1 + x2 − x + 1 dx 5 1 ( 3 x − 3 ) 10 2x − 1 = − x2 − x + 1 + 3 √ 3 arctan với C3 là m®t hang so. Như v y, ta có √ 3 + C3, ∫ f(x)dx = ln(x2 4 — x + 1) + √ 3 arctan 2x 1 √ 3 + 5 1 4 2x − 1 ( 3 x − 3 ) 10 2x − 1 + 3 √ 3 arctan √ 3 − x2 − x + 1 + 3 √ 3 arctan √ 3 + C 2 5x − 1 26 2x − 1 = ln(x − x + 1) − 3(x2 − x + 1) + 3 √ 3 arctan √ 3 + C, với C là m®t hang so. Tiep theo, ta xét m®t so ví dụ ve tính toán các nguyên hàm của m®t so lớp hàm tőng quát.
  • 41. 37 Viết đề tài giá sinh viên – ZALO:0973.287.149-TEAMLUANVAN.COM ∫ − ∫ = − ∫ ∫ h ∫ Bài toán 2.18. Tính tích phân I = dx . n (ax2 + bx + c)n Lài giai. Đưa tam thác b c hai ve dạng chính tac ax2 + bx + c = a x + b 2 + 2a 4ac − b2 4a2 = a[t2 + ∆] b trong đó t = x + ; ∆ = 2a Khi đó 4ac b2 4a2 . dx In (ax2 + bx + c)n 1 = an dt . (t2 + ∆)n • Neu ∆ = 4ac b2 4a2 = 0 thì In 4ac − b2 1 = an dt t2n 1 = an(1 − 2n)t2n−1) + C. • Neu ∆ = 4a2 /= 0 thì ta có M t khác 1 In = an dt (t2 + ∆)n 1 = an Jn. J = dt n (t2 + ∆)n 1 t = + 2n − 3 J 2(n − 1)∆ (t2 + ∆)n − 1 mà I = 1 dt 2(n − 1)∆ 1 = J n−1 . n−1 an−1 (t2 + ∆)n − 1 an−1 n−1 Suy ra Jn−1 = an−1 In−1 nên J = dt n (t2 + ∆)n 1 = 2(n − 1)∆ và t . (t2 + ∆)n−1 + 2n − 3 2(n − 1)∆ an−1 In−1 In = 1 an Jn i ∫ ∫ ∫
  • 42. 38 Viết đề tài giá sinh viên – ZALO:0973.287.149-TEAMLUANVAN.COM ∫ ∫ ∫ ∫ ∫ | | a sin x + b cos x + c a sin x + b cos x + c 1 = 2an(n − 1)∆ t . (t2 + ∆)n−1 + 2n − 3 2a(n − 1)∆ an−1 In−1. Đây là công thác truy hoi cho phép tính các tích phân I2, I3, . . . sau khi biet dx I1 = ax2 + bx + c . Bài toán 2.19. Tính tích phân I = a1 sin x + b1 cos x dx. a sin x + b cos x Lài giai. Ta có a1 sin x + b1 cos x = A(a sin x + b cos x) + B(a sin x + b cos x)′. Suy ra I = A(a sin x + b cos x) + B(a sin x + b cos x)′ dx a sin x + b cos x =Ax + B ln |a sin x + b cos x| + C. Bài toán 2.20. Tính tích phân I = a1 sin x + b1 cos x + c1 dx. a sin x + b cos x + c Lài giai. Ta có a1 sin x + b1 cos x + c1 = A(a sin x + b cos x + c)+ + B(a sin x + b cos x + c)′ + C. Khi đó I =A ∫ dx + B ∫ d(a sin x + b cos x + c) + C ∫ dx =Ax + B ln a sin x + b cos x + c + C dx . a sin x + b cos x + c Với tích phân dx , a sin x + b cos x + c ∫
  • 43. 39 Viết đề tài giá sinh viên – ZALO:0973.287.149-TEAMLUANVAN.COM − ∫ ∫ ∫ ∫ ∫ ∫ n x đ t tan 2 Do đó = t ta được dx = 2dt 1 + t2 ; sin x = 2t 1 + t2 ; cos x = 1 t2 1 + t2 . dx a sin x + b cos x + c Bài toán 2.21. Tính tích phân = 2dt . (c − b)t2 + 2at + b + c Lài giai. Đ t In I = dx dx. (a + b cos x)n dx = (a + b cos x)n . Ta có In−2 = dx (a + bcosx)n−2 = (a + b sin x)dx (a + b cos x)n−1 = aIn−1 + b d(sin x) . (a + b cos x)n−1 Ta có sin x In−2 = aIn−1 + b (a + b cos x)n−1 − (n − 1) b2 sin2 xdx (a + b cos x)n . Khi đó b2 sin2 x = −(a2 − b2 ) + 2a(a + b cos x) − (a + b cos x)2 . I = −b sin x + (n − 1)(a2 − b2)(a + b cos x)n−1 + (2n − 3)a I (n − 1)(a2 − b2) Asin x n−1 (n − 2) I (n − 1)(a2 − b2) n−2, với In = + BIn−1 + CIn−2, (a + bcos x)n−1 A = −b ; B = (2n − 3)a ; (n − 1)(a2 − b2) (n − 1)(a2 − b2) C = n − 2 . (n − 1)(a2 − b2) ∫ ∫ −
  • 44. 40 Viết đề tài giá sinh viên – ZALO:0973.287.149-TEAMLUANVAN.COM ∫ ∫ x x x2 e Bài toán 2.22. Tính tích phân I = ∫ Lài giai. Ta có Pn(x)dx √ ax2 + bx + c . 2Pn(x) = 2Rn−2(x)(ax2 + bx + c) + Qn−1(x)(2ax + b) + 2α. Đong nhat h so hai ve ta được Rn−2(x), Qn−1(x), α hay Pn(x) √ = Q ′ (x) √ ax2 + bx + c+ Qn √ −1(x)(2ax + b) +√ α . ax2 + bx + c Suy ra n−1 2 2a2 + bx + c 2a2 + bx + c I = ∫ = Q Pn(x)dx √ ax2 + bx + c (x) √ ax2 + bx + c + α √ dx . n−1 Bài toán 2.23. Tính tích phân ax2 + bx + c P (x)dx I = Q(x) √ ax2 + bx + c . Nh n xét 2.4. Đe tính tích phân này ta thực hi n như sau: P (x) - Phân tích phân thác Q(x) thành các phân thác toi giản. - Tách thành các tích phân tương áng và chon các phương pháp thích hợp đe tính các tích phân tàng phan. Bài toán 2.24. Tìm đieu ki n đoi với đa thác P(x) đe tích phân I = ∫ P 1 ex dx là m®t hàm sơ cap và xét các bài toán tương tự . . . Lài giai. Ta có ∫ P 1 dx = ∫ a0 + a1 + · · · + an 1 n x = a0ex + a1 1 ex dx + a x 2 ∫ 1 ex dx + · · · + a 1 ex dx. xn x ∫ ∫ dx n
  • 45. 41 Viết đề tài giá sinh viên – ZALO:0973.287.149-TEAMLUANVAN.COM ∫ x ∫ x xn − 3 3 2x2 2x 2 ( ) x 0 2 2! x 3 3! x2 1 1! 2! 3! (n − 1)! 1 Kí hi u In Đ t 1 = e dx, ta tính I xn n−1 1 = e dx. xn−1 Suy ra u = 1 xn−1 dv = ex dx du = − n − 1 dx Khi đó v = ex . I = 1 ex + (n − 1) ∫ 1 ex dx = 1 ex + (n − 1)I hay n−1 xn−1 xn xn−1 n I = − 1 ex + 1 I . Tà đó suy ra (n − 1)xn−1 n − 1 n−1 a2I2 = a2 ex + a I x 2 1 a I = − a3 ex − a3 ex + a3 I a I = − a4 ex − a4 ex − a4 ex + a4 I 4 4 3x3 . . . . . . . . . 6x2 6x 6 1 Suy ra ∫ P 1 ex dx = ex h a − a + a3 + . . . 1 − — a + a4 + . . . i 1 − . . . i + + a + a2 + a3 + a4 + · · · + an I . Đe ∫ P ( 1 )ex dx là m®t hàm sơ cap thì can x a2 a3 a4 an a + + + + · · · + = 0. 1 1! 2! 3! (n − 1)! n 1
  • 46. 42 Viết đề tài giá sinh viên – ZALO:0973.287.149-TEAMLUANVAN.COM (A − B)b = b 1 . ∫ ∫ − a e + be + c xm = (ln x) + C ; m, n ∈ N, n ≥ 2. ∫ a1ex + b1e−x + c1 dx + B Bài toán 2.25. Tính tích phân In = lnn xdx xm . Lài giai. Bang cách l p lu n tương tự bài toán trên ta có the tính được ∫ lnn xdx 1 Bài toán 2.26. Tính tích phân Lài giai. Phân tích I = aex + be−x + c dx. aex + be−x + c = A(a1ex + b1e−x + c1) + B(a1ex − b1e−x ) + C, A, B, C được xác định tà h Khi đó ∫ a1ex + b1e−x + c1 (A + B)a1 = a Ac1 + C = c. I = aex + be−x + c dx ∫ ∫ d(a1ex + b1e−x + c1) ∫ dx = Ax + B ln .a1 ex + b1e−x + c1 + C dx . a1ex + b1e−x + c1 dx Đoi với tích phân x x 1 1 1 là tam thác b c hai . ta đ t t = ex chuyen ve tích phân có mau 1 e−x + c 1 ex + b 1 a 1 1 1 n xm−1P ∫ In = = A a ex + b e−x + c + hspace ∗ 6cm + C
  • 47. Viết đề tài giá sinh viên – ZALO:0973.287.149-TEAMLUANVAN.COM n x 43 Chương 3. M t so dạng toán liên quan Công thác n®i suy Lagrange và Hermite có nhieu áng dụng thú vị. N®i dung của chương này sě trình bày m®t so áng dụng của công thác n®i suy Lagrange và Hermite. 3.1 M t so bài toán ve đa thfíc nh n giá trị nguyên Ta xét m®t so bài toán ve đa thác nh n giá trị nguyên ho c với h so nguyên. Bài toán 3.1. Đa thác x(x − 1) · · · (x − n + 1) P(x) = n! được ký hi u bởi P (x) = x là đa thác nh n giá trị nguyên. Lài giai. Nh n xét rang tích của n so tự nhiên liên tiep thì chia het cho n!. Giả sả x(x − 1) · · · (x − n + 1) là tích của n so tự nhiên liên tiep (x ≥ n). Khi đó ta có là m®t so nguyên. x(x − 1) · · · (x − n + 1) n! = Cn Tà đó, ta suy ra tích của n so nguyên liên tiep thì chia het cho n! và do đó Bài toán 3.1 được giải quyet. Nh n xét 3.5. Với n ≥ 2 cho ta ket quả sau. Ton tại đa thác nh n giá trị nguyên với các h so có the không bat bu®c là nhǎng so nguyên. Với n ≥ 2, ton tại m®t đa thác với các h so đeu hǎu t không nguyên nhưng nh n giá trị nguyên tại các điem nguyên. Bài toán 3.2. Đa thác P(x) b c n nh n giá trị nguyên tại moi điem nguyên khi và chỉ khi P(x) nh n giá trị nguyên tại (n + 1) điem nguyên liên tiep. Lài giai. Đieu ki n can là hien nhiên. Đieu ki n đủ. Sả dụng công thác khai trien Abel với xi = a+i (i = 1, 2, . . . , n), ta được P(x) = b0 + b1(x − a − 1) + b2(x − a − 1)(x − a − 2)+
  • 48. 44 Viết đề tài giá sinh viên – ZALO:0973.287.149-TEAMLUANVAN.COM 1 n 1 1 k + bn(x − a − 1) · · · (x − a − n). Ta có P(a + 1) ∈ Z nên b0 ∈ Z, P(a + 2) ∈ Z nên b0 + b1 ∈ Z và b1 ∈ Z, P(a + 3) ∈ Z nên b0 + 2b1 + 2!b2 ∈ Z và 2!b2 ∈ Z. Tương tự, ta thu được P(a + n) ∈ Z nên (n − 1)!bn ∈ Z, P(a) ∈ Z nên n!bn ∈ Z. Tà đó ta có k!bk ∈ Z (k = 0, 1, . . . , n), đieu phải cháng minh (vì tích của n so nguyên liên tiep thì chia het cho k!). Nh n xét 3.6. Thực ra, ta chỉ can đieu ki n P (x) là đa thác nh n giá trị nguyên khi và chỉ khi P (x) nh n các giá trị nguyên khi x = 0, 1, . . . , n ho c P (x) nh n các giá trị nguyên tại n + 1 giá trị nguyên liên tiep là đủ. Bài toán 3.3. a) Cháng minh rang moi đa thác b c n đeu có the bieu dien được dưới dạng Pn(x) = b0 + b1 x ! + · · · + bn−1 x n − 1 ! + bn x ! . (3.1) b) Đa thác Pn(x) là đa thác nh n giá trị nguyên khi và chỉ khi tat cả các h so b0, b1, . . ., bn là các so nguyên. Lài giai. a) Cháng minh bang quy nạp. Giả sả đa thác P(x) b c n có dạng P(x) = anxn + an−1xn−1 + · · · + a1x + a0. Khi n = 1 thì P1(x) = a1 x + a0 = a0 + a1 x ! . Chon b0 = a0, b1 = a1 ta nh n được (3.1) đúng. Giả sả (3.1) đúng với moi n ≤ k. Khi đó ta có Pk+1(x) = ak+1xk+1 + akxk + · · · + a1x + a0 = ak+1 (k + 1)! x k + 1 ! + Rk (x), deg Rk (x) ≤ k. Theo giả thiet quy nạp thì Rk(x) = b0 + b1 x ! + · · · + bk−1 x k − 1 ! + bk x ! .
  • 49. 45 Viết đề tài giá sinh viên – ZALO:0973.287.149-TEAMLUANVAN.COM 1 k 1 1 2 1 n 1 1 ! Do đó Pk+1 (x) = b0 + b1 x ! + · · · + b x ! + bk+1 x k + 1 , với bk+1 = ak+1(k + 1)!. Do đó (3.1) đúng với k + 1. b) Đieu ki n can. Giả sả Pn(x) là đa thác nh n giá trị nguyên với x ∈ Z. Khi đó P(0) = b0, P(1) = b0 P(2) = b0 + b1 + b1 1 ! , 2 ! + b 2 ! , . . . . . . . . . P(n) = b0 + b1 n ! + · · · + b n ! . Vì P(0), P(1), . . ., P(n) ∈ Z và các h so nhị thác Newton đeu nguyên nên b0, b1, . . . , bn ∈ Z. Đieu ki n đủ. Giả sả b0, b1, . . . , bn là các so nguyên. Khi đó P (0), P (1), . . . , P (n) cũng là các so nguyên nên theo Bài toán 3.2 thì Pn(x) là đa thác nh n giá trị nguyên. Bài toán 3.4. Neu đa thác P(x) b c n là đa thác nh n giá trị nguyên thì đa thác Q(x) = n!P(x) ∈ Z[x]. Lài giai. Theo Bài toán 3.3 thì P(x) là đa thác b c n nh n giá trị nguyên nên ta có bieu dien Pn(x) = b0 + b1 x ! + · · · + bn−1 x n − 1 ! + bn x ! , Do đó bj ∈ Z (j = 0, . . . , n). n!P(x) = n!b0 + n!b1 x ! + · · · + n!bn−1 x n − 1 ! + n!bn x ! . n n k 2 n
  • 50. 46 Viết đề tài giá sinh viên – ZALO:0973.287.149-TEAMLUANVAN.COM k 3 2 1 k Ta có b k! x ! = b x(x − 1)...(x − k + 1) ∈ Z. Vì v y n!P(x) ∈ Z[x]. Bài toán 3.5. Cho đa thác P(x) = anxn + an−1xn−1 + · · · + a0. Giả sả ton tại h so ak ∈ {a0, a1, . . . , an} sao cho n!ak /∈ Z thì P (x) không phải là m®t đa thác nh n giá trị nguyên. Lài giai. Giả sả P (x) là đa thác nh n giá trị nguyên. Theo Bài toán 3.4 thì n!P (x) ∈ Z[x]. Đieu này dan đen mâu thuan với giả thiet. V y P (x) không là đa thác nh n giá trị nguyên. Bài toán 3.6. Cháng minh rang đa thác f(x) = 1 x3 − 5 x2 + 19 x + 3 3 2 6 là m®t đa thác nh n giá trị nguyên. Lài giai. Ta viet f(x) dưới dạng f(x) = 2 x ! − 3 x ! + x ! + 3. Do đó theo Bài toán 3.3 thì f(x) là đa thác nh n giá trị nguyên. Bài toán 3.7. Cho đa thác x2002 2001 P(x) = 2003 + a2001x + · · · + a1x + a0, ai ∈ Z (i = 0, 1, . . . , 2001). Cháng tỏ rang ton tại x0 ∈ Z sao cho P(x0) /∈ Z với (tác P(x) không phải là m®t đa thác nh n giá trị nguyên). Lài giai. De dàng nh n thay ngay rang 1 (2002!) 2003 /∈ Z. V y theo Bài toán 3.5, thì P(x) không phải là m®t đa thác nh n giá trị nguyên. k
  • 51. 47 Viết đề tài giá sinh viên – ZALO:0973.287.149-TEAMLUANVAN.COM Bài toán 3.8. Xác định các so dương A, B, C sao cho đa thác f(x) = Ax5 + Bx3 + Cx là đa thác nh n giá trị nguyên với f(3) nh n giá trị nhỏ nhat. Lài giai. Do A, B, C dương và f (x) ∈ Z khi x ∈ Z nên f (1), f (2), f (3) là nhǎng so nguyên dương. Ta có f (1) = A + B + C, f(2) = 32A + 8B + 2C, f(3) = 243A + 27B + 3C. Suy ra f(2) − 2f(1) = 30A + 6B. (3.2) Sả dụng công thác n®i suy Lagrange, ta có bieu dien A = [f(3) − 4f(2) + 5f(1)], (3.3) B = [−f(3) + 8f(2) − 13f(1)], (3.4) C = [f(3) − 9f(2) + 45f(1)]. (3.5) Vì f(1) nguyên dương nên f(1) ≥ 1. Tà (3.2) do A, B dương và f(2), f(1) là nguyên dương nên f(2) − 2f(1) ≥ 1. Suy ra Tà (3.6) suy ra Suy ra f(2) ≥ 2f(1) + 1 ≥ 3. (3.6) f(3) = 120A + 4f(2) − 5f(1) = 120A + 4[f(2) − 2f(1)] + 3f(1). f(3) > 4[f(2) − 2f(1)] + 3f(1) = 4 + 3 = 7 và vì v y f(3) ≥ 8. Với f(1) = 1; f(2) = 3; f(3) = 8 thì tà (3.2), (3.3), (3.4) ta thu được 1 A = 120 , B = 1 13 , = 8 15 . (3.7) Ta cháng tỏ với các giá trị A, B, C ở (3.7) thì f(x) nguyên khi x nguyên.
  • 52. 48 Viết đề tài giá sinh viên – ZALO:0973.287.149-TEAMLUANVAN.COM 120 8 15 5 3 Th t v y, ta có f(x) = 1 x5 + 1 x3 + 13 x = x + 2 ! + x + 1 ! là m®t đa thác nh n giá trị nguyên. Bài toán 3.9. Cho đa thác P(x) = anxn + an−1xn−1 + · · · + a0 thỏa mãn đieu ki n P(x) ∈ Q với moi x ∈ Z. Cháng minh rang P(x) ∈ Q[x] (hay ak ∈ Q với moi k ∈ {0, . . . , n}). Lài giai. Dùng bieu dien Abel của P(x), ta có P(x) = b0 + b1x + b2x(x − 1) + · · · + bnx(x − 1) · · · (x − n + 1). Lan lượt cho x = 0, 1, . . . , n − 1 ta thu được các bi ∈ Q hay P(x) ∈ Q[x]. Bài toán 3.10. Cho f(x) = 1 ax + b ∈ Q với moi x ∈ Z. Cháng minh rang ax + b ∈ Q[x] (hay a, b ∈ Q). 1 Lài giai. Do f(x) = ∈ Q với moi x ∈ Z. nên ax + b 1 ax + b = với moi x ∈ Z. f(x) V y theo Bài toán 3.9 thì ax + b ∈ Q[x]. Bài toán 3.11. Cho hàm phân thác ax + b f(x) = ∈ Q với moi x ∈ Z. cx + d Cháng minh rang f(x) có the bieu dien được dưới dạng Ax + B f(x) = với A, B, C, D ∈ Z. (3.8) Cx + D Lài giai. Neu ad − bc = 0 thì f(x) = const (hang so) và bieu dien (3.8) là hien nhiên.
  • 53. 49 Viết đề tài giá sinh viên – ZALO:0973.287.149-TEAMLUANVAN.COM 0 0 0 Xét trường hợp ad − bc /= 0. Neu c = 0 thì bieu dien (3.8 là hien nhiên. Neu c /= 0 thì sả dụng phân tích 1 f(x) − f(0) = ta sě thu được ngay bieu dien (3.8). αx + β Bài toán 3.12. Cho f(x) là đa thác với h so thực nh n giá trị hǎu tỉ với moi so x hǎu t và giá trị vô t với moi so vô t . Cháng minh rang f (x) là đa thác tuyen tính với h so hǎu t . Lài giai. 1) Ta cháng minh rang các h so của f(x) là hǎu tỉ. Ta cháng minh bang quy nạp theo b c n của f(x). Với n = 0, f(x) là hang so và nó là m®t so hǎu t (chȁng hạn bang f(0)) Giả sả khȁng định đúng với tat cả các đa thác b c nhỏ thua so tự nhiên n (thỏa đieu ki n đe bài). Đ t f(x) = a0xn + a1xn−1 + · · · + an−1x + an. De thay an = f(0) là hǎu t . Đ t g(x) = a0xn−1 + a1 xn−2 + . . . + an−1 = f(x) − an x thì g(x) sě nh n giá trị hǎu t với bien hǎu tỉ x. Theo giả thiet qui nạp nhǎng so a0, a1, . . . , an−1 là hǎu tỉ. Như v y các h so của f(x) là các so hǎu tỉ. Với đieu đó f(x) không là hang so, vì trong trường hợp ngược lại f(x) sě là hǎu t với moi x. Cho f(x) = a0xn + a1xn−1 + · · · + an, n > 0. không mat tính tőng quát có the cho rang ai là nguyên, ngoài ra đa thác g(x) = an−1 (f(x) − an) = (a0x)n + a1(a0x)n−1 + . . . + an−1an−2 (a0x) nghĩa là đa thác h(y) = yn + a1yn−1 + · · · + an−1an−2 y, y = a0x, thỏa mãn đieu ki n đe bài. Ta sě cháng minh rang với moi so nguyên đủ lớn m, phương trình h(y) = m có nghi m. Th t v y, lay m > h(0) và ϕ(y) = h(y) − m. Khi đó ϕ(0) < 0 và
  • 54. 50 Viết đề tài giá sinh viên – ZALO:0973.287.149-TEAMLUANVAN.COM ⇔ − − ⇒ − − √ 3 2 (x 1) 3 + 6 (x 1) = 2 + 3(x 1) 2 — − − ≤ · · · − √ 3 lim y→∞ ϕ(y) = +∞. Vì the phương trình ϕ(y) = 0 hay h(y) = m có nghi m dương ym. Lay m = p là so nguyên to đủ lớn, ta có h(yp) = p. Tà giả thiet yp là so hǎu t và vì h so cao nhat của h(y) là 1, thì yp nguyên và ngoài ra yp được chia het bởi so hạng tự do của ϕ(y) ho c là yp là ước so của p. Nghĩa là yp = 1 ho c là yp = p. Nhưng đȁng thác yp = 1 chỉ có khả năng nhieu nhat với m®t p, nghĩa là yp = p cho tat cả so nguyên to đủ lớn p. Nói cách khác, ta đã nh n được h(p) = p với tat cả so nguyên to đủ lớn. Đieu này chỉ xảy ra với h(y) = y và nghĩa là f(x) = a0x + a1. 3.2 M t so bài toán xác định đa thfíc 3.2.1 Tìm đa thfíc khi biet các nghi m của nó. Bài toán 3.13. Tìm tat cả các đa thác P (x) ∈ Z[x] nh n x = 1 + √ 2 + √ 3 3 làm nghi m. Cháng minh rang deg P (x) ≥ 6. Lài giai. Ta có x = 1 + √ 2 + √ 3 3 x 1 √ 3 3 = √ 2 x 1 √ 2 3 = √ 3 3 ⇔ (x − 1) − 2 √ 2 − 3 (x − 1) √ 2 (x − 1) − √ 2 = 3 ⇔ − − − − 3 2 2 2 ⇒ (x − 1) − 3 + 6 (x − 1) = 2 + 3(x − 1) .2 Rút gon ta nh n được đa thác P (x) = x6 6x5 + 9x4 2x3 + 9x2 60x + 50. Giả sả ton tại Q (x) = a5x5 +a4x4 +a3x3 +a2x2 +a1x+a0 nh n x = 1+ √ 2+ √ 3 3 làm nghi m và deg Q (x) 5. Khí đó ta có Q 1 + √ 2 + √ 3 3 = = k0+k1 √ 2+ k2 √ 3 3 + k3 √ 3 9 + k4 √ 2 √ 3 3 + k5 √ 2 √ 3 9 = 0 không the xảy ra. V y ta có đieu phải cháng minh. Bài toán 3.14. Xét t p hợp các đa thác P (x) khác hang, thỏa mãn đieu ki n P x2 − 1 = P (x) .P (−x) , ∀x ∈ R. Hãy tìm trong t p hợp đó 1 đa thác có b c bé nhat nhưng có nghi m lớn nhat. Lài giai. Do P (x) khác hang nên ta xét các trường hợp sau: +) Trường hợp 1: Khi deg P (x) = 1 thì P (x) = ax + b, a /= 0. Đong nhat h so trong giả thiet ta nh n được a = 1, b = −1 ± 5 . 2
  • 55. 51 Viết đề tài giá sinh viên – ZALO:0973.287.149-TEAMLUANVAN.COM 0 2 2 2 44 ∈ Σ Khi đó, xét các đa thác P (x) = −x + −1 + √ 5 ∨ P (x) = −x + −1 − √ 5 . 2 Ta thay các đa thác này có nghi m lan lượt là b = −1 ± √ 5 . 2 +) Trường hợp 2: Neu deg P (x) ≥ 2 và giả sả P (x0) = 0. Khi đó, P x2 − 1 = 0 hay x2 − 1 là nghi m của P (x). 0 √ Neu x > −1 + 5 thì x2 −1 > x > −1 + √ 5 . Khi đó P (x) có vô so nghi m, 0 2 vô lý. 0 0 2 √ V y neu deg P (x) ≥ 2 có nghi m thì các nghi m của nó đeu nhỏ hơn −1 + 5 . Tà đó suy ra đa thác can tìm là P (x) = −x + −1 + √ 5 . 2 Bài toán 3.15. Tìm tat cả các đa thác b c 4 dạng P (x) = x4 +bx2 +c, (b, c > 0) sao cho P (x)−x2 = 0 không có nghi m thực nhưng P (P (x))− x4 = 0 có nghi m thực. Lài giai. Giả sả ton tại đa thác b c 4 dạng P (x) = x4 + bx2 + c, (b, c > 0). Ta có P (x) − x2 = 0 không có nghi m thực hay x4 + (b − 1) x2 + c = 0 không có nghi m thực nên suy ra Q (x) = x4 +(b − 1) x2 + c > 0, ∀x. Suy ra P (x) > x2 và P (P (x)) > (P (x)) > x = x hay phương trình P (P (x)) − x = 0 không có nghi m thực, trái với giả thiet của bài ra. V y không ton tại đa thác nào thỏa mãn yêu cau bài toán. Bài toán 3.16. Tìm tat cả các đa thác P (x) R [x] có b c n, có n nghi m thực và thỏa mãn P (x) .P 2x2 = P 2x3 + x , ∀ ∈ R. Lài giai. Giả sả P (x) = n i=0 aixi , an /= 0. Ta có P (0) = a0 theo giả thiet a0.a0 = a0 thì a0 = 1 ∨ a0 = 0. +) Trường hợp 1: Xét a0 = 0. Khi đó P (x) = xk Q (x) , k ∈ N, k ≥ 1, Q (x) ∈ R [x] , Q (0) /= 0. Thay vào giả thiet, ta thu được xk Q (x) .2k x2k Q 2x2 = 2x3 + x k Q 2x3 + x nên Q (x) .2k x2k Q 2x2 = 2x2 + 1 k Q 2x3 + x . Suy ra Q (0) = 0, vô lý. +) Trường hợp 2: Khi a0 = 1. 2 2
  • 56. 52 Viết đề tài giá sinh viên – ZALO:0973.287.149-TEAMLUANVAN.COM 0 Σ n n .Σ i i n | | ≤ n n n 0 0 Giả sả x0 là nghi m của P (x) , suy ra 2x3 + x0 là nghi m của P (x) . M t khác, ta có .2x3 + x0. = |x0| .2x2 + 1. ≥ |x0| , tà đó suy ra P (x) ∈ R [x] có vô so nghi m thực, vô lý. V y không ton tại đa thác nào thỏa mãn yêu cau bài toán. Bài toán 3.17. Tìm tat cả các đa thác P (x) ∈ Z [x], là monic b c 2, sao cho ton tại đa thác Q (x) ∈ Z [x] mà các h so của đa thác R (x) = P (x) Q (x) đeu thu®c t p {−1; 1}. Lài giai. Do P (x) ∈ Z [x] là monic b c 2 nên P (x) = x2 + ax ± 1. Giả sả R (x) = n i=0 aixi ; ai ∈ {−1; 1}. Goi z ∈ C là m®t nghi m của R (x) sao cho |z| > 1. Khi đó n−1 |z| = |z | = . ai zi . ≤ n−1 |z| = n−1 |z| = |z| − 1 . Do đó . i=0 an . n i=0 i=0 |z| − 1 z n |z| − 1 |z| − 1 ⇔ |z| (|z| − 1) ≤ |z| − 1 ⇔ |z| (|z| − 1) ≤ −1 nên |z| < 2. V y moi nghi m của R (x) đeu có modul nhỏ hơn 2. Giả sả z1, z2 là 2 nghi m của P (x) ∈ Z [x] thì z1, z2 là 2 nghi m của R (x) . Suy ra |z1| ; |z2| < 2, lại có theo định lý Viet thì |z1z2| = |z1| |z2| = 1 nên |z1| ≥ |z2| 1 ≤ |z1| < 2 0 ≤ |z2| ≤ 1 M t khác |a| = |z1 + z2| ≤ |z1| + |z2| < 3 nên a ∈ {±2; ± 1; 0}. +) Trường hợp 1: Với a = 0, ta được Q (x) = x + 1 P (x) = x2 − 1 Q (x) = 1 P (x) = x2 + 1 Σ Σ ( ( ∨
  • 57. 53 Viết đề tài giá sinh viên – ZALO:0973.287.149-TEAMLUANVAN.COM " Σ Σ ( n n 2 2 +) Trường hợp 2: Với a = ±1, ta được P (x) = x2 ± x ± 1 nên Q (x) = 1. P (x) = x2 ± 2x + 1 ⇒ Q (x) = x ± 1 +) Trường hợp 3: Với a = ±2 thì P (x) = x2 ± 2x − 1 ⇒ Q (x) do ∃zo , |zo | > 2 3.2.2 Sfi dnng công thfíc n i suy Lagrange đe xác định h so của đa thfíc. Bài toán 3.18. Cho đa thác P (x) = ax2 +bx+c, (a = / 0). Cháng minh rang ton tại không quá 1 đa thác Q (x) b c n thoả mãn đieu ki n P (Q (x)) = Q (P (x)). Lài giai. Giả sả ton tại Q1 (x) = Ta có n i=1 2 aixi ; Q2 (x) = n i=1 bixi . Σ n i P (Q1 (x)) = Q1 (P (x)) ⇔ aQ1 P (Q2 (x)) = Q2 (P (x)) ⇔ aQ2 (x) + bQ1 (x) + c = 2 (x) + bQ2 (x) + c = i=1 Σ n aiP biP (x), (1) . i (x) , (2) So sánh h so của x2n trong (1) , (2) , ta được i=1 a.a2 = a .an a.b2 n = bn.an ⇔ an = an−1 bn = an−1 nên an = bn. Tà đó, ta đ t R (x) = Q1 (x) − Q2 (x) , suy ra deg (R (x)) ≤ n − 1. Ta có R (P (x)) = Q1 (P (x)) − Q2 (P (x)) = P (Q1 (x)) − P (Q2 (x)) = aQ1 (x) + bQ1 (x) + c − aQ2 (x) + bQ2 (x) + c = (Q1 (x) − Q2 (x)) (Q1 (x) + Q2 (x) + b) = R (x) .T (x) , (∗) . Nh n thay trong (∗) deg R (x) = k nên deg (V T) = kn, deg (V P) = k + n; (k < n) không xảy ra. V y ta có đieu phải cháng minh. Bài toán 3.19. Cho 4 so nguyên to khác nhau p1, p2, p3, p4. Cháng minh rang không ton tại đa thác Q (x) b c 3 có h so nguyên thỏa mãn đieu ki n |Q (p1)| = |Q (p2)| = |Q (p3)| = |Q (p4)| = 3. (
  • 58. 54 Viết đề tài giá sinh viên – ZALO:0973.287.149-TEAMLUANVAN.COM " ⇔ Lài giai. Giả sả ton tại đa thác Q (x) b c 3 có h so nguyên thỏa mãn |Q (p1)| = |Q (p2)| = |Q (p3)| = |Q (p4)| = 3. Ta xét các trường hợp sau Trường hợp 1: Q (pi) = 3, i = 1, 2, 3, 4 Xét Q (x) = P (x) − 3 ta có deg (P (x)) = 3. Suy ra P (pi) = 6, i = 1, 2, 3, 4 v y P (x) ≡ 0. Vô lý Trường hợp 2: Khi 3 trong 4 giá trị Q (pi) , i = 1, 2, 3, 4 bang nhau. Giả sả Q (pi) = −3, i = 1, 2, 3, Q (p4) = 3 Q (x) = a (x − p1) (x − p2) (x − p3) − 3; a ∈ Z Ta có ⇒ Q (p4) = 3 = a (p4 − p1) (p4 − p2) (p4 − p3) − 3 ⇒ a (p4 − p1) (p4 − p2) (p4 − p3) = 6 Ta thay tích của 4 so nguyên khác nhau=6, vô lý. Trường hợp 3: Trong 4 giá trị Q (pi) , i = 1, 2, 3, 4 có 2 giá trị bang 3 và 2 giá trị bang -3 Giả sả Q (pi) = −3, i = 1, 2, Q (p4) = Q (p3) = 3 Ta có Q (x) = (x − p1) (x − p2) G (x)−3 ⇒ Q (p3) = (p3 − p1) (p3 − p2) G (p3)− 3 = 3, suy ra (p3 − p1) (p3 − p2) G (p3) = 6, vô lý. Vì neu p1 = 2 thì (Q (p3) − Q (2)) . (p3 − 2) |6 suy ra p3 ∈ {3; 5}, tương tự p4 ∈ {3; 5}. Tà đó suy ra p3 = 3, p4 = 5. Lại có (p4 − p1) (p4 − p2) |6 nên (5 − p2) |2 suy ra p2 ∈ {3; 7} và p2 = 7. Khi đó (p3 − p1) (p3 − p2) = −4, vô lý. Tương tự, với p2 = 2; p3 = 2; p4 = 2, vô lý. Suy ra các so pi là lẻ nên (p3 − p1) (p3 − p2) .4; (p4 − p1) (p4 − p2) .4, vô lý. V y ta có đieu phải cháng minh. Bài toán 3.20. Tìm tat cả các đa thác P(x) với h so thực thoả mãn phương trình P x2 = P2 (x) với moi x thu®c R. Lài giai. Với P (x) = C, C là hang so. Khi đó ta có C = C2 C = 0 C = 1 V y P (x) = 0; P (x) = 1. Với P (x) = anxn + · · · + a1x + a0 với các h so thực không đong thời bang 0. Khi đó ta có P x2 = an x2 +· · ·+a x2 +a ; P2 (x) = [a xn + · · · + a x + a ]2 n 0 n 1 0 Giả sả k là so lớn nhat bé hơn n sao cho ak 0. Ta đong nhat h so của xn+k ta được 0 = 2anak. Tái với giả thiet. Do đó P (x) = anxn . So sánh h so của x2n ta nh n được an = 1 1
  • 59. 55 Viết đề tài giá sinh viên – ZALO:0973.287.149-TEAMLUANVAN.COM 2 2 2 2 2 2 Khi đó (∗) trở thành Q x2 = Q2 (x) 2 2 V y P (x) = xn . Bài toán 3.21. Tìm tat cả các đa thác P (x) thỏa mãn h thác P x2 − 2x = [P (x − 2)]2 với moi giá trị thực của x. Lài giai. Tà giả thiet của bài toán ta được P (x − 1)2 − 1 = [P ((x − 1) − 1)]2 . (∗) Đ t Q (x) = P (x − 1) ⇒ Q x2 = P x2 − 1 ; Q2 (x) = P2 (x − 1) Áp dụng ket quả của Bài toán 3.20 ta được Q (x) = 0, Q (x) = 1, Q (x) = xn Do đó ta được ket quả P (x) = 0, P (x) = 1, P (x) = (x + 1)n . Bài toán 3.22. Tìm tat cả các đa thác P (x) h so thực thỏa mãn P x2 + x [3P (x) + P (−x)] = P 2 (x) + 2x2 với moi giá trị thực của x. Lài giai. Tà giả thiet của bài toán tat hay x bởi −x ta nh n được P x2 − x [3P (−x) + P (x)] = P 2 (−x) + 2x2 . Trà ve với ve phương trình cho nhau, ta nh n được [P (x) + P (−x)] [P (x) − P (−x) − 4x] = 0. Khi P (x) = −P (−x). Thay vào giả thiet ban đau ta được P x2 − x2 = [P (x) − x] . Đ t Q (x) = P (x) − x khi đó ta được Q x = Q (x). Theo ket quả của Bài toán 3.20, ta có Q (x) = 0, Q (x) = 1, Q (x) = xn hay P (x) = x, P (x) = x + 1, P (x) = xn + x. So sánh với P (x) − P (−x) = 0 ta nh n được P (x) = x, P (x) = x2k+1 + x. Khi P (x)−P (−x)−4x = 0 Thay vào giả thiet ban đau ta được P x −2x = [P (x) − 2x] . Đ t Q (x) = P (x) − 2x, khi đó ta được Q x = Q (x). Theo ket quả của Bài toán 3.20, ta có Q (x) = 0, Q (x) = 1, Q (x) = xn hay P (x) = 2x, P (x) = 2x+1, P (x) = xn +2x. Đoi chieu với P (x)−P (−x)−4x = 0 ta nh n được P (x) = 2x, P (x) = x2k + 2x.
  • 60. 56 Viết đề tài giá sinh viên – ZALO:0973.287.149-TEAMLUANVAN.COM Σ ( 3.2.3 M t so bài toán xác định đa thfíc khác không liên quan đen các công thfíc n i suy. Bài toán 3.23. Tìm tat cả các đa thác P (x) ∈ Z [x] thỏa mãn (P (a) + P (b) + P (c)) . (a + b + c) với moi so nguyên a, b, c. Lài giai. Ta luôn có (a + b + c) |(P (a) − P (−b − c)) lại có theo giả thiet (a + b + c) |(P (a) + P (b) + P (c)) tà đó suy ra (a + b + c) |(P (b) + P (c) + P (−b − c)) đúng với moi so nguyên a, b, c. Co định so nguyên b, c. Khi đó ta có (a + b + c) |(P (b) + P (c) + P (−b − c)) với moi so nguyên a. Suy ra ton tại so nguyên a đủ lớn đe (a + b + c) |(P (b) + P (c) + P (−b − c)) và a + b + c > P (b) + P (c) + P (−b − c), (hien nhiên ton tại vô so so nguyên a như the) tà đó ta suy ra P (b) + P (c) + P (−b − c) ≡ 0 với moi c p so nguyên b, c. Chon b = c ta được 2P (b) = −P (−2b). Giả sả P (x) = n i=0 aixi . Đong nhat h so của xn ta nh n được 2anxn + an(−2x)n = 0 ⇒ n = 1 ⇒ P (x) = mx + k Theo giả thiet (a + b + c) |(m (a + b + c) + 3k) ⇒ k = 0 V y P (x) = mx, m ∈ Z. Bài toán 3.24. Tìm tat cả các đa thác P (x) ∈ Z [x] thỏa mãn P (7) .5, P (5) .7 và P (12) không chia het cho 35. Lài giai. Giả sả ton tại P (x) ∈ Z [x] thỏa mãn P (7) .5, P (5) .7 Ta có (P (12) − P (7)) .5 ⇒ P (12) .5 . (P (12) − P (5)) .7 ⇒ P (12) .7 ⇒ P (12) .35, trái với giả thiet của bài toán. V y không ton tại đa thác nào thỏa mãn yêu cau bài toán. Bài toán 3.25. Tìm tat cả các đa thác P (x) b c n thỏa mãn đieu ki n P x2 − y2 = P (x + y) .P (x − y) , ∀x, y ∈ R. Lài giai. Tà giả thiet của bài toán ta có đieu ki n tương đương là P (xy) = P (x) .P (y) , ∀x, y ∈ R.
  • 61. 57 Viết đề tài giá sinh viên – ZALO:0973.287.149-TEAMLUANVAN.COM x x P Cho x = y = 0 ⇒ P (0) = 0 ∨ P (0) = 1. +) Trường hợp 1. Neu P (0) = 0 khi đó P (x) = xQ (x) , deg Q = n − 1 thay vào P (xy) = P (x) .P (y) , ∀x, y ∈ R. ta nh n được Q (xy) = Q (x) .Q (y) , ∀x, y ∈ R. suy ra Q (x) = 1 ∨ Q (x) = x.Q1 (x) , deg Q1 = n − 2 Ta nh n được P (x) = 1 ∨ P (x) = xn . +) Trường hợp 2. Neu P (0) = 1, cho y = 0 thay vào P (xy) = P (x) .P (y) , ∀x, y ∈ R. ta được P (x) = 1. Thả lại các ket quả ta thay đa thác này thỏa mãn yêu cau bài ra. Bài toán 3.26. Tìm tat cả các đa thác P (x) ∈ R [x] thỏa mãn đieu ki n P (x + y) = P (x) + P (y) + 2xy, ∀x, y ∈ R. Lài giai. Xét x = y = 0 ⇒ P (0) = 0, tà giả thiet của bài toán ta có P′ (x) = lim P (x + y) − P (x) P (y) + 2xy = lim = 2x + P′ (0) . y→0 y ∫x ′ y→0 y 2 Thả lại ta thay đa thác P (x) = x2 + ax thỏa mãn yêu cau bài toán. Bài toán 3.27. Tìm tat cả các đa thác P1 (x) , P2 (x) , P3 (x) , P4 (x) sao cho với moi x, y, z, t ∈ R thỏa mãn xy − zt = 1 thì P1 (x) P2 (y) − P3 (z) P4 (t) = 1. Lài giai. Giả sả deg P1 = n1, deg P2 = n2. Goi N ∈ N là so tự nhiên có so các ước ϕ (N) lớn hơn n1 + n2. N Chon y = được x , (x |N ) ; z = 1; t = N − 1 thay vào giả thiet của bài toán ta Nh n thay P1 (x) P2 N − P (1) P4 (N − 1) = 1. (∗) deg P1 (x) P2 N ≤ n + n2 < ϕ (N) . (∗∗) M t khác tà (∗) ta suy ra Q (x) = P1 (x) P2 có ϕ (N) > n + n N 2 nghi m. V y deg Q (x) = 0 tà đó suy ra P1 (x) = axn ; P2 (x) = bxn . Hoàn toàn tương tự ta cũng có ket quả P3 (x) = cxm ; P4 (x) = dxm . +) Cho x = y = 1; z = t = 0 ta nh n được ab = 1. 0 x Do đó P (x) = P (x) − P (0) = (t) dt = x + ax. 3 1 1
  • 62. 58 Viết đề tài giá sinh viên – ZALO:0973.287.149-TEAMLUANVAN.COM +) Cho x = y = 0; z = t = 1 ta nh n được cd = 1. +) Cho x = z = 1; t = y − 1 ta nh n được (yn − (y − 1)m = 0, ∀y ∈ R) ⇔ m = n = 1. V y các đa thác nh n được là: P1 (x) = ax; P2 (x) = x x a , (a /= 0) ; P3 (x) = cx; P4 (x) = c , (c /= 0). Thả lại thay thỏa mãn giả thiet của bài toán.
  • 63. 59 Viết đề tài giá sinh viên – ZALO:0973.287.149-TEAMLUANVAN.COM KET LU N Lu n văn ”M®t so áng dụng của công thác n®i suy Lagrange và Hermite” đã trình bày được nhǎng van đe sau: 1. Lu n văn trình bày các bài toán n®i suy Lagrange và n®i suy Hermite 2. Tiep theo, trình bày m®t so áng dụng của công thác n®i suy Lagrange và Hermite đe tìm nguyên hàm của các hàm phân thác. 3. Xét các bài toán ve đa thác nh n giá trị nguyên, các bài toán ve xác định đa thác.
  • 64. 60 Viết đề tài giá sinh viên – ZALO:0973.287.149-TEAMLUANVAN.COM TÀI LI U THAM KHẢO [A] Tieng Vi t [1] Nguyen Văn M u (2002), Đa thúc đại so và phân thúc hũu ty, NXB Giáo dục. [2] Nguyen Văn M u (2016), N®i suy đa thúc, NXB ĐHQG Hà N®i. [3] Nguyen Văn M u, Lê Ngoc Lăng, Phạm the Long, Nguyen Minh Tuan (2006), Các đe thi olympic Toán sinh viên toàn quoc, NXB Giáo dục. [4] Nguyen Thúy Vân (2014), M®t so thu¾t toán n®i suy đe xác đ nh các nguyên hàm sơ cap của hàm hũu ty, Lu n văn Thạc sy, ĐH Thái Nguyên. [B] Tieng Anh [5] Paulo Ney de Sausa, Jorge- Nume Silva (1998), Berkeley Problems in Math- ematics, Springer.